Kvanttifysiikan olemuksesta

MindOverMaterial

Vaatiiko kvanttifysiikka havaitsijan?
Kysymys liittyy alkuräjähdyksen "ensimmäiseen" hetkeen. Jos nimittäin kvanttifysiikka pitää paikkansa, ja tietoisuus romahduttaa aaltofunktion tarvitaan jokin tietoisuus, joka romahduttaa alkuräjähdyksen aaltofunktion.

Vaatiko siis alkuräjähdysteoria tietoisuutta?

60

797

    Vastaukset

    Anonyymi (Kirjaudu / Rekisteröidy)
    5000
    • Antirealism
    • Kvanttimystikko

      Kyllä tärähtää metafysiikan puolelle ja vahvasti.

    • FirstMatter

      1) Aaltofunktion romahdusta ei välttämättä edes ole objektiivisesti olemassa, koska jos kvanttifysiikan monimaailmatulkinta pitää paikkansa, niin eri vaihtoehtoiset kvanttifysikaaliset tapahtumat tapahtuvat joka tapauksessa, vaikka vain yksi niistä siis havaitaan suoraan tässä maailmankaikkeudessa. Tämäkin voi kuulostaa metafysiikalta, mutta ei sitä ole, koska kyse on pohjimmiltaan yksinkertaisesti vain siitä, että hiukkasia on tuossa tapauksessa oikeasti huomattavasti enemmän kuin mitä niitä kerralla voidaan havaita. Monimaailmatulkintaa voidaan myös testata empiirisesti tarkkailemalla lomittuneiden kvanttien käyttäytymistä, joka vastaa täysin sitä, että kullakin alkeishiukkasella olisi lukemattomia keskenään vuorovaikuttavia vastineita käytännön kannalta lähes erillisissä tila-avaruuksissa. Aaltofunktion romahtamisen olettavien tulkintojen eräs ongelma on se, että ne eivät pysty esittämään objektiivista syytä sille, miksi aaltofunktio romahtaisi nimenomaan mitatulla tavalla; eli esimerkiksi hiukkasten sijainnit noudattavat tilastollisesti siistiä todennäköisyysjakaumaa, kun taas monimaailmatulkinnassa tuota ongelmaa ei ole, koska kaikki mahdolliset alkeisvaihtoehdot siis toteutuvat sen mukaan, koska hiukkanen ja sen ei-lokaalit vastineet vuorovaikuttavat keskenään; eli esimerkiksi fotonin vastineet siroavat toisistaan kaksoisrakokokeessa. Tietty havaitsija havaitsee niistä vain yhden rajoitteistaan johtuen. Syvällinen syy tuohon rajoittuneisuuteen on hiukan antrooppisesti se, että ainakaan järjestäytynyt havaitseminen ei olisi ylipäätään mahdollista jos suoraan havaittavissa olevien hiukkasten massa olisi suurempi kuin mikä mahdollistaisi havaitsijan muodostumisen. Eli, jos kaikki hiukkaset sijaitsisivat samassa tila-avaruudessa, niin niiden massa romahduttaisi maailmankaikkeuden alkuräjähdyksessä hiukkasten liike-energiasta huolimatta ilmeisesti välittömästi takaisin pienimpään mahdolliseen tilaansa. Tila-avaruus onkin siis ilmeisesti repeytynyt jo aivan ajan alussa lukemattomiksi lähes erillisiksi osiksi; eli rinnakkaisiksi maailmankaikkeuksiksi.

      2) Vaikka aaltofunktio romahtaisi objektiivisesti, niin ei sen romahtamisella olisi välttämättä mitään tekemistä tietoisuuden kanssa, koska sen romahtamiseen riittänee mikä tahansa hiukkasten välinen vuorovaikutus kuten esimerkiksi termodynaamisesti palautumaton lämpöliike. Tämähän on hyvin tuttua esimerkiksi kvanttitietokoneista, joiden toteutuksen perusongelma on juuri se, että kvanttien superpositiot eivät pysy kasassa laskennallisen hyödynnettävyyden kannalta riittävän kauan kuin erikoisjärjestelyillä, joihin kuuluu mm. jäähdytys jopa asteen sadasosan päähän absoluuttisesta nollapisteestä (-273.14 C). Perinteinen kvanttimystiikka ei siis ole kelvollinen selitysmalli tietoisuudelle ja tietoisuuden kokemus on luontevimmin selitettävissä aineen osasten liikkeillä. Aineperusteisten selitysmallien vahvuutena on myös se, että ne ovat liitettävissä objektiivisiin mittauksiin.

      3) Tietoisuus on oletettavasti yksinkertaisesti seurausta prosesseista joissa aineen rakennetta ja sen liikkeitä kuvaava informaatio ohjaa tuon samaisen aineen rakennetta ja sen liikkeitä entropiaa paikallisesti vähentävällä tavalla, mikä lisää sen todennäköisyyttä, että paikallisesti tapahtuu sellaista havaitsemista, jonka tulokset myös tallentuvat rakenteita muuttavalla tavalla. Tämä on mahdollista jos informaatio etenee riittävän nopeasti, jotta se ehditään prosessoida tuollaista lisäarvoa tuottavalla tavalla siten, että se ehtii myös vaikuttaa noihin ohjattaviin prosesseihin. Mielenkiintoiseksi tämä entropia-tarkastelu muodostuu kuitenkin erityisesti mm. tietoisuuden kannalta vasta, kun tarkasteluun otetaan koko todellisuus, eli myös paikallinen vuorovaikutus rinnakkaisten maailmankaikkeuksien kanssa. Kuten aiemmin todettiin, hyödynnettävissä olevaa kvanttimekaanista vuorovaikutusta tapahtuu vain hyvin kylmissä lämpötiloissa, joissa taas toisaalta ei esiinny elämää, mutta kuitenkin tietyssä merkityksessä lokaalisti identtiset maailmankaikkeudet ovat erittäin todennäköisesti hyvin samankaltaisia myös muiden niiden paikallisesti hyvin läheisten rakenteellisten vastineidensa osalta, jotka ohjaavat tuota rakenteita ohjaavaa tietojenkäsittelyä. Kukaan ei kaiketi vielä oikein tiedä tarkalleen, mitä tästä seuraa, mutta en olisi hämmästynyt, jos se osoittautuisi oleelliseksi tietoisuuden entistäkin paremman luonnontieteellisen selittämisen kannalta...

      • metafysiikkaa

        "1) jos kvanttifysiikan monimaailmatulkinta pitää paikkansa, niin eri vaihtoehtoiset kvanttifysikaaliset tapahtumat tapahtuvat joka tapauksessa, vaikka vain yksi niistä siis havaitaan suoraan tässä maailmankaikkeudessa."

        Niin JOS. Monimaailmatulkinta voi tuntua houkuttelevalta selitysvaihtoehdolta koska silloin ei tarvitse vaivata päätään tietoisuuden vaikutuksella.

        "Tämäkin voi kuulostaa metafysiikalta, mutta ei sitä ole, koska kyse on pohjimmiltaan yksinkertaisesti vain siitä, että hiukkasia on tuossa tapauksessa oikeasti huomattavasti enemmän kuin mitä niitä kerralla voidaan havaita. "

        Se ei vain kuullosta metafysiikalta vaan nimenomaan on silkkaa metafysiikkaa (metafysiikka= aistihavaintojen ja mittausvälineiden ulottumattomissa)

        "Monimaailmatulkintaa voidaan myös testata empiirisesti tarkkailemalla lomittuneiden kvanttien käyttäytymistä, joka vastaa täysin sitä, että kullakin alkeishiukkasella olisi lukemattomia keskenään vuorovaikuttavia vastineita käytännön kannalta lähes erillisissä tila-avaruuksissa. "

        Monimaailmatulkinta ei ole tieteellinen väite koska se ei ole kumottavissa havainnoin ja mittauksin koska siinä mallissa kaikki mahdollinen tapahtuu eikä siis mikään kumoa mitään.

        "Aaltofunktion romahtamisen olettavien tulkintojen eräs ongelma on se, että ne eivät pysty esittämään objektiivista syytä sille, miksi aaltofunktio romahtaisi nimenomaan mitatulla tavalla."

        Onko monimaailmatulkinnassa sitten mitään objektiivista syytä? Käsite "objektiivinen" edellyttää että on olemassa jokin havaitsija tai tietoisuus jolle ko. asia on objektiivinen. Objektia ei voi olla ilman subjektia eli tietoisuutta eli objektiivisuus aina edellyttää subjektin eli tietoisen havaitsijan tai kokijan olemassaolon.

        "kaikki mahdolliset alkeisvaihtoehdot siis toteutuvat sen mukaan, koska hiukkanen ja sen ei-lokaalit vastineet vuorovaikuttavat keskenään"

        Kun kaikki mahdolliset vaihtoehdot toteutuvat niin mikään ei kelpaa todisteeksi tai kumoamiseksi yhtään mistään...

        "Syvällinen syy tuohon rajoittuneisuuteen on hiukan antrooppisesti se, että ainakaan järjestäytynyt havaitseminen ei olisi ylipäätään mahdollista jos suoraan havaittavissa olevien hiukkasten massa olisi suurempi kuin mikä mahdollistaisi havaitsijan muodostumisen. "

        Tuo väite sisältää sen hyvin metafyysisen oletuksen että tietoisuus on vain jonkinlainen järjestäytynyt hiukkaskokonaisuus joka edellyttää tietyn minimimäärän hiukkasia.
        Mitä tulee ns. alkuräjähdykseen niin toisaalta tietoisuuttakaan ei voi olla olemassa olemassa ilman jotain sen tietoisuuden ulkopuolella olevaa eli ei-minää. Jos alkuräjähdys on kaiken olevaisen alkuhetki niin on vaikea kuvitella tietoisuutta ilman jotain kokemisen tai havainnon objektia (maailmankaikkeus). Voisi oikeastaan sanoa että tietoisuuden ilmaantuminen edellyttää maailmankaikkeuden ilmaantumista.

        2) "Vaikka aaltofunktio romahtaisi objektiivisesti, niin ei sen romahtamisella olisi välttämättä mitään tekemistä tietoisuuden kanssa, koska sen romahtamiseen riittänee mikä tahansa hiukkasten välinen vuorovaikutus kuten esimerkiksi termodynaamisesti palautumaton lämpöliike. "

        Ei olekaan. Toisaalta fysiikkaa (eikä mitään muutakaan tiedettä) ei olisi olemassa ilman tietoisuutta. Tietoisuus on kaiken havainon ja mittaamisen ehdoton ja välttämätön edellytys.
        "Perinteinen kvanttimystiikka ei siis ole kelvollinen selitysmalli tietoisuudelle ja tietoisuuden kokemus on luontevimmin selitettävissä aineen osasten liikkeillä. "
        Kaikki mahdollinen fysiikka edellyttää tietoisuuden mutta mikään mahdollinen fysiikka ei riitä selittämään tietoisuutta. Koetapa palauttaa kaikki fysiikan matemaattiset kaavat hiukkasten liikkeisiin aika-avaruudessa. Onnistuuko?

        Tietoisuus liitetään elävien olentojen toimintaan ja elävät olennot ovat fysikaalisesti avoimia systeemeitä jotka ylläpitävät rakennettaan imemällä matalaa entropiaa ympäristöstään ja samalla poistamalla korkeampaa entropia omasta kehostaan. Pelkkä aineen liike tuskin riittää tietoisuuden selityksen pohjaksi mutta jonkinlainen lupaava suunta voisi olla tietoisuuden hahmottaminen jonkinlaisena monimutkaisena avoimien systeemeiden takaisinkytkentäprosessina (epälineaarinen kaoottinen systeemi ks esim. Prigogine).
        "hyödynnettävissä olevaa kvanttimekaanista vuorovaikutusta tapahtuu vain hyvin kylmissä lämpötiloissa,"

        Tuo on hyvin kiistanalaista (esim. fotosynteesin taustalla monet olettavat olevan kvanttivaikutuksia).


      • FirstMatter
        metafysiikkaa kirjoitti:

        "1) jos kvanttifysiikan monimaailmatulkinta pitää paikkansa, niin eri vaihtoehtoiset kvanttifysikaaliset tapahtumat tapahtuvat joka tapauksessa, vaikka vain yksi niistä siis havaitaan suoraan tässä maailmankaikkeudessa."

        Niin JOS. Monimaailmatulkinta voi tuntua houkuttelevalta selitysvaihtoehdolta koska silloin ei tarvitse vaivata päätään tietoisuuden vaikutuksella.

        "Tämäkin voi kuulostaa metafysiikalta, mutta ei sitä ole, koska kyse on pohjimmiltaan yksinkertaisesti vain siitä, että hiukkasia on tuossa tapauksessa oikeasti huomattavasti enemmän kuin mitä niitä kerralla voidaan havaita. "

        Se ei vain kuullosta metafysiikalta vaan nimenomaan on silkkaa metafysiikkaa (metafysiikka= aistihavaintojen ja mittausvälineiden ulottumattomissa)

        "Monimaailmatulkintaa voidaan myös testata empiirisesti tarkkailemalla lomittuneiden kvanttien käyttäytymistä, joka vastaa täysin sitä, että kullakin alkeishiukkasella olisi lukemattomia keskenään vuorovaikuttavia vastineita käytännön kannalta lähes erillisissä tila-avaruuksissa. "

        Monimaailmatulkinta ei ole tieteellinen väite koska se ei ole kumottavissa havainnoin ja mittauksin koska siinä mallissa kaikki mahdollinen tapahtuu eikä siis mikään kumoa mitään.

        "Aaltofunktion romahtamisen olettavien tulkintojen eräs ongelma on se, että ne eivät pysty esittämään objektiivista syytä sille, miksi aaltofunktio romahtaisi nimenomaan mitatulla tavalla."

        Onko monimaailmatulkinnassa sitten mitään objektiivista syytä? Käsite "objektiivinen" edellyttää että on olemassa jokin havaitsija tai tietoisuus jolle ko. asia on objektiivinen. Objektia ei voi olla ilman subjektia eli tietoisuutta eli objektiivisuus aina edellyttää subjektin eli tietoisen havaitsijan tai kokijan olemassaolon.

        "kaikki mahdolliset alkeisvaihtoehdot siis toteutuvat sen mukaan, koska hiukkanen ja sen ei-lokaalit vastineet vuorovaikuttavat keskenään"

        Kun kaikki mahdolliset vaihtoehdot toteutuvat niin mikään ei kelpaa todisteeksi tai kumoamiseksi yhtään mistään...

        "Syvällinen syy tuohon rajoittuneisuuteen on hiukan antrooppisesti se, että ainakaan järjestäytynyt havaitseminen ei olisi ylipäätään mahdollista jos suoraan havaittavissa olevien hiukkasten massa olisi suurempi kuin mikä mahdollistaisi havaitsijan muodostumisen. "

        Tuo väite sisältää sen hyvin metafyysisen oletuksen että tietoisuus on vain jonkinlainen järjestäytynyt hiukkaskokonaisuus joka edellyttää tietyn minimimäärän hiukkasia.
        Mitä tulee ns. alkuräjähdykseen niin toisaalta tietoisuuttakaan ei voi olla olemassa olemassa ilman jotain sen tietoisuuden ulkopuolella olevaa eli ei-minää. Jos alkuräjähdys on kaiken olevaisen alkuhetki niin on vaikea kuvitella tietoisuutta ilman jotain kokemisen tai havainnon objektia (maailmankaikkeus). Voisi oikeastaan sanoa että tietoisuuden ilmaantuminen edellyttää maailmankaikkeuden ilmaantumista.

        2) "Vaikka aaltofunktio romahtaisi objektiivisesti, niin ei sen romahtamisella olisi välttämättä mitään tekemistä tietoisuuden kanssa, koska sen romahtamiseen riittänee mikä tahansa hiukkasten välinen vuorovaikutus kuten esimerkiksi termodynaamisesti palautumaton lämpöliike. "

        Ei olekaan. Toisaalta fysiikkaa (eikä mitään muutakaan tiedettä) ei olisi olemassa ilman tietoisuutta. Tietoisuus on kaiken havainon ja mittaamisen ehdoton ja välttämätön edellytys.
        "Perinteinen kvanttimystiikka ei siis ole kelvollinen selitysmalli tietoisuudelle ja tietoisuuden kokemus on luontevimmin selitettävissä aineen osasten liikkeillä. "
        Kaikki mahdollinen fysiikka edellyttää tietoisuuden mutta mikään mahdollinen fysiikka ei riitä selittämään tietoisuutta. Koetapa palauttaa kaikki fysiikan matemaattiset kaavat hiukkasten liikkeisiin aika-avaruudessa. Onnistuuko?

        Tietoisuus liitetään elävien olentojen toimintaan ja elävät olennot ovat fysikaalisesti avoimia systeemeitä jotka ylläpitävät rakennettaan imemällä matalaa entropiaa ympäristöstään ja samalla poistamalla korkeampaa entropia omasta kehostaan. Pelkkä aineen liike tuskin riittää tietoisuuden selityksen pohjaksi mutta jonkinlainen lupaava suunta voisi olla tietoisuuden hahmottaminen jonkinlaisena monimutkaisena avoimien systeemeiden takaisinkytkentäprosessina (epälineaarinen kaoottinen systeemi ks esim. Prigogine).
        "hyödynnettävissä olevaa kvanttimekaanista vuorovaikutusta tapahtuu vain hyvin kylmissä lämpötiloissa,"

        Tuo on hyvin kiistanalaista (esim. fotosynteesin taustalla monet olettavat olevan kvanttivaikutuksia).

        "Niin JOS. Monimaailmatulkinta voi tuntua houkuttelevalta selitysvaihtoehdolta koska silloin ei tarvitse vaivata päätään tietoisuuden vaikutuksella."

        Monimaailmatulkinta tuntuu houkuttelevalta ja elegantilta vaihtoehdolta mm., koska se selittää havainnot havaittujen hiukkastyyppien kautta ilman oletuksia tyhjästä syntyvistä vaikutuksista. Toisaalta, filosofit ovat vaivanneet päätään tietoisuuspohdinnoilla vuosituhansia, eivätkä tulokset ole olleet kovin kummoisia, joten ainakaan ei kannata pompata siihen johtopäätökseen, että tietoisuudella olisi mitään aineesta riippumatonta vaikutusta. Tietoisuuden vaikutusten tutkiminen voi olla enemmän biologian kuin fysiikan aihealueeseen kuuluvaa.

        "Se ei vain kuullosta metafysiikalta vaan nimenomaan on silkkaa metafysiikkaa (metafysiikka= aistihavaintojen ja mittausvälineiden ulottumattomissa)"

        Tuo on yleinen harhaluulo. Monimaailmatulkinta on havaintojen tulkinta aivan kuten muutkin kvanttimekaaniset tulkinnat, eli se perustuu havaintoihin. Se on kuitenkin itseasiassa tieteellisesti houkuttelevampi tulkinta, kuin jotkin muista tulkinnoista. Aiemmin jo selitettiin miten kaksoisrakokokeen tulokset ovat selitettävissä aukottomasti fotonien sironnan kautta. On hyvin tieteellistä, eleganttia ja käsitteellisesti säästeliästä selittää esimerkiksi fotoneita koskevat havainnot nimenomaan fotonien avulla sen sijaan, että oletettaisiin mystisiä satunnaisvaikutuksia, kuten joissakin muissa kvanttimekaanisissa tulkinnoissa. Kuitenkin, kuten aiemmin jo selitettiin, jos sironta tapahtuu todellisten hiukkasten välillä, niin niiden täytyy sijaita nimenomaan erillisissä tilaulottuvuuksissa, koska muuten niiden massa tuottaisi mustan aukon tässä ulottuvuudessa ja niitä ei silloin tietenkään voitaisi havaita. Miksi oletus siitä, että olemassa voisi olla vain yksi yhtenäinen tilaulottuvuus olisi muka vähemmän metafyysinen väite kuin oletus siitä, että fotonien sijainnit noudattavat nimenomaan tietyntyyppistä tilastollista jakaumaa satunnaisten syiden vuoksi? No,eihän se olekaan, vaan perinteisesti on vaan totuttu olettamaan tuollaisia.

        "Monimaailmatulkinta ei ole tieteellinen väite koska se ei ole kumottavissa havainnoin ja mittauksin koska siinä mallissa kaikki mahdollinen tapahtuu eikä siis mikään kumoa mitään."

        Teorioiden falsifioitavuus on tieteessä yksi tärkemmistä asioista, mutta eihän esimerkiksi kaksoisrakokokeessa monimaailmatulkinnan mukaan tapahdu mitä tahansa, vaan juuri sitä mitä havaitaan. Jos fotonit eri maailmankaikkeuksissa käyttäytyisivät mallin vastaisesti, eli esimerkiksi kulkisivat toistensa läpi sen sijaan, että ne siroaisivat toisistaan, jolloin ei muodostuisi havaittavaa interferenssiä, niin sehän kumoaisi monimaailmatulkinnan.

        "Onko monimaailmatulkinnassa sitten mitään objektiivista syytä? Käsite "objektiivinen" edellyttää että on olemassa jokin havaitsija tai tietoisuus jolle ko. asia on objektiivinen. Objektia ei voi olla ilman subjektia eli tietoisuutta eli objektiivisuus aina edellyttää subjektin eli tietoisen havaitsijan tai kokijan olemassaolon."

        Monimaailmatulkinnassa kvanttimekaanisten ilmiöiden kausaalisena syynä pidetään todellisia hiukkasia, eli hiukkasia, joilla on massaa ja/tai energiaa. Monimaailmatulkinnan kannattajat tulkitsevat havainnot siten, että niiden aiheuttajat ovat todellisia hiukkasia, toisin kuin joissakin muissa tulkinnoissa, joissa ilmeisesti oletetaan, että pelkät matemaattiset oliot voivat aiheuttaa havaittavia fysikaalisia vaikutuksia, jonkin tuntemattoman mekanismin kautta.

        "Kun kaikki mahdolliset vaihtoehdot toteutuvat niin mikään ei kelpaa todisteeksi tai kumoamiseksi yhtään mistään..."

        Kaikki mahdolliset vaihtoehdot toteutuvat kvanttilogiikan mukaan väistämättä. Tämähän on todistettu kvanttitietokoneiden avulla. Mahdottomat vaihtoehdot sen sijaan eivät toteudu, eli esimerkiksi yksikään fotoni ei päädy kaksoisrakokokeessa mustien raitojen kohdalle. Jos ne sinne päätyisivät, niin monimaailmatulkinta kumoutuisi tai sitä pitäisi muokata.

        "Tuo väite sisältää sen hyvin metafyysisen oletuksen että tietoisuus on vain jonkinlainen järjestäytynyt hiukkaskokonaisuus joka edellyttää tietyn minimimäärän hiukkasia."

        Tietoisuusteorioiden todentaminen edellyttää mittauksia ja objektiivisesti voidaan kätevimmin mitata hiukkasten ja niistä koostuvien rakenteiden ominaisuuksia. Vaikka tietoisuus ei olisikaan vain järjestäytynyt hiukkaskokonaisuus, niin meillä ei kuitenkaan ole havaintoja ainakaan tietoisuudesta, joka ei edellyttäisi järjestäytynyttä hiukkaskokonaisuutta, joten suoraviivaisimmalta tavalta tietoisuuden selittämiseen vaikuttaisi tuollaisten hiukkaskokonaisuuksien ominaisuuksien mittaukset. Tietysti tähän liittyy käytännöllinen mittausongelma, eli mitä lukemattomista muuttujista nimenomaan pitäisi tai kannattaisi mitata.


      • FirstMatter
        metafysiikkaa kirjoitti:

        "1) jos kvanttifysiikan monimaailmatulkinta pitää paikkansa, niin eri vaihtoehtoiset kvanttifysikaaliset tapahtumat tapahtuvat joka tapauksessa, vaikka vain yksi niistä siis havaitaan suoraan tässä maailmankaikkeudessa."

        Niin JOS. Monimaailmatulkinta voi tuntua houkuttelevalta selitysvaihtoehdolta koska silloin ei tarvitse vaivata päätään tietoisuuden vaikutuksella.

        "Tämäkin voi kuulostaa metafysiikalta, mutta ei sitä ole, koska kyse on pohjimmiltaan yksinkertaisesti vain siitä, että hiukkasia on tuossa tapauksessa oikeasti huomattavasti enemmän kuin mitä niitä kerralla voidaan havaita. "

        Se ei vain kuullosta metafysiikalta vaan nimenomaan on silkkaa metafysiikkaa (metafysiikka= aistihavaintojen ja mittausvälineiden ulottumattomissa)

        "Monimaailmatulkintaa voidaan myös testata empiirisesti tarkkailemalla lomittuneiden kvanttien käyttäytymistä, joka vastaa täysin sitä, että kullakin alkeishiukkasella olisi lukemattomia keskenään vuorovaikuttavia vastineita käytännön kannalta lähes erillisissä tila-avaruuksissa. "

        Monimaailmatulkinta ei ole tieteellinen väite koska se ei ole kumottavissa havainnoin ja mittauksin koska siinä mallissa kaikki mahdollinen tapahtuu eikä siis mikään kumoa mitään.

        "Aaltofunktion romahtamisen olettavien tulkintojen eräs ongelma on se, että ne eivät pysty esittämään objektiivista syytä sille, miksi aaltofunktio romahtaisi nimenomaan mitatulla tavalla."

        Onko monimaailmatulkinnassa sitten mitään objektiivista syytä? Käsite "objektiivinen" edellyttää että on olemassa jokin havaitsija tai tietoisuus jolle ko. asia on objektiivinen. Objektia ei voi olla ilman subjektia eli tietoisuutta eli objektiivisuus aina edellyttää subjektin eli tietoisen havaitsijan tai kokijan olemassaolon.

        "kaikki mahdolliset alkeisvaihtoehdot siis toteutuvat sen mukaan, koska hiukkanen ja sen ei-lokaalit vastineet vuorovaikuttavat keskenään"

        Kun kaikki mahdolliset vaihtoehdot toteutuvat niin mikään ei kelpaa todisteeksi tai kumoamiseksi yhtään mistään...

        "Syvällinen syy tuohon rajoittuneisuuteen on hiukan antrooppisesti se, että ainakaan järjestäytynyt havaitseminen ei olisi ylipäätään mahdollista jos suoraan havaittavissa olevien hiukkasten massa olisi suurempi kuin mikä mahdollistaisi havaitsijan muodostumisen. "

        Tuo väite sisältää sen hyvin metafyysisen oletuksen että tietoisuus on vain jonkinlainen järjestäytynyt hiukkaskokonaisuus joka edellyttää tietyn minimimäärän hiukkasia.
        Mitä tulee ns. alkuräjähdykseen niin toisaalta tietoisuuttakaan ei voi olla olemassa olemassa ilman jotain sen tietoisuuden ulkopuolella olevaa eli ei-minää. Jos alkuräjähdys on kaiken olevaisen alkuhetki niin on vaikea kuvitella tietoisuutta ilman jotain kokemisen tai havainnon objektia (maailmankaikkeus). Voisi oikeastaan sanoa että tietoisuuden ilmaantuminen edellyttää maailmankaikkeuden ilmaantumista.

        2) "Vaikka aaltofunktio romahtaisi objektiivisesti, niin ei sen romahtamisella olisi välttämättä mitään tekemistä tietoisuuden kanssa, koska sen romahtamiseen riittänee mikä tahansa hiukkasten välinen vuorovaikutus kuten esimerkiksi termodynaamisesti palautumaton lämpöliike. "

        Ei olekaan. Toisaalta fysiikkaa (eikä mitään muutakaan tiedettä) ei olisi olemassa ilman tietoisuutta. Tietoisuus on kaiken havainon ja mittaamisen ehdoton ja välttämätön edellytys.
        "Perinteinen kvanttimystiikka ei siis ole kelvollinen selitysmalli tietoisuudelle ja tietoisuuden kokemus on luontevimmin selitettävissä aineen osasten liikkeillä. "
        Kaikki mahdollinen fysiikka edellyttää tietoisuuden mutta mikään mahdollinen fysiikka ei riitä selittämään tietoisuutta. Koetapa palauttaa kaikki fysiikan matemaattiset kaavat hiukkasten liikkeisiin aika-avaruudessa. Onnistuuko?

        Tietoisuus liitetään elävien olentojen toimintaan ja elävät olennot ovat fysikaalisesti avoimia systeemeitä jotka ylläpitävät rakennettaan imemällä matalaa entropiaa ympäristöstään ja samalla poistamalla korkeampaa entropia omasta kehostaan. Pelkkä aineen liike tuskin riittää tietoisuuden selityksen pohjaksi mutta jonkinlainen lupaava suunta voisi olla tietoisuuden hahmottaminen jonkinlaisena monimutkaisena avoimien systeemeiden takaisinkytkentäprosessina (epälineaarinen kaoottinen systeemi ks esim. Prigogine).
        "hyödynnettävissä olevaa kvanttimekaanista vuorovaikutusta tapahtuu vain hyvin kylmissä lämpötiloissa,"

        Tuo on hyvin kiistanalaista (esim. fotosynteesin taustalla monet olettavat olevan kvanttivaikutuksia).

        "Mitä tulee ns. alkuräjähdykseen niin toisaalta tietoisuuttakaan ei voi olla olemassa olemassa ilman jotain sen tietoisuuden ulkopuolella olevaa eli ei-minää."

        Järjestys, jota ainakin siedettävä tietoisuus edellyttää, edellyttää epäjärjestystä jossain muualla. Jos oletetaan, että tietoisuus edellyttää hyvin suurta järjestystä, niin on hyvin luontevaa olettaa, että se edellyttää myös hyvin suurta epäjärjestystä jossain muualla. Kysymys kuuluukin, että onko tässä maailmankaikkeudessa riittävästi hiukkasia, jotta havaittavan tasoisen järjestyksen muodostumista voitaisiin pitää todennäköisenä vai täytyykö maailmankaikkeuden olla potentiaalisesti avoin jokaisessa Planck-pituuden mittaisessa pisteessään rinnakkaisiin maailmankaikkeuksiin? Tai yksinkertaisemmin ilmaisten, voiko tietoisuutta muodostua suljetussa järjestelmässä vai tarvitaanko siihen vähintään osittain avoin järjestelmä, jollainen kvanttimekaanisesti vuorovaikuttava rinnakkaisten maailmankaikkeuksien joukko olisi?

        "Toisaalta fysiikkaa (eikä mitään muutakaan tiedettä) ei olisi olemassa ilman tietoisuutta. Tietoisuus on kaiken havainon ja mittaamisen ehdoton ja välttämätön edellytys."

        Kuulostaa nurinpäin käännetyltä käsitepyramidilta. Kyllähän kokonaisuus aina edellyttää osansa, eikä päinvastoin. Eli, esimerkiksi tietoisuus edellyttää havaintoja, koska muuten noista havainnoista ei voida olla tietoisia. Abstraktien käsitteiden viittaamien olioiden reaalinen olemassaolo edellyttää sitä, että noilla olioilla on sisäinen rakenne, joka koostuu osista. Mahdolliset oletukset jakamattomista osista ovat vain tietoisuutemme ja/tai tietämyksemme rajoitteista johtuvia typistyksiä. Esimerkiksi tyypillinen fysiikan kansanomaistus, jossa aine koostuu hiukkaspalloista on harhaanjohtava ja voi aiheuttaa sen, että ainetta pidetään jotenkin pinnallisena asiana, vaikka todellisuus on toisenlainen.

        "Kaikki mahdollinen fysiikka edellyttää tietoisuuden mutta mikään mahdollinen fysiikka ei riitä selittämään tietoisuutta. Koetapa palauttaa kaikki fysiikan matemaattiset kaavat hiukkasten liikkeisiin aika-avaruudessa. Onnistuuko?"

        Kaikki matematiikka edellyttää fysikaalisia rakenteita, joihin tuo matematiikka on koodattu. Matematiikka on ihmisen kehittämä kieli, jolla kuvataan todellisia tai kuviteltuja asioita. Jos tuota kieltä ei olisi koodattu fysikaalisiin rakenteisiin, niin sillä ei olisi mitään kuvaavaa tai ohjaavaa vaikutusta mihinkään. Matematiikka on siis aina koodattu hiukkasten liikkeisiin aika-avaruudessa, eikä sitä tarvitse erikseen niihin palauttaa.

        "Pelkkä aineen liike tuskin riittää tietoisuuden selityksen pohjaksi mutta jonkinlainen lupaava suunta voisi olla tietoisuuden hahmottaminen jonkinlaisena monimutkaisena avoimien systeemeiden takaisinkytkentäprosessina (epälineaarinen kaoottinen systeemi ks esim. Prigogine)."

        Tapahtuuko tietoisuuden laadullinen kehittyminen mielestäsi asteittain vai hyppäyksittäin? Eli, voiko tietoisuuden kehittyminen liittyä jonkinlaiseen itseorganisoituvien rakenteiden faasimuutokseen? Miten tuo suhtautuu rinnakkaisten maailmankaikkeuksien vuorovaikutuksiin?

        "Tuo on hyvin kiistanalaista (esim. fotosynteesin taustalla monet olettavat olevan kvanttivaikutuksia)."

        Muistan, että myös joidenkin lintulajien silmistä väitetään löydetyn kvanttivaikutuksiin perustuvia osia. Elämä on kylläkin ihmeellistä ja olisi tietysti mm. kvanttitekniikan kehittämisen kannalta hyvin toivottavaa, että jotain tuonkaltaista löydettäisiinkin, mutta kaiken järjen mukaan realismia kuitenkin on se, että kvanttien superpositiot hajoavat äärimmäisen vähäistenkin häiriöiden seurauksena.


      • metafysiikkaa
        FirstMatter kirjoitti:

        "Niin JOS. Monimaailmatulkinta voi tuntua houkuttelevalta selitysvaihtoehdolta koska silloin ei tarvitse vaivata päätään tietoisuuden vaikutuksella."

        Monimaailmatulkinta tuntuu houkuttelevalta ja elegantilta vaihtoehdolta mm., koska se selittää havainnot havaittujen hiukkastyyppien kautta ilman oletuksia tyhjästä syntyvistä vaikutuksista. Toisaalta, filosofit ovat vaivanneet päätään tietoisuuspohdinnoilla vuosituhansia, eivätkä tulokset ole olleet kovin kummoisia, joten ainakaan ei kannata pompata siihen johtopäätökseen, että tietoisuudella olisi mitään aineesta riippumatonta vaikutusta. Tietoisuuden vaikutusten tutkiminen voi olla enemmän biologian kuin fysiikan aihealueeseen kuuluvaa.

        "Se ei vain kuullosta metafysiikalta vaan nimenomaan on silkkaa metafysiikkaa (metafysiikka= aistihavaintojen ja mittausvälineiden ulottumattomissa)"

        Tuo on yleinen harhaluulo. Monimaailmatulkinta on havaintojen tulkinta aivan kuten muutkin kvanttimekaaniset tulkinnat, eli se perustuu havaintoihin. Se on kuitenkin itseasiassa tieteellisesti houkuttelevampi tulkinta, kuin jotkin muista tulkinnoista. Aiemmin jo selitettiin miten kaksoisrakokokeen tulokset ovat selitettävissä aukottomasti fotonien sironnan kautta. On hyvin tieteellistä, eleganttia ja käsitteellisesti säästeliästä selittää esimerkiksi fotoneita koskevat havainnot nimenomaan fotonien avulla sen sijaan, että oletettaisiin mystisiä satunnaisvaikutuksia, kuten joissakin muissa kvanttimekaanisissa tulkinnoissa. Kuitenkin, kuten aiemmin jo selitettiin, jos sironta tapahtuu todellisten hiukkasten välillä, niin niiden täytyy sijaita nimenomaan erillisissä tilaulottuvuuksissa, koska muuten niiden massa tuottaisi mustan aukon tässä ulottuvuudessa ja niitä ei silloin tietenkään voitaisi havaita. Miksi oletus siitä, että olemassa voisi olla vain yksi yhtenäinen tilaulottuvuus olisi muka vähemmän metafyysinen väite kuin oletus siitä, että fotonien sijainnit noudattavat nimenomaan tietyntyyppistä tilastollista jakaumaa satunnaisten syiden vuoksi? No,eihän se olekaan, vaan perinteisesti on vaan totuttu olettamaan tuollaisia.

        "Monimaailmatulkinta ei ole tieteellinen väite koska se ei ole kumottavissa havainnoin ja mittauksin koska siinä mallissa kaikki mahdollinen tapahtuu eikä siis mikään kumoa mitään."

        Teorioiden falsifioitavuus on tieteessä yksi tärkemmistä asioista, mutta eihän esimerkiksi kaksoisrakokokeessa monimaailmatulkinnan mukaan tapahdu mitä tahansa, vaan juuri sitä mitä havaitaan. Jos fotonit eri maailmankaikkeuksissa käyttäytyisivät mallin vastaisesti, eli esimerkiksi kulkisivat toistensa läpi sen sijaan, että ne siroaisivat toisistaan, jolloin ei muodostuisi havaittavaa interferenssiä, niin sehän kumoaisi monimaailmatulkinnan.

        "Onko monimaailmatulkinnassa sitten mitään objektiivista syytä? Käsite "objektiivinen" edellyttää että on olemassa jokin havaitsija tai tietoisuus jolle ko. asia on objektiivinen. Objektia ei voi olla ilman subjektia eli tietoisuutta eli objektiivisuus aina edellyttää subjektin eli tietoisen havaitsijan tai kokijan olemassaolon."

        Monimaailmatulkinnassa kvanttimekaanisten ilmiöiden kausaalisena syynä pidetään todellisia hiukkasia, eli hiukkasia, joilla on massaa ja/tai energiaa. Monimaailmatulkinnan kannattajat tulkitsevat havainnot siten, että niiden aiheuttajat ovat todellisia hiukkasia, toisin kuin joissakin muissa tulkinnoissa, joissa ilmeisesti oletetaan, että pelkät matemaattiset oliot voivat aiheuttaa havaittavia fysikaalisia vaikutuksia, jonkin tuntemattoman mekanismin kautta.

        "Kun kaikki mahdolliset vaihtoehdot toteutuvat niin mikään ei kelpaa todisteeksi tai kumoamiseksi yhtään mistään..."

        Kaikki mahdolliset vaihtoehdot toteutuvat kvanttilogiikan mukaan väistämättä. Tämähän on todistettu kvanttitietokoneiden avulla. Mahdottomat vaihtoehdot sen sijaan eivät toteudu, eli esimerkiksi yksikään fotoni ei päädy kaksoisrakokokeessa mustien raitojen kohdalle. Jos ne sinne päätyisivät, niin monimaailmatulkinta kumoutuisi tai sitä pitäisi muokata.

        "Tuo väite sisältää sen hyvin metafyysisen oletuksen että tietoisuus on vain jonkinlainen järjestäytynyt hiukkaskokonaisuus joka edellyttää tietyn minimimäärän hiukkasia."

        Tietoisuusteorioiden todentaminen edellyttää mittauksia ja objektiivisesti voidaan kätevimmin mitata hiukkasten ja niistä koostuvien rakenteiden ominaisuuksia. Vaikka tietoisuus ei olisikaan vain järjestäytynyt hiukkaskokonaisuus, niin meillä ei kuitenkaan ole havaintoja ainakaan tietoisuudesta, joka ei edellyttäisi järjestäytynyttä hiukkaskokonaisuutta, joten suoraviivaisimmalta tavalta tietoisuuden selittämiseen vaikuttaisi tuollaisten hiukkaskokonaisuuksien ominaisuuksien mittaukset. Tietysti tähän liittyy käytännöllinen mittausongelma, eli mitä lukemattomista muuttujista nimenomaan pitäisi tai kannattaisi mitata.

        "Monimaailmatulkinta tuntuu houkuttelevalta ja elegantilta vaihtoehdolta mm., koska se selittää havainnot havaittujen hiukkastyyppien kautta ilman oletuksia tyhjästä syntyvistä vaikutuksista."

        Jostain syystä fysiikassa se tyhjästä nyhjääminen on joka tapauksessa käytössä Big bangin tapauksessa. Alkeishiukkasia ei ole kukaan koskaan suoraan havainnut joten on aika kyseenalaista puhua "havaituista hiukkastyypeistä" kun koko fysiikan standardimalli on matemaattinen konstruktio eli fysiikankin pohjalla on metafyysinen ja abstrakti mentaalinen konstruktio ja muutenkaan mitään puhtaasti empiiristä havaintoa ei ole olemassakaan vaan kaikki ns. aistihavainnot ovat aina vahvasti prosessoituja ja niiden tulkinta aina riippuvaista valitusta käsitejärjestelmästä ja kielestä.

        "Toisaalta, filosofit ovat vaivanneet päätään tietoisuuspohdinnoilla vuosituhansia, eivätkä tulokset ole olleet kovin kummoisia, joten ainakaan ei kannata pompata siihen johtopäätökseen, että tietoisuudella olisi mitään aineesta riippumatonta vaikutusta. "

        Fysiikan tulokset ovat tuottaneet toimivaa tekniikkaa nimenomaan juuri sen takia että tutkimuskohde on niin säännönmukainen ja mekaaninen eli suhteellisen helposti matemaattisesti mallinnettavissa verrattuna esim. eliöiden ja ihmisten toimintaan.

        Arkikielessä tietoisuuden tuottamia fysikaalisia kokonaisuuksia kutsutaan "keinotekoisiksi" kun taas ns. "luonnonlakien" tuottamia vastaavia kokonaisuuksia "luonnollisiksi" vaikka periaatteessa kai molemmat pitäisi tulkita luonnollisiksi. Jossain mielessä siis vaistomaisesti koetaan tietoisuuden vaikutus ilmiöihin jollain tavalla luonnolle vieraana asiana.

        Monimaailmatulkinnan varmaan yksi fyysikoita houkutteleva ominaisuus suhteessa aaltofunktion romahdusmalleihin on että mm-tulkinta on deterministinen kun taas romahdustulkinta ei ole ja mm-tulkinnassa ajalla ei ole suuntaa kun taas romahdus on palautumaton ajan suhteen.

        "Tietoisuuden vaikutusten tutkiminen voi olla enemmän biologian kuin fysiikan aihealueeseen kuuluvaa."

        Niin tietysti mutta ilmiöiden kokonaisuus ei ole sillä tavalla selkeästi lokeroitavissa eri tieteenalojen reviireihin.

        "Monimaailmatulkinta on havaintojen tulkinta aivan kuten muutkin kvanttimekaaniset tulkinnat, eli se perustuu havaintoihin. "

        Kvanttifýsiikan tulkintoja on olemassa toistakymmentä ja ne ovat kaikki sisäisti koherentteja malleja suhteessa havaintoihin eikä niiden välillä ainakaan toistaiseksi voi tehdä valintaa muuten kuin mieltymyksien perusteella eli kyseessä on samalla tavalla makuasia kuin suklaajäätelä vs mansikkajäätelö.

        "Aiemmin jo selitettiin miten kaksoisrakokokeen tulokset ovat selitettävissä aukottomasti fotonien sironnan kautta. "

        Pointti onkin se että erilaisia "aukottomia" taustatarinoita voi kehitellä mutta valinta niiden välillä on edelleen vahvasti makuasia.

        "On hyvin tieteellistä, eleganttia ja käsitteellisesti säästeliästä selittää esimerkiksi fotoneita koskevat havainnot nimenomaan fotonien avulla sen sijaan, että oletettaisiin mystisiä satunnaisvaikutuksia, kuten joissakin muissa kvanttimekaanisissa tulkinnoissa."

        "Tieteellisyys" (?), eleganssi ja käsitteellinen säästeliäisyys ovat luonteeltaan laadullisia makuasioita eikä mitään mallin vahvistuksia sinänsä. Jonkun toisen tulkinnan kannattaja voisi sanoa samat asiat.

        "että oletettaisiin mystisiä satunnaisvaikutuksia, kuten joissakin muissa kvanttimekaanisissa tulkinnoissa"

        Satunnaisvaikutukset ovat todellakin "mystisiä" siinä mielessä että niitä ei voi mekaanisella mallilla kuvata mutta toisaalta satunnaisuuksiakin on erilaisia ja se satunnaisuus voi tarkoittaa että ilmöitä ei riittävästi ymmärretä vielä.

        "jos sironta tapahtuu todellisten hiukkasten välillä, niin niiden täytyy sijaita nimenomaan erillisissä tilaulottuvuuksissa, koska muuten niiden massa tuottaisi mustan aukon tässä ulottuvuudessa ja niitä ei silloin tietenkään voitaisi havaita."

        Mikä on "todellinen hiukkanen"? Asian voi myös hahmottaa niin päin että näennäisesti erilliset hiukkaset ovat saman kokonaisuuden osia jotka vaikuttavat toisiinsa ajasta ja paikasta riippumatta ikäänkuin hyperdimensionaalisesti (kuten lomittumisilmiössä).

        Suhtista ja kvanttiteoriaa ei ole kyetty yhdistämään joten paras jättää ne mustat aukot pois tästä kuvioista kokonaan kun tuo skenaario vaikuttaa melko spekulatiiviselta muutenkin.

        "Miksi oletus siitä, että olemassa voisi olla vain yksi yhtenäinen tilaulottuvuus olisi muka vähemmän metafyysinen väite kuin oletus siitä, että fotonien sijainnit noudattavat nimenomaan tietyntyyppistä tilastollista jakaumaa satunnaisten syiden vuoksi?"
        Ei se olekaan vähemmän metafyysinen kun periaatteessa koko fysiikan käsitejärjestelmä on metafysiikkaa siinä mielessä että se on vain tulkinta havainnoista ja metafysiikasta vapaata "puhdasta" havainnointia ei ole olemassakaan. (jatkuu...)


      • metafysiikkaa
        metafysiikkaa kirjoitti:

        "Monimaailmatulkinta tuntuu houkuttelevalta ja elegantilta vaihtoehdolta mm., koska se selittää havainnot havaittujen hiukkastyyppien kautta ilman oletuksia tyhjästä syntyvistä vaikutuksista."

        Jostain syystä fysiikassa se tyhjästä nyhjääminen on joka tapauksessa käytössä Big bangin tapauksessa. Alkeishiukkasia ei ole kukaan koskaan suoraan havainnut joten on aika kyseenalaista puhua "havaituista hiukkastyypeistä" kun koko fysiikan standardimalli on matemaattinen konstruktio eli fysiikankin pohjalla on metafyysinen ja abstrakti mentaalinen konstruktio ja muutenkaan mitään puhtaasti empiiristä havaintoa ei ole olemassakaan vaan kaikki ns. aistihavainnot ovat aina vahvasti prosessoituja ja niiden tulkinta aina riippuvaista valitusta käsitejärjestelmästä ja kielestä.

        "Toisaalta, filosofit ovat vaivanneet päätään tietoisuuspohdinnoilla vuosituhansia, eivätkä tulokset ole olleet kovin kummoisia, joten ainakaan ei kannata pompata siihen johtopäätökseen, että tietoisuudella olisi mitään aineesta riippumatonta vaikutusta. "

        Fysiikan tulokset ovat tuottaneet toimivaa tekniikkaa nimenomaan juuri sen takia että tutkimuskohde on niin säännönmukainen ja mekaaninen eli suhteellisen helposti matemaattisesti mallinnettavissa verrattuna esim. eliöiden ja ihmisten toimintaan.

        Arkikielessä tietoisuuden tuottamia fysikaalisia kokonaisuuksia kutsutaan "keinotekoisiksi" kun taas ns. "luonnonlakien" tuottamia vastaavia kokonaisuuksia "luonnollisiksi" vaikka periaatteessa kai molemmat pitäisi tulkita luonnollisiksi. Jossain mielessä siis vaistomaisesti koetaan tietoisuuden vaikutus ilmiöihin jollain tavalla luonnolle vieraana asiana.

        Monimaailmatulkinnan varmaan yksi fyysikoita houkutteleva ominaisuus suhteessa aaltofunktion romahdusmalleihin on että mm-tulkinta on deterministinen kun taas romahdustulkinta ei ole ja mm-tulkinnassa ajalla ei ole suuntaa kun taas romahdus on palautumaton ajan suhteen.

        "Tietoisuuden vaikutusten tutkiminen voi olla enemmän biologian kuin fysiikan aihealueeseen kuuluvaa."

        Niin tietysti mutta ilmiöiden kokonaisuus ei ole sillä tavalla selkeästi lokeroitavissa eri tieteenalojen reviireihin.

        "Monimaailmatulkinta on havaintojen tulkinta aivan kuten muutkin kvanttimekaaniset tulkinnat, eli se perustuu havaintoihin. "

        Kvanttifýsiikan tulkintoja on olemassa toistakymmentä ja ne ovat kaikki sisäisti koherentteja malleja suhteessa havaintoihin eikä niiden välillä ainakaan toistaiseksi voi tehdä valintaa muuten kuin mieltymyksien perusteella eli kyseessä on samalla tavalla makuasia kuin suklaajäätelä vs mansikkajäätelö.

        "Aiemmin jo selitettiin miten kaksoisrakokokeen tulokset ovat selitettävissä aukottomasti fotonien sironnan kautta. "

        Pointti onkin se että erilaisia "aukottomia" taustatarinoita voi kehitellä mutta valinta niiden välillä on edelleen vahvasti makuasia.

        "On hyvin tieteellistä, eleganttia ja käsitteellisesti säästeliästä selittää esimerkiksi fotoneita koskevat havainnot nimenomaan fotonien avulla sen sijaan, että oletettaisiin mystisiä satunnaisvaikutuksia, kuten joissakin muissa kvanttimekaanisissa tulkinnoissa."

        "Tieteellisyys" (?), eleganssi ja käsitteellinen säästeliäisyys ovat luonteeltaan laadullisia makuasioita eikä mitään mallin vahvistuksia sinänsä. Jonkun toisen tulkinnan kannattaja voisi sanoa samat asiat.

        "että oletettaisiin mystisiä satunnaisvaikutuksia, kuten joissakin muissa kvanttimekaanisissa tulkinnoissa"

        Satunnaisvaikutukset ovat todellakin "mystisiä" siinä mielessä että niitä ei voi mekaanisella mallilla kuvata mutta toisaalta satunnaisuuksiakin on erilaisia ja se satunnaisuus voi tarkoittaa että ilmöitä ei riittävästi ymmärretä vielä.

        "jos sironta tapahtuu todellisten hiukkasten välillä, niin niiden täytyy sijaita nimenomaan erillisissä tilaulottuvuuksissa, koska muuten niiden massa tuottaisi mustan aukon tässä ulottuvuudessa ja niitä ei silloin tietenkään voitaisi havaita."

        Mikä on "todellinen hiukkanen"? Asian voi myös hahmottaa niin päin että näennäisesti erilliset hiukkaset ovat saman kokonaisuuden osia jotka vaikuttavat toisiinsa ajasta ja paikasta riippumatta ikäänkuin hyperdimensionaalisesti (kuten lomittumisilmiössä).

        Suhtista ja kvanttiteoriaa ei ole kyetty yhdistämään joten paras jättää ne mustat aukot pois tästä kuvioista kokonaan kun tuo skenaario vaikuttaa melko spekulatiiviselta muutenkin.

        "Miksi oletus siitä, että olemassa voisi olla vain yksi yhtenäinen tilaulottuvuus olisi muka vähemmän metafyysinen väite kuin oletus siitä, että fotonien sijainnit noudattavat nimenomaan tietyntyyppistä tilastollista jakaumaa satunnaisten syiden vuoksi?"
        Ei se olekaan vähemmän metafyysinen kun periaatteessa koko fysiikan käsitejärjestelmä on metafysiikkaa siinä mielessä että se on vain tulkinta havainnoista ja metafysiikasta vapaata "puhdasta" havainnointia ei ole olemassakaan. (jatkuu...)

        "Monimaailmatulkinnassa kvanttimekaanisten ilmiöiden kausaalisena syynä pidetään todellisia hiukkasia, eli hiukkasia, joilla on massaa ja/tai energiaa. "

        Hiukkaset ovat matemaattisia konstruktioita ja kausaliteetti tai determinismi pätee vain suurten hiukkasjoukkojen suhteen tilastollisesti. Tässä yritetään vääntää todellisuutta epätoivoisesti tutun ja turvallisen newtonilaisen mallin suuntaan.

        "pelkät matemaattiset oliot voivat aiheuttaa havaittavia fysikaalisia vaikutuksia, jonkin tuntemattoman mekanismin kautta."

        Maastokartoissa käytetään symboleita kuvaamaan erilaisia maaston ominaisuuksia eikä ne symbolit kirjaimellisesti ole siellä maastossa mutta viittaavat johonkin maaston havaittavissa olevaan ominaisuuteen. Vahvasti matematisoiduissa tieteissä helposti sotketaan se kartta itse maastoon ja aletaan pitää joidenkin asioiden symboleja identtisinä itse ilmiöiden kanssa.

        Alkeishiukkanen on symboli jossa on osittain konkreettiseen havaintoon tai mittaukseen liittyvää mutta myös siihen abstraktiin teoriaan tai malliin liittyviä asioita eli se on ikäänkuin hyperlinkki. On yhtä suuri virhe mieltää se jotenkin konkreettiseksi fysikaaliseksi asiaksi tai vastaavasti puhtaasti teoreettiseksi matemattiseksi asiaksi.

        "Kaikki mahdolliset vaihtoehdot toteutuvat kvanttilogiikan mukaan väistämättä. Tämähän on todistettu kvanttitietokoneiden avulla."

        Miten? Jos kaikki vaihtoehdot toteutuvat niin miten kvanttitietokone voi tuottaa laskutoimitukseen oikean tuloksen?

        "Tietoisuusteorioiden todentaminen edellyttää mittauksia ja objektiivisesti voidaan kätevimmin mitata hiukkasten ja niistä koostuvien rakenteiden ominaisuuksia."

        Tietoisuus on luonteeltaan aina subjektiivista eli omaa kokemuksellisuutta. Toisten olioiden tietoisuus on aina käyttäytymisestä pääteltyä eli virhealtista jos vuorovaikutus tapahtuu esim. netin kautta ilman katsekontaktia eli ts. ns. tietoinen käyttäytyminen on feikattavissa. Millä konstilla päättelet hiukkasten käyttäytymisestä tai systeemin rakenteesta virheettömästi tietoisuuden olemassaolon?

        "Järjestys, jota ainakin siedettävä tietoisuus edellyttää, edellyttää epäjärjestystä jossain muualla. Jos oletetaan, että tietoisuus edellyttää hyvin suurta järjestystä, niin on hyvin luontevaa olettaa, että se edellyttää myös hyvin suurta epäjärjestystä jossain muualla. "

        Auringon järjestäytynyt energia mahdollistaa elämän maapallolla ja vaikka auringon epäjärjestys tässä jonkin verran lisääntyy niin se lisääntyisi joka tapauksessa samalla tavalla vaikka maapallolla ei olisikaan elämää. Kun ympäristössä on korkealaatuista energiaa mistä ottaa niin se epäjärjestyksen lisääntyminen ei tunnu mitenkään kovin dramaattiselta.

        Itse luulen että tietoisuus on pohjimmiltaan enemmän non-lokaali hyperdimensioonaalinen ilmiö ja se subjektiivinen kokemuksellisuus taas sen nonlokaalin asian monimutkainen vuorovaikutussuhde lokaalin fysikaalisen maailman kanssa.

        "Kysymys kuuluukin, että onko tässä maailmankaikkeudessa riittävästi hiukkasia, jotta havaittavan tasoisen järjestyksen muodostumista voitaisiin pitää todennäköisenä vai täytyykö maailmankaikkeuden olla potentiaalisesti avoin jokaisessa Planck-pituuden mittaisessa pisteessään rinnakkaisiin maailmankaikkeuksiin? "

        Nythän ei tiedetä kuinka todennäköistä ja yleistä biologinen elämä on maailmankaikkeudessa ja jos oletetan tietoisuuden edellyttävän biologista elämää niin sitäkään ei tiedetä mikä on tietoisuuden ilmenemisen minimiedellytys eli onko bakteeri tietoinen. Tässä on hyvä taas muistaa että ainoa tietoisuus jonka olemassaolosta voimme olla 100% varmoja on oma tietoisuutemme.

        "Tai yksinkertaisemmin ilmaisten, voiko tietoisuutta muodostua suljetussa järjestelmässä vai tarvitaanko siihen vähintään osittain avoin järjestelmä, jollainen kvanttimekaanisesti vuorovaikuttava rinnakkaisten maailmankaikkeuksien joukko olisi?"

        Jos nyt oletetaan että perusoletuksesi tietoisuuden pohjautumisesta hiukkasiin pitäisi paikkansa niin ei edes tiedetä kuinka paljon energiaa ja hiukkasia tarvitaan minimaalisen tietoisuuden muodostamiseksi joten tässäkääm ei päästä arvailuja pidemmälle.

        "Eli, esimerkiksi tietoisuus edellyttää havaintoja, koska muuten noista havainnoista ei voida olla tietoisia. "

        Kyllä tietoisuus kykenee tuottamaan sisältöjä ilman havaintojakin varsin hyvin esim. unien näkemisessä ja hallusinaatioissa. Tiede on inhimillistä sosiaalista toimintaa eikä mitään jumalallista itseriittoista kuviokelluntaa ilman edellytyksiä.

        " Abstraktien käsitteiden viittaamien olioiden reaalinen olemassaolo edellyttää sitä, että noilla olioilla on sisäinen rakenne, joka koostuu osista."

        Tuo on se reduktionismin uskontunnustus. Yhtä hyvin voisi ajetella että kokonaisuus erilaistuu osiksi kuten esim. solujen jakautumisessa biologisissa ilmiöissä jolloin sen alkusolun geneettinen informaatio siirtyy niihin jakautuneisiin soluihin. Tietoisuuden voisi myös ajatella olevan yksi kokonaisuus joka sitten jakautuu osiksi.


      • metafysiikkaa
        metafysiikkaa kirjoitti:

        "Monimaailmatulkinnassa kvanttimekaanisten ilmiöiden kausaalisena syynä pidetään todellisia hiukkasia, eli hiukkasia, joilla on massaa ja/tai energiaa. "

        Hiukkaset ovat matemaattisia konstruktioita ja kausaliteetti tai determinismi pätee vain suurten hiukkasjoukkojen suhteen tilastollisesti. Tässä yritetään vääntää todellisuutta epätoivoisesti tutun ja turvallisen newtonilaisen mallin suuntaan.

        "pelkät matemaattiset oliot voivat aiheuttaa havaittavia fysikaalisia vaikutuksia, jonkin tuntemattoman mekanismin kautta."

        Maastokartoissa käytetään symboleita kuvaamaan erilaisia maaston ominaisuuksia eikä ne symbolit kirjaimellisesti ole siellä maastossa mutta viittaavat johonkin maaston havaittavissa olevaan ominaisuuteen. Vahvasti matematisoiduissa tieteissä helposti sotketaan se kartta itse maastoon ja aletaan pitää joidenkin asioiden symboleja identtisinä itse ilmiöiden kanssa.

        Alkeishiukkanen on symboli jossa on osittain konkreettiseen havaintoon tai mittaukseen liittyvää mutta myös siihen abstraktiin teoriaan tai malliin liittyviä asioita eli se on ikäänkuin hyperlinkki. On yhtä suuri virhe mieltää se jotenkin konkreettiseksi fysikaaliseksi asiaksi tai vastaavasti puhtaasti teoreettiseksi matemattiseksi asiaksi.

        "Kaikki mahdolliset vaihtoehdot toteutuvat kvanttilogiikan mukaan väistämättä. Tämähän on todistettu kvanttitietokoneiden avulla."

        Miten? Jos kaikki vaihtoehdot toteutuvat niin miten kvanttitietokone voi tuottaa laskutoimitukseen oikean tuloksen?

        "Tietoisuusteorioiden todentaminen edellyttää mittauksia ja objektiivisesti voidaan kätevimmin mitata hiukkasten ja niistä koostuvien rakenteiden ominaisuuksia."

        Tietoisuus on luonteeltaan aina subjektiivista eli omaa kokemuksellisuutta. Toisten olioiden tietoisuus on aina käyttäytymisestä pääteltyä eli virhealtista jos vuorovaikutus tapahtuu esim. netin kautta ilman katsekontaktia eli ts. ns. tietoinen käyttäytyminen on feikattavissa. Millä konstilla päättelet hiukkasten käyttäytymisestä tai systeemin rakenteesta virheettömästi tietoisuuden olemassaolon?

        "Järjestys, jota ainakin siedettävä tietoisuus edellyttää, edellyttää epäjärjestystä jossain muualla. Jos oletetaan, että tietoisuus edellyttää hyvin suurta järjestystä, niin on hyvin luontevaa olettaa, että se edellyttää myös hyvin suurta epäjärjestystä jossain muualla. "

        Auringon järjestäytynyt energia mahdollistaa elämän maapallolla ja vaikka auringon epäjärjestys tässä jonkin verran lisääntyy niin se lisääntyisi joka tapauksessa samalla tavalla vaikka maapallolla ei olisikaan elämää. Kun ympäristössä on korkealaatuista energiaa mistä ottaa niin se epäjärjestyksen lisääntyminen ei tunnu mitenkään kovin dramaattiselta.

        Itse luulen että tietoisuus on pohjimmiltaan enemmän non-lokaali hyperdimensioonaalinen ilmiö ja se subjektiivinen kokemuksellisuus taas sen nonlokaalin asian monimutkainen vuorovaikutussuhde lokaalin fysikaalisen maailman kanssa.

        "Kysymys kuuluukin, että onko tässä maailmankaikkeudessa riittävästi hiukkasia, jotta havaittavan tasoisen järjestyksen muodostumista voitaisiin pitää todennäköisenä vai täytyykö maailmankaikkeuden olla potentiaalisesti avoin jokaisessa Planck-pituuden mittaisessa pisteessään rinnakkaisiin maailmankaikkeuksiin? "

        Nythän ei tiedetä kuinka todennäköistä ja yleistä biologinen elämä on maailmankaikkeudessa ja jos oletetan tietoisuuden edellyttävän biologista elämää niin sitäkään ei tiedetä mikä on tietoisuuden ilmenemisen minimiedellytys eli onko bakteeri tietoinen. Tässä on hyvä taas muistaa että ainoa tietoisuus jonka olemassaolosta voimme olla 100% varmoja on oma tietoisuutemme.

        "Tai yksinkertaisemmin ilmaisten, voiko tietoisuutta muodostua suljetussa järjestelmässä vai tarvitaanko siihen vähintään osittain avoin järjestelmä, jollainen kvanttimekaanisesti vuorovaikuttava rinnakkaisten maailmankaikkeuksien joukko olisi?"

        Jos nyt oletetaan että perusoletuksesi tietoisuuden pohjautumisesta hiukkasiin pitäisi paikkansa niin ei edes tiedetä kuinka paljon energiaa ja hiukkasia tarvitaan minimaalisen tietoisuuden muodostamiseksi joten tässäkääm ei päästä arvailuja pidemmälle.

        "Eli, esimerkiksi tietoisuus edellyttää havaintoja, koska muuten noista havainnoista ei voida olla tietoisia. "

        Kyllä tietoisuus kykenee tuottamaan sisältöjä ilman havaintojakin varsin hyvin esim. unien näkemisessä ja hallusinaatioissa. Tiede on inhimillistä sosiaalista toimintaa eikä mitään jumalallista itseriittoista kuviokelluntaa ilman edellytyksiä.

        " Abstraktien käsitteiden viittaamien olioiden reaalinen olemassaolo edellyttää sitä, että noilla olioilla on sisäinen rakenne, joka koostuu osista."

        Tuo on se reduktionismin uskontunnustus. Yhtä hyvin voisi ajetella että kokonaisuus erilaistuu osiksi kuten esim. solujen jakautumisessa biologisissa ilmiöissä jolloin sen alkusolun geneettinen informaatio siirtyy niihin jakautuneisiin soluihin. Tietoisuuden voisi myös ajatella olevan yksi kokonaisuus joka sitten jakautuu osiksi.

        "Kaikki matematiikka edellyttää fysikaalisia rakenteita, joihin tuo matematiikka on koodattu."

        Tuskin. Kyllä matemaattinen mielikuvitus on paljon laajempi kuin fysikaaliset rakenteet.
        (esim. Mandelbrotin joukko). Mikä tahansa looginen konstruktio toimii jos se on sisäisesti koherentti. Ei luonnossa ole autoja, lentokoneita eikä tietokoneitakaan vaan ne ovat inhimillisen luomiskyvyn tuotteita.

        "Jos tuota kieltä ei olisi koodattu fysikaalisiin rakenteisiin, niin sillä ei olisi mitään kuvaavaa tai ohjaavaa vaikutusta mihinkään."

        Niin jos kiellät tietoisuuden olemassaolon ja vaikutuksen niin tuollainen johtopäätös syntyy helposti.

        "Tapahtuuko tietoisuuden laadullinen kehittyminen mielestäsi asteittain vai hyppäyksittäin? "

        Sekä että paitsi että se mitä kutsumme tietoisuudeksi on lähinnä monimutkainen takaisinkytkentäjärjestelmä joka ei sinänsä edellytä jatkuvaa tietoisuutta. Inhimillinen toiminta on hyvin suurelta osin automaattista.

        "Eli, voiko tietoisuuden kehittyminen liittyä jonkinlaiseen itseorganisoituvien rakenteiden faasimuutokseen? "

        Tuo voi liittyä ainakin elämän kehittymiseen. Tuossa voisi olla hyödyllistä ajatella jotenkin kaksoisaspektimallin kautta siten että käyttäytyminen (aine, hiukkasten liike) on olevaisen objektiivinen aspekti ja tietoisuus saman asian subjektiivinen aspekti ainakin jos on kyse elävistä tietoisista olennoista. Tietoisuus sinänsä jää siten aina tieteellisen metodin ulottumattomiin ja ainoastaan voimme havaita tietoisuuden vaikutukset vain epäsuorasti kun kyse ei ole omasta tietoisuudestamme.

        "Miten tuo suhtautuu rinnakkaisten maailmankaikkeuksien vuorovaikutuksiin?"

        En tiedä kun en edes usko mm-malliin jossa on sotkettu potentiaalisuus aktuaaliseen. Tulevaisuus on aina potentiaalista ja vastaavasti menneisyys on aina aktuaalista ja nykyhetkessä (joka voi olla eri ulotteinen eri oloilla) tapahtuu valinta tai valikoituu mekaanisesti yksi vaihtoehto aktuaaliseksi.

        " mutta kaiken järjen mukaan realismia kuitenkin on se, että kvanttien superpositiot hajoavat äärimmäisen vähäistenkin häiriöiden seurauksena."

        On eri asia pystyä mittaamaan ne superpositiot kuin että ne ovat olemassa. Determinismi on tuttua ja turvallista ja mm-tulkinta tarjoaa mahdollisuuden takertua siihen vanhaan malliin.

        ...

        Vahva kiihtyvyyskään ei viimeisimpien tutkimusten mukaan sotke lomittumisilmiötä:

        http://www.sciencealert.com/scientists-have-set-a-limit-for-quantum-entanglement-and-it-s-really-freaking-powerful

        kvanttikommunikaatio ilman hiukkasten siirtoa:

        http://www.pnas.org/content/114/19/4920.abstract


      • FirstMatter
        metafysiikkaa kirjoitti:

        "Monimaailmatulkinnassa kvanttimekaanisten ilmiöiden kausaalisena syynä pidetään todellisia hiukkasia, eli hiukkasia, joilla on massaa ja/tai energiaa. "

        Hiukkaset ovat matemaattisia konstruktioita ja kausaliteetti tai determinismi pätee vain suurten hiukkasjoukkojen suhteen tilastollisesti. Tässä yritetään vääntää todellisuutta epätoivoisesti tutun ja turvallisen newtonilaisen mallin suuntaan.

        "pelkät matemaattiset oliot voivat aiheuttaa havaittavia fysikaalisia vaikutuksia, jonkin tuntemattoman mekanismin kautta."

        Maastokartoissa käytetään symboleita kuvaamaan erilaisia maaston ominaisuuksia eikä ne symbolit kirjaimellisesti ole siellä maastossa mutta viittaavat johonkin maaston havaittavissa olevaan ominaisuuteen. Vahvasti matematisoiduissa tieteissä helposti sotketaan se kartta itse maastoon ja aletaan pitää joidenkin asioiden symboleja identtisinä itse ilmiöiden kanssa.

        Alkeishiukkanen on symboli jossa on osittain konkreettiseen havaintoon tai mittaukseen liittyvää mutta myös siihen abstraktiin teoriaan tai malliin liittyviä asioita eli se on ikäänkuin hyperlinkki. On yhtä suuri virhe mieltää se jotenkin konkreettiseksi fysikaaliseksi asiaksi tai vastaavasti puhtaasti teoreettiseksi matemattiseksi asiaksi.

        "Kaikki mahdolliset vaihtoehdot toteutuvat kvanttilogiikan mukaan väistämättä. Tämähän on todistettu kvanttitietokoneiden avulla."

        Miten? Jos kaikki vaihtoehdot toteutuvat niin miten kvanttitietokone voi tuottaa laskutoimitukseen oikean tuloksen?

        "Tietoisuusteorioiden todentaminen edellyttää mittauksia ja objektiivisesti voidaan kätevimmin mitata hiukkasten ja niistä koostuvien rakenteiden ominaisuuksia."

        Tietoisuus on luonteeltaan aina subjektiivista eli omaa kokemuksellisuutta. Toisten olioiden tietoisuus on aina käyttäytymisestä pääteltyä eli virhealtista jos vuorovaikutus tapahtuu esim. netin kautta ilman katsekontaktia eli ts. ns. tietoinen käyttäytyminen on feikattavissa. Millä konstilla päättelet hiukkasten käyttäytymisestä tai systeemin rakenteesta virheettömästi tietoisuuden olemassaolon?

        "Järjestys, jota ainakin siedettävä tietoisuus edellyttää, edellyttää epäjärjestystä jossain muualla. Jos oletetaan, että tietoisuus edellyttää hyvin suurta järjestystä, niin on hyvin luontevaa olettaa, että se edellyttää myös hyvin suurta epäjärjestystä jossain muualla. "

        Auringon järjestäytynyt energia mahdollistaa elämän maapallolla ja vaikka auringon epäjärjestys tässä jonkin verran lisääntyy niin se lisääntyisi joka tapauksessa samalla tavalla vaikka maapallolla ei olisikaan elämää. Kun ympäristössä on korkealaatuista energiaa mistä ottaa niin se epäjärjestyksen lisääntyminen ei tunnu mitenkään kovin dramaattiselta.

        Itse luulen että tietoisuus on pohjimmiltaan enemmän non-lokaali hyperdimensioonaalinen ilmiö ja se subjektiivinen kokemuksellisuus taas sen nonlokaalin asian monimutkainen vuorovaikutussuhde lokaalin fysikaalisen maailman kanssa.

        "Kysymys kuuluukin, että onko tässä maailmankaikkeudessa riittävästi hiukkasia, jotta havaittavan tasoisen järjestyksen muodostumista voitaisiin pitää todennäköisenä vai täytyykö maailmankaikkeuden olla potentiaalisesti avoin jokaisessa Planck-pituuden mittaisessa pisteessään rinnakkaisiin maailmankaikkeuksiin? "

        Nythän ei tiedetä kuinka todennäköistä ja yleistä biologinen elämä on maailmankaikkeudessa ja jos oletetan tietoisuuden edellyttävän biologista elämää niin sitäkään ei tiedetä mikä on tietoisuuden ilmenemisen minimiedellytys eli onko bakteeri tietoinen. Tässä on hyvä taas muistaa että ainoa tietoisuus jonka olemassaolosta voimme olla 100% varmoja on oma tietoisuutemme.

        "Tai yksinkertaisemmin ilmaisten, voiko tietoisuutta muodostua suljetussa järjestelmässä vai tarvitaanko siihen vähintään osittain avoin järjestelmä, jollainen kvanttimekaanisesti vuorovaikuttava rinnakkaisten maailmankaikkeuksien joukko olisi?"

        Jos nyt oletetaan että perusoletuksesi tietoisuuden pohjautumisesta hiukkasiin pitäisi paikkansa niin ei edes tiedetä kuinka paljon energiaa ja hiukkasia tarvitaan minimaalisen tietoisuuden muodostamiseksi joten tässäkääm ei päästä arvailuja pidemmälle.

        "Eli, esimerkiksi tietoisuus edellyttää havaintoja, koska muuten noista havainnoista ei voida olla tietoisia. "

        Kyllä tietoisuus kykenee tuottamaan sisältöjä ilman havaintojakin varsin hyvin esim. unien näkemisessä ja hallusinaatioissa. Tiede on inhimillistä sosiaalista toimintaa eikä mitään jumalallista itseriittoista kuviokelluntaa ilman edellytyksiä.

        " Abstraktien käsitteiden viittaamien olioiden reaalinen olemassaolo edellyttää sitä, että noilla olioilla on sisäinen rakenne, joka koostuu osista."

        Tuo on se reduktionismin uskontunnustus. Yhtä hyvin voisi ajetella että kokonaisuus erilaistuu osiksi kuten esim. solujen jakautumisessa biologisissa ilmiöissä jolloin sen alkusolun geneettinen informaatio siirtyy niihin jakautuneisiin soluihin. Tietoisuuden voisi myös ajatella olevan yksi kokonaisuus joka sitten jakautuu osiksi.

        ""Kaikki mahdolliset vaihtoehdot toteutuvat kvanttilogiikan mukaan väistämättä. Tämähän on todistettu kvanttitietokoneiden avulla.""
        "Miten? Jos kaikki vaihtoehdot toteutuvat niin miten kvanttitietokone voi tuottaa laskutoimitukseen oikean tuloksen?"

        Kvanttitietokoneiden toiminta on suhteellisen mutkikasta. Niiden kvanttilaskentaa hyödyntävät kvanttipiirit suunnitellaan erityisesti siten, että tietyn ongelmatyypin ratkaisussa on mahdollista hyödyntää kvanttilaskentaa. Ensimmäisessä vaiheessa tietyn ongelman ratkaisuun liittyvät kaikki mahdolliset lähtötiedot tallennetaan ensimmäiseen kvanttimuistiin; eli kubittien joukkoon, noiden lähtötietojen superpositiona. Tämä saadaan aikaan esimerkiksi käyttämällä joukkoa toisiinsa kytkettyjä Hadamard-portteja, joista kukin suorittaa yleistetyn loogisen operaation, joka vastaa NOT:n neliöjuurta.

        Toisessa vaiheessa suoritetaan varsinainen kvanttilaskenta. Eli, esimerkiksi tekijöihinjaon mahdollistavan Shorin kvanttialgoritmin logiikan toteuttavassa piiristössä laskutoimituksena on: (x^a) mod n, jossa x on satunnaisesti valittu kokonaisluku ja a on ensimmäisen kvanttimuistin sisältö. Kvanttitietokone suorittaa piiriensä ohjaamana kaikki mahdolliset tarvittavat erilaiset laskutoimitukset periaatteessa viiveettömästi. Aikaa tietysti kuitenkin käytännössä kuluu siihen kun signaalit joutuvat kulkemaan piireissä klassiseen tapaan. Käytännössä tuloksen esiinkaivaminen vie myös aikaa, koska dekoherenssista johtuen joudutaan turvautumaan kvanttivirheenkorjaukseen, eli suorittamaan samoja laskutoimituksia niin monta kertaa, että tulos on tilastollisesti riittävän suurella todennäköisyydellä oikea. Dekoherenssi on suurin yleiskäyttöisen kvanttitietokoneen toteutukseen liittyvä tekninen ongelma, jonka osittainenkin nujertaminen edellyttänee erittäin hyvää kubittien eristystä, runsaasti "ylimääräisiä" suorituskertoja ja hyvin runsaasti "ylimääräisiä" kubitteja.

        Kolmannessa vaiheessa suoritetaan mittaus, eli luetaan laskutoimituksen tulos. Tekijöihinjaon tapauksessa kohdennetaan esimerkiksi Fourier-muunnos ensimmäisen muistin sisältöön, mikä tuo esiin eri kvanttitilojen interferenssin ja sen jälkeen mitataan kyseisen muistin sisältö, mikä paljastaa tekijöihinjaon tulokset. Tämä on Shorin ehdottama alkuperäinen tapa. Vaihtoehtoinen tapa kaiketi olisi lukea tulos toisesta muistista Ekertin ja Barencon esittämällä tavalla ennen Fourier-muunnosta. Vaikka toinen näistä kvanttimuisteista on siis sellainen, että sen tulosta ei tarvitse lukea lainkaan, niin sen sisältö vaikuttaa silti interferenssin vuoksi muuhun laskutoimitukseen ja käytännössä Fourier-muunnos täytyy suorittaa, vaikka klassisen logiikan mukaan se voikin vaikuttaa tarpeettomalta välivaiheelta. Fourier-muunnos saadaan aikaan esimerkiksi joukolla toisiinsa sopivalla tavalla kytkettyjä Hadamard-portteja ja käsiteltävien kubittien sisältöä kuvaavan tiedon ehdollisen vaihesiirron tekeviä portteja. Käytännössä prosessi on mutkikkaampi, mutta tämä on vain yksinkertaistettu yleisselitys.

        Tekijöihinjaon tapauksessa muistin muodostavien kubittien kvanttitilojen (esim. spin tai polarisaation suunta) mittaukset siis paljastavat jaettavan tai jaettavien lukujen tekijät. Jos tekijöihinjaettava luku on hyvin suuri, niin sen tekijöiden paljastaminen ei olisi mahdollista klassisen laskennan avulla käytännön kannalta hyödyllisesti, koska tekijöihinjako on laskennalliselta vaativuudeltaan luokassa NP, eli ratkaisua ei voitaisi yleisessä tapauksessa saavuttaa polynomisessa ajassa. Paras klassinen algoritmi kyseiselle ongelmalle on yleinen lukukuntaseula, jonka aikavaativuus on O(exp((64b/9)^(1/3) (log(b))^(2/3) )), jossa b on jaettavan luvun vaatima muistitila bitteinä, eli sub-eksponentiaalinen, kun taas Shorin algoritmin aikavaativuus on luokassa BQP (bounded-error quantum polynomial time) ja on vain O(b^3); eli polynominen, muistivaativuuden ollessa vain O(b) ja tarvittavien kvanttiporttien määrän ollessa luokkaa O((log N)^2(log log N)(log log log N)), jossa N on tekijöihin jaettava luku.

        Monimaailmatulkinnan mukaan tämän potentiaalisesti suunnattoman laskennallisen tehonlisäyksen mahdollistaa se, että kvanttitietokone suorittaa kussakin keskenään laskennan osana vuorovaikuttavassa maailmankaikkeudessa yhden noista syötetiedoiltaan eroavista laskutoimituksista. Muut kvanttimekaaniset tulkinnat toisaalta eivät käsittääkseni osaa selittää sitä, mihin kvanttilaskennan käytännössä osoitettu toimivuus ylipäätään ja sen teoreettisesti todistettu potentiaalinen teho erityisesti voisivat perustua. Tämä oli hyödyllinen, mutta vastattavaksi aika työläs kysymys, joten täytyy jatkaa keskustelua muilta osin vasta myöhemmin.


      • metafysiikkaa
        FirstMatter kirjoitti:

        ""Kaikki mahdolliset vaihtoehdot toteutuvat kvanttilogiikan mukaan väistämättä. Tämähän on todistettu kvanttitietokoneiden avulla.""
        "Miten? Jos kaikki vaihtoehdot toteutuvat niin miten kvanttitietokone voi tuottaa laskutoimitukseen oikean tuloksen?"

        Kvanttitietokoneiden toiminta on suhteellisen mutkikasta. Niiden kvanttilaskentaa hyödyntävät kvanttipiirit suunnitellaan erityisesti siten, että tietyn ongelmatyypin ratkaisussa on mahdollista hyödyntää kvanttilaskentaa. Ensimmäisessä vaiheessa tietyn ongelman ratkaisuun liittyvät kaikki mahdolliset lähtötiedot tallennetaan ensimmäiseen kvanttimuistiin; eli kubittien joukkoon, noiden lähtötietojen superpositiona. Tämä saadaan aikaan esimerkiksi käyttämällä joukkoa toisiinsa kytkettyjä Hadamard-portteja, joista kukin suorittaa yleistetyn loogisen operaation, joka vastaa NOT:n neliöjuurta.

        Toisessa vaiheessa suoritetaan varsinainen kvanttilaskenta. Eli, esimerkiksi tekijöihinjaon mahdollistavan Shorin kvanttialgoritmin logiikan toteuttavassa piiristössä laskutoimituksena on: (x^a) mod n, jossa x on satunnaisesti valittu kokonaisluku ja a on ensimmäisen kvanttimuistin sisältö. Kvanttitietokone suorittaa piiriensä ohjaamana kaikki mahdolliset tarvittavat erilaiset laskutoimitukset periaatteessa viiveettömästi. Aikaa tietysti kuitenkin käytännössä kuluu siihen kun signaalit joutuvat kulkemaan piireissä klassiseen tapaan. Käytännössä tuloksen esiinkaivaminen vie myös aikaa, koska dekoherenssista johtuen joudutaan turvautumaan kvanttivirheenkorjaukseen, eli suorittamaan samoja laskutoimituksia niin monta kertaa, että tulos on tilastollisesti riittävän suurella todennäköisyydellä oikea. Dekoherenssi on suurin yleiskäyttöisen kvanttitietokoneen toteutukseen liittyvä tekninen ongelma, jonka osittainenkin nujertaminen edellyttänee erittäin hyvää kubittien eristystä, runsaasti "ylimääräisiä" suorituskertoja ja hyvin runsaasti "ylimääräisiä" kubitteja.

        Kolmannessa vaiheessa suoritetaan mittaus, eli luetaan laskutoimituksen tulos. Tekijöihinjaon tapauksessa kohdennetaan esimerkiksi Fourier-muunnos ensimmäisen muistin sisältöön, mikä tuo esiin eri kvanttitilojen interferenssin ja sen jälkeen mitataan kyseisen muistin sisältö, mikä paljastaa tekijöihinjaon tulokset. Tämä on Shorin ehdottama alkuperäinen tapa. Vaihtoehtoinen tapa kaiketi olisi lukea tulos toisesta muistista Ekertin ja Barencon esittämällä tavalla ennen Fourier-muunnosta. Vaikka toinen näistä kvanttimuisteista on siis sellainen, että sen tulosta ei tarvitse lukea lainkaan, niin sen sisältö vaikuttaa silti interferenssin vuoksi muuhun laskutoimitukseen ja käytännössä Fourier-muunnos täytyy suorittaa, vaikka klassisen logiikan mukaan se voikin vaikuttaa tarpeettomalta välivaiheelta. Fourier-muunnos saadaan aikaan esimerkiksi joukolla toisiinsa sopivalla tavalla kytkettyjä Hadamard-portteja ja käsiteltävien kubittien sisältöä kuvaavan tiedon ehdollisen vaihesiirron tekeviä portteja. Käytännössä prosessi on mutkikkaampi, mutta tämä on vain yksinkertaistettu yleisselitys.

        Tekijöihinjaon tapauksessa muistin muodostavien kubittien kvanttitilojen (esim. spin tai polarisaation suunta) mittaukset siis paljastavat jaettavan tai jaettavien lukujen tekijät. Jos tekijöihinjaettava luku on hyvin suuri, niin sen tekijöiden paljastaminen ei olisi mahdollista klassisen laskennan avulla käytännön kannalta hyödyllisesti, koska tekijöihinjako on laskennalliselta vaativuudeltaan luokassa NP, eli ratkaisua ei voitaisi yleisessä tapauksessa saavuttaa polynomisessa ajassa. Paras klassinen algoritmi kyseiselle ongelmalle on yleinen lukukuntaseula, jonka aikavaativuus on O(exp((64b/9)^(1/3) (log(b))^(2/3) )), jossa b on jaettavan luvun vaatima muistitila bitteinä, eli sub-eksponentiaalinen, kun taas Shorin algoritmin aikavaativuus on luokassa BQP (bounded-error quantum polynomial time) ja on vain O(b^3); eli polynominen, muistivaativuuden ollessa vain O(b) ja tarvittavien kvanttiporttien määrän ollessa luokkaa O((log N)^2(log log N)(log log log N)), jossa N on tekijöihin jaettava luku.

        Monimaailmatulkinnan mukaan tämän potentiaalisesti suunnattoman laskennallisen tehonlisäyksen mahdollistaa se, että kvanttitietokone suorittaa kussakin keskenään laskennan osana vuorovaikuttavassa maailmankaikkeudessa yhden noista syötetiedoiltaan eroavista laskutoimituksista. Muut kvanttimekaaniset tulkinnat toisaalta eivät käsittääkseni osaa selittää sitä, mihin kvanttilaskennan käytännössä osoitettu toimivuus ylipäätään ja sen teoreettisesti todistettu potentiaalinen teho erityisesti voisivat perustua. Tämä oli hyödyllinen, mutta vastattavaksi aika työläs kysymys, joten täytyy jatkaa keskustelua muilta osin vasta myöhemmin.

        Kiitos vastauksesta. Olen jonkun verran aikoinaan ohjelmoinut assemblerilla ja tuo kvanttilaskennan tulos kuullostaa varsin monimutkaiselta perinteisiin tietokoneisiin verrattuna.

        Max Tegmark taisi esittää tuon väitteen että kvanttitietokone todistaa monimaailmamallin oikeaksi. Eikö se kvanttilaskennan tulos ole myös tulkittavissa aaltofunktion romahdukseksi yhtä hyvin? Sehän on kai selvää että tämä maailmankaikkeus ei missään tapauksessa ole klassisen fysiikan mukainen. Monimaailmatulkinnassa käsittääkseni vain oletetaan lukematon määrä deterministisesti toimivia klassisen fysiikan maailmankaikkeuksia eli sama asia hahmotetaan hieman eri tavalla.

        Myös sellainen monimaailma voisi olla mahdollinen jossa tietoisuus virittäytyy ikään kuin eri aallonpituuksille radiovirittimen tapaan ja kokee erilaisia maailmoja (kuten skitsofreniassa, unissa ja hallusinaatioissa) jos lähdetään tarkastelemaan asiaa tietoisuuden eikä perinteisen fysiikkatieteen kannalta. Tietoisuuden kannalta fysiikan tutkima todellisuus on vain tässä kulttuurissa elävien ihmisten konsensustodellisuutta joka koetaan samalla tavalla fysikaalisena kuin unessa koetaan unimaailma todellisena.

        ....

        Romahdusmalleissa ymmärtääkseni ei postuloida lisää maailmankaikkeuksia vaan annetaan mahdollisuuksille eli potentiaalisille tapahtumille ontologinen olemassaolo ja merkitys.

        Tietoisuuden kysymyksen kannalta tällä on suuri merkitys koska se mahdollistaa aidon valinnan eri vaihtoehtojen välillä eli jonkinlaisen vapaan tahdon mitä klassinen deterministinen maailmankaikkeus eikä sen monistus monimaailmoiksi mahdollista. Tietoisuus ei varsinaisesti vaikuta laisinkaan ns. fysikaalisessa maailmassa vaan nimenomaan ns. potentiaalisen maailman asukki eli ts. tietoisuuden vaikutus ilmenee vain ja ainoastaan ns. aaltofunktion romahduksen kautta.

        Koska romahdusmalli ottaa huomioon ajan suunnan niin aito vapaa tahto edellyttää moniulotteisen ajan. Yksiulotteinen (jana) ajassa tapahtumat ovat deterministisiä ja täysin mekaanisia ja epäjärjestys voi ainoastaan lisääntyä.

        2-ulotteisessa ajassa (taso) on olemassa useita vaihtoehtoja mutta siinäkin niiden vaihtoehtojen kokonaismäärä vähenee ajan kuluessa paitsi tilanteessa missä kaikki energia on potentiaalienergiaa eikä mitään tapahdu.

        3-ulotteinen aika mahdollistaa potentiaalisuuksien uusiutumisen syklisesti eli dekoherenssia seuraa aina rekoherenssi ja se mahdollistaa olioiden identiteetin säilymisen ja uusiutumisen.

        Ilman jonkinlaista vapaata tahtoa tietoisuus on pelkkä haamu ja korkeintaan pelkästään ulkopuolinen tarkkailija ilman minkäänlaista vaikutusmahdollisuutta tapahtumien kulkuun.

        Tämä siis tietenkin edellyttää että tietoisuus ei ole sama asia kuin fysikaalisten säännönmukaisuuksien determinoima aivo - tai hermostokudos vaikka yksilön kokema tietoisuus varmaan suuressa määrin onkin aivojen ja epälokaalin tietoisuuden yhteisvaikutusta ja takaisinkytkentää fysikaalisen ympäristön kanssa.

        Ketjun aihe olisi tietysti huomattavasti helpompi käsitellä pelkästään momimaailmamallin vs. romahdustulkintojen suhteen. Tietoisuus on tavallaan aina jonkinlainen ulkopuolinen häirikkö tai Pandoran lipas puhtaasti luonnontieteelliseltä kannalta tarkasteltuna.

        Sosologiaa ja ihmisjoukkojen käyttäytymistä varmasti voisi ja voikin mallintaa fysiikkatieteen tarkkuudella ellei aika ajoin ilmeistyisi poikkeuksellisia ihmisyksilöitä jotka romahduttavat koko vallitsevan systeemin. Historian aikana näitä on ollut useita (esim. Tsingis kaani, Napoleon, Hitler jne). Isaac Asimov kuvasi tällaista teossarjassaan Säätiö aika mielenkiintoisella tavalla.

        Tuskin edes tulevaisuuden kvanttitietokoneet pystyvät ennustamaan tulevaisuutta aukottoman tarkasti kun aina jostain voi kehkeytyä kaoottinen prosessi joka pakottaa järjestelmän uusiutumaan. Kvanttitietokoneiden uusi uljas maailma tuskin on kovin miellyttävä paikkaa elää ihmisyksilön kannalta kun tämä perinteisen tietotekniikan kyllästämä kulttuurikin ilmeisesti heikentää ihmisten kognitiivisia kykyjä jka muokkaa ajattelua binääriseen ajatteluun jossa ajatellaan äärimmäisten vastakohtien kautta eikä ymmärretä että maailma on oikeasti analoginen eli vastkohtien välillä on paljon variaatiota.

        Heh. Tulipa pitkä jaaritus mutta aihe on monin tavoin mielenkiintoinen ja innostaa spekuloimaan melko villistikin....


      • FirstMatter
        metafysiikkaa kirjoitti:

        "Monimaailmatulkinta tuntuu houkuttelevalta ja elegantilta vaihtoehdolta mm., koska se selittää havainnot havaittujen hiukkastyyppien kautta ilman oletuksia tyhjästä syntyvistä vaikutuksista."

        Jostain syystä fysiikassa se tyhjästä nyhjääminen on joka tapauksessa käytössä Big bangin tapauksessa. Alkeishiukkasia ei ole kukaan koskaan suoraan havainnut joten on aika kyseenalaista puhua "havaituista hiukkastyypeistä" kun koko fysiikan standardimalli on matemaattinen konstruktio eli fysiikankin pohjalla on metafyysinen ja abstrakti mentaalinen konstruktio ja muutenkaan mitään puhtaasti empiiristä havaintoa ei ole olemassakaan vaan kaikki ns. aistihavainnot ovat aina vahvasti prosessoituja ja niiden tulkinta aina riippuvaista valitusta käsitejärjestelmästä ja kielestä.

        "Toisaalta, filosofit ovat vaivanneet päätään tietoisuuspohdinnoilla vuosituhansia, eivätkä tulokset ole olleet kovin kummoisia, joten ainakaan ei kannata pompata siihen johtopäätökseen, että tietoisuudella olisi mitään aineesta riippumatonta vaikutusta. "

        Fysiikan tulokset ovat tuottaneet toimivaa tekniikkaa nimenomaan juuri sen takia että tutkimuskohde on niin säännönmukainen ja mekaaninen eli suhteellisen helposti matemaattisesti mallinnettavissa verrattuna esim. eliöiden ja ihmisten toimintaan.

        Arkikielessä tietoisuuden tuottamia fysikaalisia kokonaisuuksia kutsutaan "keinotekoisiksi" kun taas ns. "luonnonlakien" tuottamia vastaavia kokonaisuuksia "luonnollisiksi" vaikka periaatteessa kai molemmat pitäisi tulkita luonnollisiksi. Jossain mielessä siis vaistomaisesti koetaan tietoisuuden vaikutus ilmiöihin jollain tavalla luonnolle vieraana asiana.

        Monimaailmatulkinnan varmaan yksi fyysikoita houkutteleva ominaisuus suhteessa aaltofunktion romahdusmalleihin on että mm-tulkinta on deterministinen kun taas romahdustulkinta ei ole ja mm-tulkinnassa ajalla ei ole suuntaa kun taas romahdus on palautumaton ajan suhteen.

        "Tietoisuuden vaikutusten tutkiminen voi olla enemmän biologian kuin fysiikan aihealueeseen kuuluvaa."

        Niin tietysti mutta ilmiöiden kokonaisuus ei ole sillä tavalla selkeästi lokeroitavissa eri tieteenalojen reviireihin.

        "Monimaailmatulkinta on havaintojen tulkinta aivan kuten muutkin kvanttimekaaniset tulkinnat, eli se perustuu havaintoihin. "

        Kvanttifýsiikan tulkintoja on olemassa toistakymmentä ja ne ovat kaikki sisäisti koherentteja malleja suhteessa havaintoihin eikä niiden välillä ainakaan toistaiseksi voi tehdä valintaa muuten kuin mieltymyksien perusteella eli kyseessä on samalla tavalla makuasia kuin suklaajäätelä vs mansikkajäätelö.

        "Aiemmin jo selitettiin miten kaksoisrakokokeen tulokset ovat selitettävissä aukottomasti fotonien sironnan kautta. "

        Pointti onkin se että erilaisia "aukottomia" taustatarinoita voi kehitellä mutta valinta niiden välillä on edelleen vahvasti makuasia.

        "On hyvin tieteellistä, eleganttia ja käsitteellisesti säästeliästä selittää esimerkiksi fotoneita koskevat havainnot nimenomaan fotonien avulla sen sijaan, että oletettaisiin mystisiä satunnaisvaikutuksia, kuten joissakin muissa kvanttimekaanisissa tulkinnoissa."

        "Tieteellisyys" (?), eleganssi ja käsitteellinen säästeliäisyys ovat luonteeltaan laadullisia makuasioita eikä mitään mallin vahvistuksia sinänsä. Jonkun toisen tulkinnan kannattaja voisi sanoa samat asiat.

        "että oletettaisiin mystisiä satunnaisvaikutuksia, kuten joissakin muissa kvanttimekaanisissa tulkinnoissa"

        Satunnaisvaikutukset ovat todellakin "mystisiä" siinä mielessä että niitä ei voi mekaanisella mallilla kuvata mutta toisaalta satunnaisuuksiakin on erilaisia ja se satunnaisuus voi tarkoittaa että ilmöitä ei riittävästi ymmärretä vielä.

        "jos sironta tapahtuu todellisten hiukkasten välillä, niin niiden täytyy sijaita nimenomaan erillisissä tilaulottuvuuksissa, koska muuten niiden massa tuottaisi mustan aukon tässä ulottuvuudessa ja niitä ei silloin tietenkään voitaisi havaita."

        Mikä on "todellinen hiukkanen"? Asian voi myös hahmottaa niin päin että näennäisesti erilliset hiukkaset ovat saman kokonaisuuden osia jotka vaikuttavat toisiinsa ajasta ja paikasta riippumatta ikäänkuin hyperdimensionaalisesti (kuten lomittumisilmiössä).

        Suhtista ja kvanttiteoriaa ei ole kyetty yhdistämään joten paras jättää ne mustat aukot pois tästä kuvioista kokonaan kun tuo skenaario vaikuttaa melko spekulatiiviselta muutenkin.

        "Miksi oletus siitä, että olemassa voisi olla vain yksi yhtenäinen tilaulottuvuus olisi muka vähemmän metafyysinen väite kuin oletus siitä, että fotonien sijainnit noudattavat nimenomaan tietyntyyppistä tilastollista jakaumaa satunnaisten syiden vuoksi?"
        Ei se olekaan vähemmän metafyysinen kun periaatteessa koko fysiikan käsitejärjestelmä on metafysiikkaa siinä mielessä että se on vain tulkinta havainnoista ja metafysiikasta vapaata "puhdasta" havainnointia ei ole olemassakaan. (jatkuu...)

        "Jostain syystä fysiikassa se tyhjästä nyhjääminen on joka tapauksessa käytössä Big bangin tapauksessa."

        Maailmankaikkeuden ollessa hyvin pieni, sen osien sijainteihin liittyvät epämääräisyydet olivat suurempia kuin sen koko, eli vaikuttaisi siltä, että maailmankaikkeus ei välttämättä olisi koskaan ollut äärettömän pieni, vaan vain suhteellisen pieni.

        "Alkeishiukkasia ei ole kukaan koskaan suoraan havainnut joten on aika kyseenalaista puhua "havaituista hiukkastyypeistä" kun koko fysiikan standardimalli on matemaattinen konstruktio eli fysiikankin pohjalla on metafyysinen ja abstrakti mentaalinen konstruktio ja muutenkaan mitään puhtaasti empiiristä havaintoa ei ole olemassakaan vaan kaikki ns. aistihavainnot ovat aina vahvasti prosessoituja ja niiden tulkinta aina riippuvaista valitusta käsitejärjestelmästä ja kielestä."

        Kovin on pitkä lause, kielellisesti hyvin muotoiltu, monenlaista asiaa, mutta oleellista tässä on se, että tyhjästä syntyvistä vaikutuksista on kaikesta huolimatta vähemmän näyttöä kuin havaituista hiukkastyypeistä. Tässä tarkoitetut havainnot ovat luonteeltaan sellaisia, että samat mittaustulokset havaitaan mittaajasta ja monista muista tekijöistä riippumatta. On siis ilmeistä, että taustalla on tekijöitä, jotka ovat luonteeltaan objektiivisia, koska todennäköisyys sille, että samoja mittaustuloksia saataisiin muuten, olisi hyvin pieni.

        Onkin täysin ilmeistä, että on olemassa jotakin, joka vaikuttaa mittaajien tietoisuuteen tavalla, joka mahdollistaa tuon. Jos tuota vaikutusta yritetään selittää pelkällä tietoisuudella, niin päädytään kehäpäätelmään joka ei selitä mitään, eli kyseessä ei silloin ole tieteellinen selittäminen. Puhtaita havaintoja sen sijaan on olemassa, koska ne ovat edellytys sille, että niitä voidaan alkaa tulkita. Tämä on loogisesti ilmeistä, vaikka havaintojen selitykset kielen kautta sisältävätkin tietysti aina tulkintaa. Kyllä ihminen siis pystyy havaitsemaan ilmankin, että alkaa tulkita tietoisesti havaitsemaansa.

        "Fysiikan tulokset ovat tuottaneet toimivaa tekniikkaa nimenomaan juuri sen takia että tutkimuskohde on niin säännönmukainen ja mekaaninen eli suhteellisen helposti matemaattisesti mallinnettavissa verrattuna esim. eliöiden ja ihmisten toimintaan."

        Kyllä. Kyse on tarkasteltavan järjestelmän osien lukumäärästä. Osien lukumäärän kasvaessa lineaarisesti, niiden välisten mahdollisten vuorovaikutusten määrä kasvaa eksponentiaalisesti. Tuossa ei kuitenkaan ole mitään mystistä, vaan kyse on vain kompleksisuudesta. Ihmisenkin osat noudattavat luonnonlakeja.

        "Arkikielessä tietoisuuden tuottamia fysikaalisia kokonaisuuksia kutsutaan "keinotekoisiksi" kun taas ns. "luonnonlakien" tuottamia vastaavia kokonaisuuksia "luonnollisiksi" vaikka periaatteessa kai molemmat pitäisi tulkita luonnollisiksi."

        Kyllä.

        "Jossain mielessä siis vaistomaisesti koetaan tietoisuuden vaikutus ilmiöihin jollain tavalla luonnolle vieraana asiana."

        Kyse on siitä, että ihminen on etääntynyt lajina suorasta havaitsemisesta ja luonut käsitteellisen kahtiajaon itsensä ja luonnon välille. Ilmeisesti itsetietoisuuden kehittyminen erityisesti siitä kommunikoinnin asteelle edellyttää tuollaisen kahtiajaon. Eli, vaikka ihmisen ja luonnon välillä ei olisikaan mitään oleellista eroa, niin olisi epätodennäköisempää, että keskustelun kohteena olisi tuo identtisyys kuin sen puute.


      • FirstMatter
        metafysiikkaa kirjoitti:

        "Monimaailmatulkinta tuntuu houkuttelevalta ja elegantilta vaihtoehdolta mm., koska se selittää havainnot havaittujen hiukkastyyppien kautta ilman oletuksia tyhjästä syntyvistä vaikutuksista."

        Jostain syystä fysiikassa se tyhjästä nyhjääminen on joka tapauksessa käytössä Big bangin tapauksessa. Alkeishiukkasia ei ole kukaan koskaan suoraan havainnut joten on aika kyseenalaista puhua "havaituista hiukkastyypeistä" kun koko fysiikan standardimalli on matemaattinen konstruktio eli fysiikankin pohjalla on metafyysinen ja abstrakti mentaalinen konstruktio ja muutenkaan mitään puhtaasti empiiristä havaintoa ei ole olemassakaan vaan kaikki ns. aistihavainnot ovat aina vahvasti prosessoituja ja niiden tulkinta aina riippuvaista valitusta käsitejärjestelmästä ja kielestä.

        "Toisaalta, filosofit ovat vaivanneet päätään tietoisuuspohdinnoilla vuosituhansia, eivätkä tulokset ole olleet kovin kummoisia, joten ainakaan ei kannata pompata siihen johtopäätökseen, että tietoisuudella olisi mitään aineesta riippumatonta vaikutusta. "

        Fysiikan tulokset ovat tuottaneet toimivaa tekniikkaa nimenomaan juuri sen takia että tutkimuskohde on niin säännönmukainen ja mekaaninen eli suhteellisen helposti matemaattisesti mallinnettavissa verrattuna esim. eliöiden ja ihmisten toimintaan.

        Arkikielessä tietoisuuden tuottamia fysikaalisia kokonaisuuksia kutsutaan "keinotekoisiksi" kun taas ns. "luonnonlakien" tuottamia vastaavia kokonaisuuksia "luonnollisiksi" vaikka periaatteessa kai molemmat pitäisi tulkita luonnollisiksi. Jossain mielessä siis vaistomaisesti koetaan tietoisuuden vaikutus ilmiöihin jollain tavalla luonnolle vieraana asiana.

        Monimaailmatulkinnan varmaan yksi fyysikoita houkutteleva ominaisuus suhteessa aaltofunktion romahdusmalleihin on että mm-tulkinta on deterministinen kun taas romahdustulkinta ei ole ja mm-tulkinnassa ajalla ei ole suuntaa kun taas romahdus on palautumaton ajan suhteen.

        "Tietoisuuden vaikutusten tutkiminen voi olla enemmän biologian kuin fysiikan aihealueeseen kuuluvaa."

        Niin tietysti mutta ilmiöiden kokonaisuus ei ole sillä tavalla selkeästi lokeroitavissa eri tieteenalojen reviireihin.

        "Monimaailmatulkinta on havaintojen tulkinta aivan kuten muutkin kvanttimekaaniset tulkinnat, eli se perustuu havaintoihin. "

        Kvanttifýsiikan tulkintoja on olemassa toistakymmentä ja ne ovat kaikki sisäisti koherentteja malleja suhteessa havaintoihin eikä niiden välillä ainakaan toistaiseksi voi tehdä valintaa muuten kuin mieltymyksien perusteella eli kyseessä on samalla tavalla makuasia kuin suklaajäätelä vs mansikkajäätelö.

        "Aiemmin jo selitettiin miten kaksoisrakokokeen tulokset ovat selitettävissä aukottomasti fotonien sironnan kautta. "

        Pointti onkin se että erilaisia "aukottomia" taustatarinoita voi kehitellä mutta valinta niiden välillä on edelleen vahvasti makuasia.

        "On hyvin tieteellistä, eleganttia ja käsitteellisesti säästeliästä selittää esimerkiksi fotoneita koskevat havainnot nimenomaan fotonien avulla sen sijaan, että oletettaisiin mystisiä satunnaisvaikutuksia, kuten joissakin muissa kvanttimekaanisissa tulkinnoissa."

        "Tieteellisyys" (?), eleganssi ja käsitteellinen säästeliäisyys ovat luonteeltaan laadullisia makuasioita eikä mitään mallin vahvistuksia sinänsä. Jonkun toisen tulkinnan kannattaja voisi sanoa samat asiat.

        "että oletettaisiin mystisiä satunnaisvaikutuksia, kuten joissakin muissa kvanttimekaanisissa tulkinnoissa"

        Satunnaisvaikutukset ovat todellakin "mystisiä" siinä mielessä että niitä ei voi mekaanisella mallilla kuvata mutta toisaalta satunnaisuuksiakin on erilaisia ja se satunnaisuus voi tarkoittaa että ilmöitä ei riittävästi ymmärretä vielä.

        "jos sironta tapahtuu todellisten hiukkasten välillä, niin niiden täytyy sijaita nimenomaan erillisissä tilaulottuvuuksissa, koska muuten niiden massa tuottaisi mustan aukon tässä ulottuvuudessa ja niitä ei silloin tietenkään voitaisi havaita."

        Mikä on "todellinen hiukkanen"? Asian voi myös hahmottaa niin päin että näennäisesti erilliset hiukkaset ovat saman kokonaisuuden osia jotka vaikuttavat toisiinsa ajasta ja paikasta riippumatta ikäänkuin hyperdimensionaalisesti (kuten lomittumisilmiössä).

        Suhtista ja kvanttiteoriaa ei ole kyetty yhdistämään joten paras jättää ne mustat aukot pois tästä kuvioista kokonaan kun tuo skenaario vaikuttaa melko spekulatiiviselta muutenkin.

        "Miksi oletus siitä, että olemassa voisi olla vain yksi yhtenäinen tilaulottuvuus olisi muka vähemmän metafyysinen väite kuin oletus siitä, että fotonien sijainnit noudattavat nimenomaan tietyntyyppistä tilastollista jakaumaa satunnaisten syiden vuoksi?"
        Ei se olekaan vähemmän metafyysinen kun periaatteessa koko fysiikan käsitejärjestelmä on metafysiikkaa siinä mielessä että se on vain tulkinta havainnoista ja metafysiikasta vapaata "puhdasta" havainnointia ei ole olemassakaan. (jatkuu...)

        "Monimaailmatulkinnan varmaan yksi fyysikoita houkutteleva ominaisuus suhteessa aaltofunktion romahdusmalleihin on että mm-tulkinta on deterministinen kun taas romahdustulkinta ei ole ja mm-tulkinnassa ajalla ei ole suuntaa kun taas romahdus on palautumaton ajan suhteen."

        Kyllä. Mielestäni tieteelliseen selittämiseen kuuluu se, että pyritään selittämään havaittujen tapahtumien syyt entistä paremmin. Tilanne, jossa selitysmalli sisältää satunnaismuuttujia ei voi olla tavoite, vaan vain välivaihe ja tavoitteena tulee olla korvata se sellaisella tarkemmalla mallilla, jonka antamat ennusteet ovat testattavissa. On tietysti mahdollista, että tuossa ei onnistuta, mutta se olisi tieteen kannalta valitettavaa, koska etenkin pysyvillä aukkokohdilla on taipumus täyttyä kaikenkirjavalla epätieteellisellä sisällöllä.

        "Kvanttifýsiikan tulkintoja on olemassa toistakymmentä ja ne ovat kaikki sisäisti koherentteja malleja suhteessa havaintoihin eikä niiden välillä ainakaan toistaiseksi voi tehdä valintaa muuten kuin mieltymyksien perusteella eli kyseessä on samalla tavalla makuasia kuin suklaajäätelä vs mansikkajäätelö."

        Kyllä siinä merkityksessä, että niitä ei olla voitu falsifioida empirian perusteella.

        ""Tieteellisyys" (?), eleganssi ja käsitteellinen säästeliäisyys ovat luonteeltaan laadullisia makuasioita eikä mitään mallin vahvistuksia sinänsä. Jonkun toisen tulkinnan kannattaja voisi sanoa samat asiat."

        Käsitteellinen säästeliäisyys (Occamin periaate) on yksi tieteellisten teorioiden tärkeimmistä laatukriteereistä; ehkäpä jopa toiseksi tärkein falsifioitavuuden jälkeen ja kyllähän sen perusteella monet teoriat voidaan laittaa hyvin perustellusti järjestykseen. Tietysti selitysvoima on myös tärkeää ja kuten mainittiin, monimaailmamalli suoriutuu parhaiten kvanttilaskennan toimivuuden selittämisestä. Vaikka muutkin mallit ehkä voidaan jonkinlaisella vääntelehtimisellä saada selittämään asiat ainakin jotenkin sinne päin olettaen joitakin hokkus-pokkuksia, niin monimaailmamalli on siinä merkityksessä elegantti, että tarvittava käsitteellinen vääntelehtiminen minimoituu. Maailmankaikkeuksia tosin tarvitaan erittäin suuri määrä, mutta kyseessä on vain määrällinen, eikä laadullinen asia, eli se ei ole Occamilainen rasite.

        "Satunnaisvaikutukset ovat todellakin "mystisiä" siinä mielessä että niitä ei voi mekaanisella mallilla kuvata mutta toisaalta satunnaisuuksiakin on erilaisia ja se satunnaisuus voi tarkoittaa että ilmöitä ei riittävästi ymmärretä vielä."

        Tässä tullaan myös kuvausten kompleksisuuteen, eli vaikka monimaailmamalli onkin deterministinen, niin mikään ei takaa, että sekään olisi vielä todellisuuden täydellinen kuvaus. Voi olla siis niin, että vaikka todellisuus onkin täysin deterministinen, niin sen täydellinen kuvaus ei edes ole mahdollista.


      • FirstMatter
        metafysiikkaa kirjoitti:

        "Monimaailmatulkinta tuntuu houkuttelevalta ja elegantilta vaihtoehdolta mm., koska se selittää havainnot havaittujen hiukkastyyppien kautta ilman oletuksia tyhjästä syntyvistä vaikutuksista."

        Jostain syystä fysiikassa se tyhjästä nyhjääminen on joka tapauksessa käytössä Big bangin tapauksessa. Alkeishiukkasia ei ole kukaan koskaan suoraan havainnut joten on aika kyseenalaista puhua "havaituista hiukkastyypeistä" kun koko fysiikan standardimalli on matemaattinen konstruktio eli fysiikankin pohjalla on metafyysinen ja abstrakti mentaalinen konstruktio ja muutenkaan mitään puhtaasti empiiristä havaintoa ei ole olemassakaan vaan kaikki ns. aistihavainnot ovat aina vahvasti prosessoituja ja niiden tulkinta aina riippuvaista valitusta käsitejärjestelmästä ja kielestä.

        "Toisaalta, filosofit ovat vaivanneet päätään tietoisuuspohdinnoilla vuosituhansia, eivätkä tulokset ole olleet kovin kummoisia, joten ainakaan ei kannata pompata siihen johtopäätökseen, että tietoisuudella olisi mitään aineesta riippumatonta vaikutusta. "

        Fysiikan tulokset ovat tuottaneet toimivaa tekniikkaa nimenomaan juuri sen takia että tutkimuskohde on niin säännönmukainen ja mekaaninen eli suhteellisen helposti matemaattisesti mallinnettavissa verrattuna esim. eliöiden ja ihmisten toimintaan.

        Arkikielessä tietoisuuden tuottamia fysikaalisia kokonaisuuksia kutsutaan "keinotekoisiksi" kun taas ns. "luonnonlakien" tuottamia vastaavia kokonaisuuksia "luonnollisiksi" vaikka periaatteessa kai molemmat pitäisi tulkita luonnollisiksi. Jossain mielessä siis vaistomaisesti koetaan tietoisuuden vaikutus ilmiöihin jollain tavalla luonnolle vieraana asiana.

        Monimaailmatulkinnan varmaan yksi fyysikoita houkutteleva ominaisuus suhteessa aaltofunktion romahdusmalleihin on että mm-tulkinta on deterministinen kun taas romahdustulkinta ei ole ja mm-tulkinnassa ajalla ei ole suuntaa kun taas romahdus on palautumaton ajan suhteen.

        "Tietoisuuden vaikutusten tutkiminen voi olla enemmän biologian kuin fysiikan aihealueeseen kuuluvaa."

        Niin tietysti mutta ilmiöiden kokonaisuus ei ole sillä tavalla selkeästi lokeroitavissa eri tieteenalojen reviireihin.

        "Monimaailmatulkinta on havaintojen tulkinta aivan kuten muutkin kvanttimekaaniset tulkinnat, eli se perustuu havaintoihin. "

        Kvanttifýsiikan tulkintoja on olemassa toistakymmentä ja ne ovat kaikki sisäisti koherentteja malleja suhteessa havaintoihin eikä niiden välillä ainakaan toistaiseksi voi tehdä valintaa muuten kuin mieltymyksien perusteella eli kyseessä on samalla tavalla makuasia kuin suklaajäätelä vs mansikkajäätelö.

        "Aiemmin jo selitettiin miten kaksoisrakokokeen tulokset ovat selitettävissä aukottomasti fotonien sironnan kautta. "

        Pointti onkin se että erilaisia "aukottomia" taustatarinoita voi kehitellä mutta valinta niiden välillä on edelleen vahvasti makuasia.

        "On hyvin tieteellistä, eleganttia ja käsitteellisesti säästeliästä selittää esimerkiksi fotoneita koskevat havainnot nimenomaan fotonien avulla sen sijaan, että oletettaisiin mystisiä satunnaisvaikutuksia, kuten joissakin muissa kvanttimekaanisissa tulkinnoissa."

        "Tieteellisyys" (?), eleganssi ja käsitteellinen säästeliäisyys ovat luonteeltaan laadullisia makuasioita eikä mitään mallin vahvistuksia sinänsä. Jonkun toisen tulkinnan kannattaja voisi sanoa samat asiat.

        "että oletettaisiin mystisiä satunnaisvaikutuksia, kuten joissakin muissa kvanttimekaanisissa tulkinnoissa"

        Satunnaisvaikutukset ovat todellakin "mystisiä" siinä mielessä että niitä ei voi mekaanisella mallilla kuvata mutta toisaalta satunnaisuuksiakin on erilaisia ja se satunnaisuus voi tarkoittaa että ilmöitä ei riittävästi ymmärretä vielä.

        "jos sironta tapahtuu todellisten hiukkasten välillä, niin niiden täytyy sijaita nimenomaan erillisissä tilaulottuvuuksissa, koska muuten niiden massa tuottaisi mustan aukon tässä ulottuvuudessa ja niitä ei silloin tietenkään voitaisi havaita."

        Mikä on "todellinen hiukkanen"? Asian voi myös hahmottaa niin päin että näennäisesti erilliset hiukkaset ovat saman kokonaisuuden osia jotka vaikuttavat toisiinsa ajasta ja paikasta riippumatta ikäänkuin hyperdimensionaalisesti (kuten lomittumisilmiössä).

        Suhtista ja kvanttiteoriaa ei ole kyetty yhdistämään joten paras jättää ne mustat aukot pois tästä kuvioista kokonaan kun tuo skenaario vaikuttaa melko spekulatiiviselta muutenkin.

        "Miksi oletus siitä, että olemassa voisi olla vain yksi yhtenäinen tilaulottuvuus olisi muka vähemmän metafyysinen väite kuin oletus siitä, että fotonien sijainnit noudattavat nimenomaan tietyntyyppistä tilastollista jakaumaa satunnaisten syiden vuoksi?"
        Ei se olekaan vähemmän metafyysinen kun periaatteessa koko fysiikan käsitejärjestelmä on metafysiikkaa siinä mielessä että se on vain tulkinta havainnoista ja metafysiikasta vapaata "puhdasta" havainnointia ei ole olemassakaan. (jatkuu...)

        "Mikä on "todellinen hiukkanen"? Asian voi myös hahmottaa niin päin että näennäisesti erilliset hiukkaset ovat saman kokonaisuuden osia jotka vaikuttavat toisiinsa ajasta ja paikasta riippumatta ikäänkuin hyperdimensionaalisesti (kuten lomittumisilmiössä)."

        Todellisia hiukkasia ovat ne, jotka voivat suorittaa sellaista laskentaa, jota voidaan käyttää pienentämään entropiaa paikallisesti. Jos tarkoitat 'samalla kokonaisuudella' yksittäistä maailmankaikkeutta, niin kvanttilaskennan teoreettinen teho on käytännössä mahdollista selittää vain sellaisella maailmankaikkeudella, jossa olisi 2^q alkeishiukkasta, jossa q on kvanttilaskentaan osallistuvien alkeishiukkasten määrä. Eli, tarvittavien hiukkasten määrä on kvanttilaskentaan osallistuvien hiukkasten lukumäärän eksponenttifunktio.

        Näkyvän maailmankaikkeuden koko massa riittäisi selittämään vain noin 300:n kvanttilaskentaan osallistuvan alkeishiukkasen toiminnan. Koska todellisuus on kvanttimekaaninen, niin jokainen hiukkanen kuitenkin on potentiaalinen kvanttilaskentaan osallistuva hiukkanen ja selittäviä hiukkasia tarvitaan siis noin 2^(10^90), mikä on luku, jossa on suuruusluokaltaan yhtä monta nollaa kuin näkyvässä maailmankaikkeudessa on alkeishiukkasia.

        Periaatteessa nuo tarvittavat hiukkaset voisivat sijaita samassakin tila-avaruudessa, jolloin voitaisiin puhua maailmankaikkeudesta ilman rinnakkaisia maailmankaikkeuksia termin varsinaisessa merkityksessä, mutta silloin noiden hiukkasten tulisi sijaita niin kaukana toisistaan, että niiden massa ei romahtaisi yhteen pisteeseen ennen kuin havaitsevia ja noista ilmiöistä keskustelevia olentoja olisi kehittynyt, jolloin ne olisivat käytännössä niin kaukana toisistaan, että ne olisivat ikään kuin rinnakkaisia maailmankaikkeuksia, koska niiden välillä ei voisi liikkua informaatiota klassisia kanavia pitkin. Tietysti monimaailmatulkintaa vastaan voitaisiin yrittää argumentoida esittämällä esim., että selittäviä hiukkasia ei tarvittaisikaan noin paljoa, koska ne suorittaisivatkin kvanttilaskentaa tms., mutta silloin niiden toiminnan selittämiseen tarvittaisiin vastaavasti suurempi määrä hiukkasia jne.

        "Suhtista ja kvanttiteoriaa ei ole kyetty yhdistämään joten paras jättää ne mustat aukot pois tästä kuvioista kokonaan kun tuo skenaario vaikuttaa melko spekulatiiviselta muutenkin."

        Musta aukko on väistämätön seuraus, jos paikallinen massatiheys on riittävän suuri. Neutronitähdet ovat tiheimpiä tunnettuja ei-luhistuvia tähtitieteellisiä objekteja ja niiden tiheys on noin 10^15 kg./(dm)^3.


      • FirstMatter
        metafysiikkaa kirjoitti:

        "Monimaailmatulkinnassa kvanttimekaanisten ilmiöiden kausaalisena syynä pidetään todellisia hiukkasia, eli hiukkasia, joilla on massaa ja/tai energiaa. "

        Hiukkaset ovat matemaattisia konstruktioita ja kausaliteetti tai determinismi pätee vain suurten hiukkasjoukkojen suhteen tilastollisesti. Tässä yritetään vääntää todellisuutta epätoivoisesti tutun ja turvallisen newtonilaisen mallin suuntaan.

        "pelkät matemaattiset oliot voivat aiheuttaa havaittavia fysikaalisia vaikutuksia, jonkin tuntemattoman mekanismin kautta."

        Maastokartoissa käytetään symboleita kuvaamaan erilaisia maaston ominaisuuksia eikä ne symbolit kirjaimellisesti ole siellä maastossa mutta viittaavat johonkin maaston havaittavissa olevaan ominaisuuteen. Vahvasti matematisoiduissa tieteissä helposti sotketaan se kartta itse maastoon ja aletaan pitää joidenkin asioiden symboleja identtisinä itse ilmiöiden kanssa.

        Alkeishiukkanen on symboli jossa on osittain konkreettiseen havaintoon tai mittaukseen liittyvää mutta myös siihen abstraktiin teoriaan tai malliin liittyviä asioita eli se on ikäänkuin hyperlinkki. On yhtä suuri virhe mieltää se jotenkin konkreettiseksi fysikaaliseksi asiaksi tai vastaavasti puhtaasti teoreettiseksi matemattiseksi asiaksi.

        "Kaikki mahdolliset vaihtoehdot toteutuvat kvanttilogiikan mukaan väistämättä. Tämähän on todistettu kvanttitietokoneiden avulla."

        Miten? Jos kaikki vaihtoehdot toteutuvat niin miten kvanttitietokone voi tuottaa laskutoimitukseen oikean tuloksen?

        "Tietoisuusteorioiden todentaminen edellyttää mittauksia ja objektiivisesti voidaan kätevimmin mitata hiukkasten ja niistä koostuvien rakenteiden ominaisuuksia."

        Tietoisuus on luonteeltaan aina subjektiivista eli omaa kokemuksellisuutta. Toisten olioiden tietoisuus on aina käyttäytymisestä pääteltyä eli virhealtista jos vuorovaikutus tapahtuu esim. netin kautta ilman katsekontaktia eli ts. ns. tietoinen käyttäytyminen on feikattavissa. Millä konstilla päättelet hiukkasten käyttäytymisestä tai systeemin rakenteesta virheettömästi tietoisuuden olemassaolon?

        "Järjestys, jota ainakin siedettävä tietoisuus edellyttää, edellyttää epäjärjestystä jossain muualla. Jos oletetaan, että tietoisuus edellyttää hyvin suurta järjestystä, niin on hyvin luontevaa olettaa, että se edellyttää myös hyvin suurta epäjärjestystä jossain muualla. "

        Auringon järjestäytynyt energia mahdollistaa elämän maapallolla ja vaikka auringon epäjärjestys tässä jonkin verran lisääntyy niin se lisääntyisi joka tapauksessa samalla tavalla vaikka maapallolla ei olisikaan elämää. Kun ympäristössä on korkealaatuista energiaa mistä ottaa niin se epäjärjestyksen lisääntyminen ei tunnu mitenkään kovin dramaattiselta.

        Itse luulen että tietoisuus on pohjimmiltaan enemmän non-lokaali hyperdimensioonaalinen ilmiö ja se subjektiivinen kokemuksellisuus taas sen nonlokaalin asian monimutkainen vuorovaikutussuhde lokaalin fysikaalisen maailman kanssa.

        "Kysymys kuuluukin, että onko tässä maailmankaikkeudessa riittävästi hiukkasia, jotta havaittavan tasoisen järjestyksen muodostumista voitaisiin pitää todennäköisenä vai täytyykö maailmankaikkeuden olla potentiaalisesti avoin jokaisessa Planck-pituuden mittaisessa pisteessään rinnakkaisiin maailmankaikkeuksiin? "

        Nythän ei tiedetä kuinka todennäköistä ja yleistä biologinen elämä on maailmankaikkeudessa ja jos oletetan tietoisuuden edellyttävän biologista elämää niin sitäkään ei tiedetä mikä on tietoisuuden ilmenemisen minimiedellytys eli onko bakteeri tietoinen. Tässä on hyvä taas muistaa että ainoa tietoisuus jonka olemassaolosta voimme olla 100% varmoja on oma tietoisuutemme.

        "Tai yksinkertaisemmin ilmaisten, voiko tietoisuutta muodostua suljetussa järjestelmässä vai tarvitaanko siihen vähintään osittain avoin järjestelmä, jollainen kvanttimekaanisesti vuorovaikuttava rinnakkaisten maailmankaikkeuksien joukko olisi?"

        Jos nyt oletetaan että perusoletuksesi tietoisuuden pohjautumisesta hiukkasiin pitäisi paikkansa niin ei edes tiedetä kuinka paljon energiaa ja hiukkasia tarvitaan minimaalisen tietoisuuden muodostamiseksi joten tässäkääm ei päästä arvailuja pidemmälle.

        "Eli, esimerkiksi tietoisuus edellyttää havaintoja, koska muuten noista havainnoista ei voida olla tietoisia. "

        Kyllä tietoisuus kykenee tuottamaan sisältöjä ilman havaintojakin varsin hyvin esim. unien näkemisessä ja hallusinaatioissa. Tiede on inhimillistä sosiaalista toimintaa eikä mitään jumalallista itseriittoista kuviokelluntaa ilman edellytyksiä.

        " Abstraktien käsitteiden viittaamien olioiden reaalinen olemassaolo edellyttää sitä, että noilla olioilla on sisäinen rakenne, joka koostuu osista."

        Tuo on se reduktionismin uskontunnustus. Yhtä hyvin voisi ajetella että kokonaisuus erilaistuu osiksi kuten esim. solujen jakautumisessa biologisissa ilmiöissä jolloin sen alkusolun geneettinen informaatio siirtyy niihin jakautuneisiin soluihin. Tietoisuuden voisi myös ajatella olevan yksi kokonaisuus joka sitten jakautuu osiksi.

        "Ei se olekaan vähemmän metafyysinen kun periaatteessa koko fysiikan käsitejärjestelmä on metafysiikkaa siinä mielessä että se on vain tulkinta havainnoista ja metafysiikasta vapaata "puhdasta" havainnointia ei ole olemassakaan. "

        Kuulostaa siltä, että sekoitat tuossa fysiikan ja metafysiikan toisiinsa kuin irti päästetty tehosekoitin. Oleellista tässä on se, että fysiikan antamat ennusteet ovat objektiivisesti todennettavissa, mikä tarkoittaa sitä, että ennusteita voidaan käyttää mm. mahdollistamaan entropian paikallinen vähentäminen. Jos ennusteet eivät olisi objektiivisesti paikkansapitäviä, niin tuon onnistuminen olisi tilastollisesti äärimmäisen epätodennäköistä.

        "Hiukkaset ovat matemaattisia konstruktioita ja kausaliteetti tai determinismi pätee vain suurten hiukkasjoukkojen suhteen tilastollisesti. Tässä yritetään vääntää todellisuutta epätoivoisesti tutun ja turvallisen newtonilaisen mallin suuntaan."

        Käsite ja se mihin se viittaa ovat eri asioita. Puhun tässä hiukkasista todellisina olioina, en matemaattisina konstruktioina. Hiukkaset ovat mitä todellisimpia, koska niiden avulla voidaan suorittaa laskentaa. Matemaattisten konstruktioiden avulla sen sijaan ei voida suorittaa laskentaa. Monimaailmatulkinta on deterministinen ja selittää mm. sen miten laskentaa voidaan suorittaa tehokkaasti.

        "Alkeishiukkanen on symboli jossa on osittain konkreettiseen havaintoon tai mittaukseen liittyvää mutta myös siihen abstraktiin teoriaan tai malliin liittyviä asioita eli se on ikäänkuin hyperlinkki..."

        Alkeishiukkanen on käytännössä osoittautunut yhdeksi toimivimmista käsitteistä, joten se ei ole virheellisesti mielletty. Konkreettiset asiat ovat välttämättömyys, joita ilman teoretisointi, filosofointi ja muu kermankuorinta eivät ylipäätään olisi mahdollisia. Esimerkiksi tietoisuus ei pysty suorittamaan yhtään kommunikoitavissa olevaa laskutoimitusta, vaan siihen tarvitaan alkeishiukkasia. Entropian paikallinen vähentäminen ja siten järjestyksen lisääminen on mahdollista vain laskennan avulla ja se taas toisaalta ei edellytä tietoisuutta.

        "Millä konstilla päättelet hiukkasten käyttäytymisestä tai systeemin rakenteesta virheettömästi tietoisuuden olemassaolon?"

        En tiedä, sillä en ole ehtinyt pohtia tuota paljoa. Itseä tarkkailemalla voi kuitenkin havaita, että tietoisuuden taso vaihtelee. Jos oletetaan, että samankaltaisista rakenteista koostuvat olennot omaavat myös samankaltaisia ominaisuuksia, niin silloin myös noilla olennoilla voi olla tietoisuus. Oma arvaukseni on se, että tietoisuuden taso laskee silloin jos järjestelmän osien välinen eläville olennoille tyypillinen tietoliikenne estyy. Tai käänteisesti ilmaisten, tiedon määrällä, jonka olento pystyy tiedostamaan laskennan kannalta hyödynnettävällä tavalla on taipumus kasvaa silloin jos ne osat, joista olennon tietojenkäsittelyjärjestelmä koostuu ovat asianmukaisessa vuorovaikutussuhteessa keskenään. Ilmeisesti tuo asianmukainen vuorovaikutus pitää sisällään mm. sellaisen tiedon vaihtoa, joka kuvaa järjestelmän itsensä tilaa.


      • FirstMatter
        metafysiikkaa kirjoitti:

        "Monimaailmatulkinnassa kvanttimekaanisten ilmiöiden kausaalisena syynä pidetään todellisia hiukkasia, eli hiukkasia, joilla on massaa ja/tai energiaa. "

        Hiukkaset ovat matemaattisia konstruktioita ja kausaliteetti tai determinismi pätee vain suurten hiukkasjoukkojen suhteen tilastollisesti. Tässä yritetään vääntää todellisuutta epätoivoisesti tutun ja turvallisen newtonilaisen mallin suuntaan.

        "pelkät matemaattiset oliot voivat aiheuttaa havaittavia fysikaalisia vaikutuksia, jonkin tuntemattoman mekanismin kautta."

        Maastokartoissa käytetään symboleita kuvaamaan erilaisia maaston ominaisuuksia eikä ne symbolit kirjaimellisesti ole siellä maastossa mutta viittaavat johonkin maaston havaittavissa olevaan ominaisuuteen. Vahvasti matematisoiduissa tieteissä helposti sotketaan se kartta itse maastoon ja aletaan pitää joidenkin asioiden symboleja identtisinä itse ilmiöiden kanssa.

        Alkeishiukkanen on symboli jossa on osittain konkreettiseen havaintoon tai mittaukseen liittyvää mutta myös siihen abstraktiin teoriaan tai malliin liittyviä asioita eli se on ikäänkuin hyperlinkki. On yhtä suuri virhe mieltää se jotenkin konkreettiseksi fysikaaliseksi asiaksi tai vastaavasti puhtaasti teoreettiseksi matemattiseksi asiaksi.

        "Kaikki mahdolliset vaihtoehdot toteutuvat kvanttilogiikan mukaan väistämättä. Tämähän on todistettu kvanttitietokoneiden avulla."

        Miten? Jos kaikki vaihtoehdot toteutuvat niin miten kvanttitietokone voi tuottaa laskutoimitukseen oikean tuloksen?

        "Tietoisuusteorioiden todentaminen edellyttää mittauksia ja objektiivisesti voidaan kätevimmin mitata hiukkasten ja niistä koostuvien rakenteiden ominaisuuksia."

        Tietoisuus on luonteeltaan aina subjektiivista eli omaa kokemuksellisuutta. Toisten olioiden tietoisuus on aina käyttäytymisestä pääteltyä eli virhealtista jos vuorovaikutus tapahtuu esim. netin kautta ilman katsekontaktia eli ts. ns. tietoinen käyttäytyminen on feikattavissa. Millä konstilla päättelet hiukkasten käyttäytymisestä tai systeemin rakenteesta virheettömästi tietoisuuden olemassaolon?

        "Järjestys, jota ainakin siedettävä tietoisuus edellyttää, edellyttää epäjärjestystä jossain muualla. Jos oletetaan, että tietoisuus edellyttää hyvin suurta järjestystä, niin on hyvin luontevaa olettaa, että se edellyttää myös hyvin suurta epäjärjestystä jossain muualla. "

        Auringon järjestäytynyt energia mahdollistaa elämän maapallolla ja vaikka auringon epäjärjestys tässä jonkin verran lisääntyy niin se lisääntyisi joka tapauksessa samalla tavalla vaikka maapallolla ei olisikaan elämää. Kun ympäristössä on korkealaatuista energiaa mistä ottaa niin se epäjärjestyksen lisääntyminen ei tunnu mitenkään kovin dramaattiselta.

        Itse luulen että tietoisuus on pohjimmiltaan enemmän non-lokaali hyperdimensioonaalinen ilmiö ja se subjektiivinen kokemuksellisuus taas sen nonlokaalin asian monimutkainen vuorovaikutussuhde lokaalin fysikaalisen maailman kanssa.

        "Kysymys kuuluukin, että onko tässä maailmankaikkeudessa riittävästi hiukkasia, jotta havaittavan tasoisen järjestyksen muodostumista voitaisiin pitää todennäköisenä vai täytyykö maailmankaikkeuden olla potentiaalisesti avoin jokaisessa Planck-pituuden mittaisessa pisteessään rinnakkaisiin maailmankaikkeuksiin? "

        Nythän ei tiedetä kuinka todennäköistä ja yleistä biologinen elämä on maailmankaikkeudessa ja jos oletetan tietoisuuden edellyttävän biologista elämää niin sitäkään ei tiedetä mikä on tietoisuuden ilmenemisen minimiedellytys eli onko bakteeri tietoinen. Tässä on hyvä taas muistaa että ainoa tietoisuus jonka olemassaolosta voimme olla 100% varmoja on oma tietoisuutemme.

        "Tai yksinkertaisemmin ilmaisten, voiko tietoisuutta muodostua suljetussa järjestelmässä vai tarvitaanko siihen vähintään osittain avoin järjestelmä, jollainen kvanttimekaanisesti vuorovaikuttava rinnakkaisten maailmankaikkeuksien joukko olisi?"

        Jos nyt oletetaan että perusoletuksesi tietoisuuden pohjautumisesta hiukkasiin pitäisi paikkansa niin ei edes tiedetä kuinka paljon energiaa ja hiukkasia tarvitaan minimaalisen tietoisuuden muodostamiseksi joten tässäkääm ei päästä arvailuja pidemmälle.

        "Eli, esimerkiksi tietoisuus edellyttää havaintoja, koska muuten noista havainnoista ei voida olla tietoisia. "

        Kyllä tietoisuus kykenee tuottamaan sisältöjä ilman havaintojakin varsin hyvin esim. unien näkemisessä ja hallusinaatioissa. Tiede on inhimillistä sosiaalista toimintaa eikä mitään jumalallista itseriittoista kuviokelluntaa ilman edellytyksiä.

        " Abstraktien käsitteiden viittaamien olioiden reaalinen olemassaolo edellyttää sitä, että noilla olioilla on sisäinen rakenne, joka koostuu osista."

        Tuo on se reduktionismin uskontunnustus. Yhtä hyvin voisi ajetella että kokonaisuus erilaistuu osiksi kuten esim. solujen jakautumisessa biologisissa ilmiöissä jolloin sen alkusolun geneettinen informaatio siirtyy niihin jakautuneisiin soluihin. Tietoisuuden voisi myös ajatella olevan yksi kokonaisuus joka sitten jakautuu osiksi.

        "Auringon järjestäytynyt energia mahdollistaa elämän maapallolla ja vaikka auringon epäjärjestys tässä jonkin verran lisääntyy niin se lisääntyisi joka tapauksessa samalla tavalla vaikka maapallolla ei olisikaan..." "Nythän ei tiedetä kuinka todennäköistä ja yleistä biologinen elämä on maailmankaikkeudessa..."

        Eihän tässä ongelmana olekaan se, että havaitaan, että olot ovat elämälle täällä suotuisat, vaan sen selittäminen miten nuo suotuisat olosuhteet ylipäätään ovat mahdollistuneet. Voidaan tietysti ajatella, että jos olosuhteet eivät olisi suotuisat, niin emme havaitsisi niitä, mutta jos halutaan välttää sattuman käyttö selityksissä, niin joudutaan olettamaan erilaisia vaihtoehtoisia kehityskulkuja. Monimaailmamallin mukaan voidaan ajatella, että käytännössä lähes kaikki epäjärjestys sijaitsee niissä rinnakkaisissa maailmankaikkeuksissa, joissa asiat ovat ns. menneet pieleen, eli niissä ei ole muodostunut sellaisia havaitsevia olentoja, jotka olisivat säilyneet hengissä, etenkään niin kauan, että olisivat voineet keskustella esimerkiksi argumenteistaan sen puolesta ja vastaan, että havaitun järjestyksen selittämiseen tarvitaan rinnakkaisia maailmankaikkeuksia tms.

        Älyllisten olentojen muodostuminen sattumalta ensimmäisellä yrityksellä (eli ensimmäisessä satunnaisessa ja ainoassa maailmankaikkeudessa) olisi erittäin epätodennäköistä, vaikka tuon maailmankaikkeuden massa olisikin näkyvän maailmankaikkeuden suuruusluokaltaan noin 10^23 Auringon massaa, koska oleellista tässä ei ole hiukkasten lukumäärä, vaan hiukkasten kokoonpano ja vaihtoehtoisten kokoonpanojen lukumäärä on hiukkasten lukumäärän eksponenttifunktio. Vaikka aika monetkin vaihtoehtoiset kokoonpanot tuottaisivat jonkinlaisen elävän olennon, niin silti elinkelvottomien kokoonpanojen lukumäärä olisi hiukkasten lukumäärän eksponenttifunktio aivan yksinkertaistenkin olentojen osalta.

        "Itse luulen että tietoisuus on pohjimmiltaan enemmän non-lokaali hyperdimensioonaalinen ilmiö ja se subjektiivinen kokemuksellisuus taas sen nonlokaalin asian monimutkainen vuorovaikutussuhde lokaalin fysikaalisen maailman kanssa."

        Tieteelliseltä kannalta oleellisinta on se, mitä testattavia ja mittauksilla todennettavissa olevia ennusteita erilaiset todellisuuden selitysmallit kykenevät tarjoamaan.

        "Kyllä tietoisuus kykenee tuottamaan sisältöjä ilman havaintojakin varsin hyvin esim. unien näkemisessä ja hallusinaatioissa."

        Tuo on kyllä jonkinlaista ula-ajattelua. Sisällöt ovat fysikaalisia asioita, joiden liikkeitä ohjaavat luonnonlait. Unien näkeminen ja hallusinaatiot eivät eroa fysikaalisten perusteidensa osalta havainnoista. Niiden osalta fysikaaliset osat vaan esiintyvät "vähän löyhemmässä muodostelmassa kuin tavallisesti", eli niiden osat ovat toisenlaisessa järjestyksessä kuin tavallisesti havaittaessa. Asia erikseen on se, että noita erilaisia järjestyksiä on todella todella valtavia määriä.

        "Tuo on se reduktionismin uskontunnustus. Yhtä hyvin voisi ajetella että kokonaisuus erilaistuu osiksi kuten esim. solujen jakautumisessa biologisissa ilmiöissä jolloin sen alkusolun geneettinen informaatio siirtyy niihin jakautuneisiin soluihin. Tietoisuuden voisi myös ajatella olevan yksi kokonaisuus joka sitten jakautuu osiksi."

        En miellä reduktionismia tunnustukselliseksi vaan pragmaattiseksi valinnaksi, koska tieteellisen selittämisen kannalta on hyödyllisempää tarkastella asioita riittävän tarkasti kuin epämääräisinä klöntteinä. Insinööreillä on tapana kiinnittää huomionsa käytännössä hyödynnettäviin asioihin. Esimerkiksi solut ovat konkreettisia asioita, jotka koostuvat osista ja myös informaatio koostuu osista. Onkin siis tietysti toivottavaa, että myös tietoisuuden osat havaitaan ja niitä mittaillaan.


      • FirstMatter
        metafysiikkaa kirjoitti:

        "Kaikki matematiikka edellyttää fysikaalisia rakenteita, joihin tuo matematiikka on koodattu."

        Tuskin. Kyllä matemaattinen mielikuvitus on paljon laajempi kuin fysikaaliset rakenteet.
        (esim. Mandelbrotin joukko). Mikä tahansa looginen konstruktio toimii jos se on sisäisesti koherentti. Ei luonnossa ole autoja, lentokoneita eikä tietokoneitakaan vaan ne ovat inhimillisen luomiskyvyn tuotteita.

        "Jos tuota kieltä ei olisi koodattu fysikaalisiin rakenteisiin, niin sillä ei olisi mitään kuvaavaa tai ohjaavaa vaikutusta mihinkään."

        Niin jos kiellät tietoisuuden olemassaolon ja vaikutuksen niin tuollainen johtopäätös syntyy helposti.

        "Tapahtuuko tietoisuuden laadullinen kehittyminen mielestäsi asteittain vai hyppäyksittäin? "

        Sekä että paitsi että se mitä kutsumme tietoisuudeksi on lähinnä monimutkainen takaisinkytkentäjärjestelmä joka ei sinänsä edellytä jatkuvaa tietoisuutta. Inhimillinen toiminta on hyvin suurelta osin automaattista.

        "Eli, voiko tietoisuuden kehittyminen liittyä jonkinlaiseen itseorganisoituvien rakenteiden faasimuutokseen? "

        Tuo voi liittyä ainakin elämän kehittymiseen. Tuossa voisi olla hyödyllistä ajatella jotenkin kaksoisaspektimallin kautta siten että käyttäytyminen (aine, hiukkasten liike) on olevaisen objektiivinen aspekti ja tietoisuus saman asian subjektiivinen aspekti ainakin jos on kyse elävistä tietoisista olennoista. Tietoisuus sinänsä jää siten aina tieteellisen metodin ulottumattomiin ja ainoastaan voimme havaita tietoisuuden vaikutukset vain epäsuorasti kun kyse ei ole omasta tietoisuudestamme.

        "Miten tuo suhtautuu rinnakkaisten maailmankaikkeuksien vuorovaikutuksiin?"

        En tiedä kun en edes usko mm-malliin jossa on sotkettu potentiaalisuus aktuaaliseen. Tulevaisuus on aina potentiaalista ja vastaavasti menneisyys on aina aktuaalista ja nykyhetkessä (joka voi olla eri ulotteinen eri oloilla) tapahtuu valinta tai valikoituu mekaanisesti yksi vaihtoehto aktuaaliseksi.

        " mutta kaiken järjen mukaan realismia kuitenkin on se, että kvanttien superpositiot hajoavat äärimmäisen vähäistenkin häiriöiden seurauksena."

        On eri asia pystyä mittaamaan ne superpositiot kuin että ne ovat olemassa. Determinismi on tuttua ja turvallista ja mm-tulkinta tarjoaa mahdollisuuden takertua siihen vanhaan malliin.

        ...

        Vahva kiihtyvyyskään ei viimeisimpien tutkimusten mukaan sotke lomittumisilmiötä:

        http://www.sciencealert.com/scientists-have-set-a-limit-for-quantum-entanglement-and-it-s-really-freaking-powerful

        kvanttikommunikaatio ilman hiukkasten siirtoa:

        http://www.pnas.org/content/114/19/4920.abstract

        "Tuskin. Kyllä matemaattinen mielikuvitus on paljon laajempi kuin fysikaaliset rakenteet.
        (esim. Mandelbrotin joukko)."

        Fysiikka asettaa rajat myös matematiikalle ja mielikuvitukselle. Esimerkiksi vain sellainen matematiikka voidaan havaita, jonka havaitsemiseen on käytettävissä riittävästi alkeishiukkasia, jotka ovat sopivissa suhteissa toisiinsa. Havaitsemattomasta matematiikasta puhuminen taas on puhdasta metafysiikkaa. Mandelbrotin joukon osien rakenne on mahdollista laskea ilman, että siihen tarvittaisiin lainkaan tietoisuutta, mutta tietenkin se edellyttää erittäin suurta paikallista järjestystä.

        "Mikä tahansa looginen konstruktio toimii jos se on sisäisesti koherentti. Ei luonnossa ole autoja, lentokoneita eikä tietokoneitakaan vaan ne ovat inhimillisen luomiskyvyn tuotteita.

        Kyllä ihmiset, autot, lentokoneet ja tietokoneet ovat siinä merkityksessä osa luontoa, että ne toimivat samojen luonnonlakien mukaisesti kuin muutkin asiat. Ihminen on vain luonnonvoimien agentti, joka kasaa nuo vempaimet. Luonnonvoimat siis vetävät naruista ja ihminen sätkii niiden tahdissa :)

        ""Jos tuota kieltä ei olisi koodattu fysikaalisiin rakenteisiin, niin sillä ei olisi mitään kuvaavaa tai ohjaavaa vaikutusta mihinkään.""
        "Niin jos kiellät tietoisuuden olemassaolon ja vaikutuksen niin tuollainen johtopäätös syntyy helposti."

        Kieli ilman fysiikkaa on Liisa ihmemaassa -juttuja, eli ihmisen mielikuvituksen tuotosta tietoisuudesta riippumatta.

        "Sekä että paitsi että se mitä kutsumme tietoisuudeksi on lähinnä monimutkainen takaisinkytkentäjärjestelmä joka ei sinänsä edellytä jatkuvaa tietoisuutta. Inhimillinen toiminta on hyvin suurelta osin automaattista."

        Kuulostaa järkevältä.

        "Tietoisuus sinänsä jää siten aina tieteellisen metodin ulottumattomiin ja ainoastaan voimme havaita tietoisuuden vaikutukset vain epäsuorasti kun kyse ei ole omasta tietoisuudestamme."

        Miten oletetut tietoisuuden vaikutukset voidaan erottaa aineen vaikutuksista, jos siis oletetaan, että noilla kahdella on eroa? Jos niitä ei voida erottaa, niin kyse on kaiketi joko metafysiikasta tai hiustenhalkomisesta.

        "En tiedä kun en edes usko mm-malliin jossa on sotkettu potentiaalisuus aktuaaliseen. Tulevaisuus on aina potentiaalista ja vastaavasti menneisyys on aina aktuaalista ja nykyhetkessä (joka voi olla eri ulotteinen eri oloilla) tapahtuu valinta tai valikoituu mekaanisesti yksi vaihtoehto aktuaaliseksi."

        Kuulostaa klassiselta ajattelulta, mutta todellisuushan on kvanttimekaaninen ja esimerkiksi kvanttien interferenssissä kaikki tosiaan tapahtuu samanaikaisesti. Eli, esimerkiksi kvanttitietokoneissa varsinainen kvanttilaskenta ei siis vie *lainkaan* aikaa. Tuosta huolimatta kyse on nimenomaan aktuaalisesta, eikä potentiaalisesta laskennasta, koska sillä voidaan ratkaista todellisia ongelmia ja vieläpä sellaisiakin ongelmia, joiden ratkaisu olisi muuten käytännössä mahdotontakin.

        "On eri asia pystyä mittaamaan ne superpositiot kuin että ne ovat olemassa."

        Laskennan kannalta niiden olemassaolo ei ole relevanttia, jos niitä ei pystytä mittaamaan, mutta oleellista tässä oli se, että niitä ei takuulla pystytä edes mittaamaan, jos ne ovat hajonneet mistä tahansa muistakaan syistä.


      • FirstMatter
        metafysiikkaa kirjoitti:

        Kiitos vastauksesta. Olen jonkun verran aikoinaan ohjelmoinut assemblerilla ja tuo kvanttilaskennan tulos kuullostaa varsin monimutkaiselta perinteisiin tietokoneisiin verrattuna.

        Max Tegmark taisi esittää tuon väitteen että kvanttitietokone todistaa monimaailmamallin oikeaksi. Eikö se kvanttilaskennan tulos ole myös tulkittavissa aaltofunktion romahdukseksi yhtä hyvin? Sehän on kai selvää että tämä maailmankaikkeus ei missään tapauksessa ole klassisen fysiikan mukainen. Monimaailmatulkinnassa käsittääkseni vain oletetaan lukematon määrä deterministisesti toimivia klassisen fysiikan maailmankaikkeuksia eli sama asia hahmotetaan hieman eri tavalla.

        Myös sellainen monimaailma voisi olla mahdollinen jossa tietoisuus virittäytyy ikään kuin eri aallonpituuksille radiovirittimen tapaan ja kokee erilaisia maailmoja (kuten skitsofreniassa, unissa ja hallusinaatioissa) jos lähdetään tarkastelemaan asiaa tietoisuuden eikä perinteisen fysiikkatieteen kannalta. Tietoisuuden kannalta fysiikan tutkima todellisuus on vain tässä kulttuurissa elävien ihmisten konsensustodellisuutta joka koetaan samalla tavalla fysikaalisena kuin unessa koetaan unimaailma todellisena.

        ....

        Romahdusmalleissa ymmärtääkseni ei postuloida lisää maailmankaikkeuksia vaan annetaan mahdollisuuksille eli potentiaalisille tapahtumille ontologinen olemassaolo ja merkitys.

        Tietoisuuden kysymyksen kannalta tällä on suuri merkitys koska se mahdollistaa aidon valinnan eri vaihtoehtojen välillä eli jonkinlaisen vapaan tahdon mitä klassinen deterministinen maailmankaikkeus eikä sen monistus monimaailmoiksi mahdollista. Tietoisuus ei varsinaisesti vaikuta laisinkaan ns. fysikaalisessa maailmassa vaan nimenomaan ns. potentiaalisen maailman asukki eli ts. tietoisuuden vaikutus ilmenee vain ja ainoastaan ns. aaltofunktion romahduksen kautta.

        Koska romahdusmalli ottaa huomioon ajan suunnan niin aito vapaa tahto edellyttää moniulotteisen ajan. Yksiulotteinen (jana) ajassa tapahtumat ovat deterministisiä ja täysin mekaanisia ja epäjärjestys voi ainoastaan lisääntyä.

        2-ulotteisessa ajassa (taso) on olemassa useita vaihtoehtoja mutta siinäkin niiden vaihtoehtojen kokonaismäärä vähenee ajan kuluessa paitsi tilanteessa missä kaikki energia on potentiaalienergiaa eikä mitään tapahdu.

        3-ulotteinen aika mahdollistaa potentiaalisuuksien uusiutumisen syklisesti eli dekoherenssia seuraa aina rekoherenssi ja se mahdollistaa olioiden identiteetin säilymisen ja uusiutumisen.

        Ilman jonkinlaista vapaata tahtoa tietoisuus on pelkkä haamu ja korkeintaan pelkästään ulkopuolinen tarkkailija ilman minkäänlaista vaikutusmahdollisuutta tapahtumien kulkuun.

        Tämä siis tietenkin edellyttää että tietoisuus ei ole sama asia kuin fysikaalisten säännönmukaisuuksien determinoima aivo - tai hermostokudos vaikka yksilön kokema tietoisuus varmaan suuressa määrin onkin aivojen ja epälokaalin tietoisuuden yhteisvaikutusta ja takaisinkytkentää fysikaalisen ympäristön kanssa.

        Ketjun aihe olisi tietysti huomattavasti helpompi käsitellä pelkästään momimaailmamallin vs. romahdustulkintojen suhteen. Tietoisuus on tavallaan aina jonkinlainen ulkopuolinen häirikkö tai Pandoran lipas puhtaasti luonnontieteelliseltä kannalta tarkasteltuna.

        Sosologiaa ja ihmisjoukkojen käyttäytymistä varmasti voisi ja voikin mallintaa fysiikkatieteen tarkkuudella ellei aika ajoin ilmeistyisi poikkeuksellisia ihmisyksilöitä jotka romahduttavat koko vallitsevan systeemin. Historian aikana näitä on ollut useita (esim. Tsingis kaani, Napoleon, Hitler jne). Isaac Asimov kuvasi tällaista teossarjassaan Säätiö aika mielenkiintoisella tavalla.

        Tuskin edes tulevaisuuden kvanttitietokoneet pystyvät ennustamaan tulevaisuutta aukottoman tarkasti kun aina jostain voi kehkeytyä kaoottinen prosessi joka pakottaa järjestelmän uusiutumaan. Kvanttitietokoneiden uusi uljas maailma tuskin on kovin miellyttävä paikkaa elää ihmisyksilön kannalta kun tämä perinteisen tietotekniikan kyllästämä kulttuurikin ilmeisesti heikentää ihmisten kognitiivisia kykyjä jka muokkaa ajattelua binääriseen ajatteluun jossa ajatellaan äärimmäisten vastakohtien kautta eikä ymmärretä että maailma on oikeasti analoginen eli vastkohtien välillä on paljon variaatiota.

        Heh. Tulipa pitkä jaaritus mutta aihe on monin tavoin mielenkiintoinen ja innostaa spekuloimaan melko villistikin....

        "Kiitos vastauksesta. Olen jonkun verran aikoinaan ohjelmoinut assemblerilla ja tuo kvanttilaskennan tulos kuullostaa varsin monimutkaiselta perinteisiin tietokoneisiin verrattuna."

        Kvanttilaskennan toimintaperiaatteiden ymmärtäminen on suhteellisen suoraviivaista matemaattisesti orientoituneille. Hadamardin muunnokset tarkoittavat kiertoja kvanttiominaisuuden amplitudin kuvaaville vektoreille vinon akselin ympäri kompleksiavaruudessa. Kvanttioperaatiot ovat välittömiä ja reversiibeleitä, eli ne eivät vie lainkaan aikaa, eivätkä ilmeisesti lisää entropiaa. Ilmainen lounas on siis mahdollinen, mutta se edellyttää erittäin vaativaa ja tarkkaa säätöä.

        Varsinaista vastinetta klassiselle ohjelmalle ei ainakaan vielä kehityksen nykyvaiheessa ole, koska varsinainen kvanttilaskenta suoritetaan suoraan laitteistokomponenteilla varsin suoraviivaisesti. Piirien ominaisuudet kuvataan unitaarisilla matriiseilla ja niiden väliset liittymät graafisilla kaavioilla, joissa tyypillisesti esiintyy kvanttimuistien välillä kulkevien signaalien eteneminen sellaisten komponenttien kautta, jotka siis suorittavat Hadamardin muunnoksia tai määritellyn suuruisia ehdollisia vaihesiirtoja. Ensimmäisessä kvanttimuistissa voivat esimerkiksi olla syöttötiedot ja toisessa kvanttimuistissa tulostiedot. Fourier-muunnokset kohdistuvat kahteen joukkoon kompleksiarvoisia reaalimuuttujia. Niiden avulla saadaan yleisessä tapauksessa ratkaistua osittaisdifferentiaaliyhtälöitä ja tekijöihinjaon tapauksessa selvitettyä tutkittavan funktion jaksollisuus.

        "Eikö se kvanttilaskennan tulos ole myös tulkittavissa aaltofunktion romahdukseksi yhtä hyvin?"

        Olisi hauska kuulla selitysyritys sille, miten tuo romahdus suorittaa kvanttilaskennan.

        "Myös sellainen monimaailma voisi olla mahdollinen jossa tietoisuus virittäytyy ikään kuin eri aallonpituuksille radiovirittimen tapaan ja kokee erilaisia maailmoja (kuten skitsofreniassa, unissa ja hallusinaatioissa) jos lähdetään tarkastelemaan asiaa tietoisuuden eikä perinteisen fysiikkatieteen kannalta."

        Okay, mutta miten tuota voitaisiin testata?

        "Romahdusmalleissa ymmärtääkseni ei postuloida lisää maailmankaikkeuksia vaan annetaan mahdollisuuksille eli potentiaalisille tapahtumille ontologinen olemassaolo ja merkitys."

        Heh-heh! Selitys vielä vaan jää puuttumaan, eli miten nuo potentiaaliset, mutta ilmeisesti kuitenkin ei-aktuaaliset tapahtumat suorittavat supertehokasta laskentaa?


      • FirstMatter
        metafysiikkaa kirjoitti:

        Kiitos vastauksesta. Olen jonkun verran aikoinaan ohjelmoinut assemblerilla ja tuo kvanttilaskennan tulos kuullostaa varsin monimutkaiselta perinteisiin tietokoneisiin verrattuna.

        Max Tegmark taisi esittää tuon väitteen että kvanttitietokone todistaa monimaailmamallin oikeaksi. Eikö se kvanttilaskennan tulos ole myös tulkittavissa aaltofunktion romahdukseksi yhtä hyvin? Sehän on kai selvää että tämä maailmankaikkeus ei missään tapauksessa ole klassisen fysiikan mukainen. Monimaailmatulkinnassa käsittääkseni vain oletetaan lukematon määrä deterministisesti toimivia klassisen fysiikan maailmankaikkeuksia eli sama asia hahmotetaan hieman eri tavalla.

        Myös sellainen monimaailma voisi olla mahdollinen jossa tietoisuus virittäytyy ikään kuin eri aallonpituuksille radiovirittimen tapaan ja kokee erilaisia maailmoja (kuten skitsofreniassa, unissa ja hallusinaatioissa) jos lähdetään tarkastelemaan asiaa tietoisuuden eikä perinteisen fysiikkatieteen kannalta. Tietoisuuden kannalta fysiikan tutkima todellisuus on vain tässä kulttuurissa elävien ihmisten konsensustodellisuutta joka koetaan samalla tavalla fysikaalisena kuin unessa koetaan unimaailma todellisena.

        ....

        Romahdusmalleissa ymmärtääkseni ei postuloida lisää maailmankaikkeuksia vaan annetaan mahdollisuuksille eli potentiaalisille tapahtumille ontologinen olemassaolo ja merkitys.

        Tietoisuuden kysymyksen kannalta tällä on suuri merkitys koska se mahdollistaa aidon valinnan eri vaihtoehtojen välillä eli jonkinlaisen vapaan tahdon mitä klassinen deterministinen maailmankaikkeus eikä sen monistus monimaailmoiksi mahdollista. Tietoisuus ei varsinaisesti vaikuta laisinkaan ns. fysikaalisessa maailmassa vaan nimenomaan ns. potentiaalisen maailman asukki eli ts. tietoisuuden vaikutus ilmenee vain ja ainoastaan ns. aaltofunktion romahduksen kautta.

        Koska romahdusmalli ottaa huomioon ajan suunnan niin aito vapaa tahto edellyttää moniulotteisen ajan. Yksiulotteinen (jana) ajassa tapahtumat ovat deterministisiä ja täysin mekaanisia ja epäjärjestys voi ainoastaan lisääntyä.

        2-ulotteisessa ajassa (taso) on olemassa useita vaihtoehtoja mutta siinäkin niiden vaihtoehtojen kokonaismäärä vähenee ajan kuluessa paitsi tilanteessa missä kaikki energia on potentiaalienergiaa eikä mitään tapahdu.

        3-ulotteinen aika mahdollistaa potentiaalisuuksien uusiutumisen syklisesti eli dekoherenssia seuraa aina rekoherenssi ja se mahdollistaa olioiden identiteetin säilymisen ja uusiutumisen.

        Ilman jonkinlaista vapaata tahtoa tietoisuus on pelkkä haamu ja korkeintaan pelkästään ulkopuolinen tarkkailija ilman minkäänlaista vaikutusmahdollisuutta tapahtumien kulkuun.

        Tämä siis tietenkin edellyttää että tietoisuus ei ole sama asia kuin fysikaalisten säännönmukaisuuksien determinoima aivo - tai hermostokudos vaikka yksilön kokema tietoisuus varmaan suuressa määrin onkin aivojen ja epälokaalin tietoisuuden yhteisvaikutusta ja takaisinkytkentää fysikaalisen ympäristön kanssa.

        Ketjun aihe olisi tietysti huomattavasti helpompi käsitellä pelkästään momimaailmamallin vs. romahdustulkintojen suhteen. Tietoisuus on tavallaan aina jonkinlainen ulkopuolinen häirikkö tai Pandoran lipas puhtaasti luonnontieteelliseltä kannalta tarkasteltuna.

        Sosologiaa ja ihmisjoukkojen käyttäytymistä varmasti voisi ja voikin mallintaa fysiikkatieteen tarkkuudella ellei aika ajoin ilmeistyisi poikkeuksellisia ihmisyksilöitä jotka romahduttavat koko vallitsevan systeemin. Historian aikana näitä on ollut useita (esim. Tsingis kaani, Napoleon, Hitler jne). Isaac Asimov kuvasi tällaista teossarjassaan Säätiö aika mielenkiintoisella tavalla.

        Tuskin edes tulevaisuuden kvanttitietokoneet pystyvät ennustamaan tulevaisuutta aukottoman tarkasti kun aina jostain voi kehkeytyä kaoottinen prosessi joka pakottaa järjestelmän uusiutumaan. Kvanttitietokoneiden uusi uljas maailma tuskin on kovin miellyttävä paikkaa elää ihmisyksilön kannalta kun tämä perinteisen tietotekniikan kyllästämä kulttuurikin ilmeisesti heikentää ihmisten kognitiivisia kykyjä jka muokkaa ajattelua binääriseen ajatteluun jossa ajatellaan äärimmäisten vastakohtien kautta eikä ymmärretä että maailma on oikeasti analoginen eli vastkohtien välillä on paljon variaatiota.

        Heh. Tulipa pitkä jaaritus mutta aihe on monin tavoin mielenkiintoinen ja innostaa spekuloimaan melko villistikin....

        "Tietoisuuden kysymyksen kannalta tällä on suuri merkitys koska se mahdollistaa aidon valinnan eri vaihtoehtojen välillä eli jonkinlaisen vapaan tahdon mitä klassinen deterministinen maailmankaikkeus eikä sen monistus monimaailmoiksi mahdollista."

        Vapaa tahto on diiba-daabaa, jota ei pitäisi ollenkaan sotkea tieteelliseen keskusteluun. Kyseessä ei ole edes tieteellinen käsite.

        "Ilman jonkinlaista vapaata tahtoa tietoisuus on pelkkä haamu ja korkeintaan pelkästään ulkopuolinen tarkkailija ilman minkäänlaista vaikutusmahdollisuutta tapahtumien kulkuun."

        Tuo on oikea kuvaus, eli tietoisuus on tavallaan taaksepäin katsova haamu, joka koostuu alkeishavainnoista ilman, että niillä on mahdollisuutta vaikuttaa mitenkään asioiden etenemiseen, sillä tapahtunuthan on jo tapahtunut myös kvanttimekaniikan puolella. Tietoisuus on myös edellytystensä funktio, kuten muutkin asiat. Eli, esimerkiksi jos on väsynyt, niin tietoisuus yksinkertaisesti häviää.

        Todennäköisesti tietoisuus on vain sivutuote siitä, että asiat ovat siinä järjestyksessä, joka mahdollistaa sen. Eli, tietoisuus edellyttää tietyn järjestyksen, josta voi syntyä illuusio siitä, että tietoisuus olisi jonkinlainen kausaalinen syy tuolle järjestykselle, vaikka todellisuudessa syy-seuraus-suhde menee juuri toisin päin. Vastaavasti esimerkiksi myös vapauden tunne on illuusio, joka syntyy siitä, että havaitsee asioiden etenevän tavalla, joka vastaa toiveita, vaikka asiat oikeasti etenevätkin luonnonlakien mukaisesti, kuten aina muulloinkin.

        "Ketjun aihe olisi tietysti huomattavasti helpompi käsitellä pelkästään momimaailmamallin vs. romahdustulkintojen suhteen."

        Keskustelu on päässyt hiukan rönsyilemään.

        "Tuskin edes tulevaisuuden kvanttitietokoneet pystyvät ennustamaan tulevaisuutta aukottoman tarkasti kun aina jostain voi kehkeytyä kaoottinen prosessi joka pakottaa järjestelmän uusiutumaan."

        Determinismi ja ennustettavuus ovat eri asioita. Vaikka todellisuus mahdollisesti pohjimmiltaan onkin täysin deterministinen, niin sen tarkka ennustaminen on silti hyvin hyvin vaikeaa tai mahdotonta. Lisäksi tietoisuuden rajallisuus mahdollistaa vapauden illuusion, joten subjektiiviselta kannalta todellisen vapauden puute ja tietoisuuden rajallisuus voivat vaikuttaa monista jopa toivottaviltakin asioilta.

        "Kvanttitietokoneiden uusi uljas maailma tuskin on kovin miellyttävä paikkaa elää ihmisyksilön kannalta..."

        Täytyy muistaa korostaa, että maailman ongelmat eivät ole tekniikan vaan sellaisten ihmisten aiheuttamia, jotka käyttävät tekniikkaa ja tietysti myös muita resursseja kyseenalaisiin tarkoituksiin. Mitä pintapuolistumiseen tulee, niin olettaisin, että kvanttitekniikan esiinmarssi tulee vähitellen lisäämään oleellisesti kysyntää matemaattisesti hyvin korkeasti koulutetuille henkilöille. Toisaalta laskennan edistyminen mahdollistaa myös sen, että ns. suuren yleisön tarvitsee vaivata entistäkin vähemmän päätänsä matematiikalla ja ongelmanratkaisulla. Ehkä tuota voidaan jopa pitää win-win -tilanteena.


      • metafysiikkaa
        FirstMatter kirjoitti:

        "Jostain syystä fysiikassa se tyhjästä nyhjääminen on joka tapauksessa käytössä Big bangin tapauksessa."

        Maailmankaikkeuden ollessa hyvin pieni, sen osien sijainteihin liittyvät epämääräisyydet olivat suurempia kuin sen koko, eli vaikuttaisi siltä, että maailmankaikkeus ei välttämättä olisi koskaan ollut äärettömän pieni, vaan vain suhteellisen pieni.

        "Alkeishiukkasia ei ole kukaan koskaan suoraan havainnut joten on aika kyseenalaista puhua "havaituista hiukkastyypeistä" kun koko fysiikan standardimalli on matemaattinen konstruktio eli fysiikankin pohjalla on metafyysinen ja abstrakti mentaalinen konstruktio ja muutenkaan mitään puhtaasti empiiristä havaintoa ei ole olemassakaan vaan kaikki ns. aistihavainnot ovat aina vahvasti prosessoituja ja niiden tulkinta aina riippuvaista valitusta käsitejärjestelmästä ja kielestä."

        Kovin on pitkä lause, kielellisesti hyvin muotoiltu, monenlaista asiaa, mutta oleellista tässä on se, että tyhjästä syntyvistä vaikutuksista on kaikesta huolimatta vähemmän näyttöä kuin havaituista hiukkastyypeistä. Tässä tarkoitetut havainnot ovat luonteeltaan sellaisia, että samat mittaustulokset havaitaan mittaajasta ja monista muista tekijöistä riippumatta. On siis ilmeistä, että taustalla on tekijöitä, jotka ovat luonteeltaan objektiivisia, koska todennäköisyys sille, että samoja mittaustuloksia saataisiin muuten, olisi hyvin pieni.

        Onkin täysin ilmeistä, että on olemassa jotakin, joka vaikuttaa mittaajien tietoisuuteen tavalla, joka mahdollistaa tuon. Jos tuota vaikutusta yritetään selittää pelkällä tietoisuudella, niin päädytään kehäpäätelmään joka ei selitä mitään, eli kyseessä ei silloin ole tieteellinen selittäminen. Puhtaita havaintoja sen sijaan on olemassa, koska ne ovat edellytys sille, että niitä voidaan alkaa tulkita. Tämä on loogisesti ilmeistä, vaikka havaintojen selitykset kielen kautta sisältävätkin tietysti aina tulkintaa. Kyllä ihminen siis pystyy havaitsemaan ilmankin, että alkaa tulkita tietoisesti havaitsemaansa.

        "Fysiikan tulokset ovat tuottaneet toimivaa tekniikkaa nimenomaan juuri sen takia että tutkimuskohde on niin säännönmukainen ja mekaaninen eli suhteellisen helposti matemaattisesti mallinnettavissa verrattuna esim. eliöiden ja ihmisten toimintaan."

        Kyllä. Kyse on tarkasteltavan järjestelmän osien lukumäärästä. Osien lukumäärän kasvaessa lineaarisesti, niiden välisten mahdollisten vuorovaikutusten määrä kasvaa eksponentiaalisesti. Tuossa ei kuitenkaan ole mitään mystistä, vaan kyse on vain kompleksisuudesta. Ihmisenkin osat noudattavat luonnonlakeja.

        "Arkikielessä tietoisuuden tuottamia fysikaalisia kokonaisuuksia kutsutaan "keinotekoisiksi" kun taas ns. "luonnonlakien" tuottamia vastaavia kokonaisuuksia "luonnollisiksi" vaikka periaatteessa kai molemmat pitäisi tulkita luonnollisiksi."

        Kyllä.

        "Jossain mielessä siis vaistomaisesti koetaan tietoisuuden vaikutus ilmiöihin jollain tavalla luonnolle vieraana asiana."

        Kyse on siitä, että ihminen on etääntynyt lajina suorasta havaitsemisesta ja luonut käsitteellisen kahtiajaon itsensä ja luonnon välille. Ilmeisesti itsetietoisuuden kehittyminen erityisesti siitä kommunikoinnin asteelle edellyttää tuollaisen kahtiajaon. Eli, vaikka ihmisen ja luonnon välillä ei olisikaan mitään oleellista eroa, niin olisi epätodennäköisempää, että keskustelun kohteena olisi tuo identtisyys kuin sen puute.

        "Maailmankaikkeuden ollessa hyvin pieni, sen osien sijainteihin liittyvät epämääräisyydet olivat suurempia kuin sen koko, eli vaikuttaisi siltä, että maailmankaikkeus ei välttämättä olisi koskaan ollut äärettömän pieni, vaan vain suhteellisen pieni."

        Tuota olen itsekin miettinyt. Noissa big bangin ja mustien aukkojen singulariteeteissa ei tunnu olevan kovin paljon mieltä elleivät ne sitten ole jonkinlaisia rajapintoja niihin muihin maailmankaikkeuksiin tai sinne kvanttityhjiöön josta kai nykyisen bb-teorian mukaan oletetaan maailmankaikkeus syntyneen jonkinlaisen "kvanttifluktuaation" seurauksena mikä taas viittaa siihen että tämä kaikkeus ei ole olekaan täysin suljettu systeemi.

        "Tässä tarkoitetut havainnot ovat luonteeltaan sellaisia, että samat mittaustulokset havaitaan mittaajasta ja monista muista tekijöistä riippumatta. "

        Niin tieteen perustahan on jonkinlaiseen invarianssiin pyrkiminen eli havaintojen tulisi olla havaitsija - ja tutkijariippumattomia eli koejärjestelyiden pitäisi aina olla toistettavissa olevia. Toinen asia on sitten onnistutaanko siinä oikeasti ja onko sellainen edes mahdollista käytännössä. Kvanttifysiikan romahdusmallihan (Bohrin köpistulkinta) oikeastaan syntyi sen takia että havainto sotkee aina tutkittavaa kohdetta ja mitä pienempi se tutkittava kohde on niin sitä merkittävämmästä häiriöstä on kyse.

        "On siis ilmeistä, että taustalla on tekijöitä, jotka ovat luonteeltaan objektiivisia, koska todennäköisyys sille, että samoja mittaustuloksia saataisiin muuten, olisi hyvin pieni. "

        Joo kyllä voidaan varmaan puhua ainakin suhteellisesta objektiivisuudesta eli invarianssista havaintojen välillä mutta toisaalta kyseessä voi olla erikoistapaus eli havaitsijan ajasta ja paikasta riippuvainen lokaali säännönmukaisuus joka ei välttämättä ole extrapoloitavissa eli laajennettavissa koko maailmankaikkeutta koskevaksi yleiseksi luonnonlaiksi.

        "Puhtaita havaintoja sen sijaan on olemassa, koska ne ovat edellytys sille, että niitä voidaan alkaa tulkita. "

        Jokin aika sitten luin artikkelin palkinnon saaneesta suomalaisesta silmän verkkokalvon tutkijasta (liittyi muistaakseni aaltoyliopistoon) joka esitti että silmän verkkokalvo suorittaa miljoonia laskutoimituksia (kvanttilaskentaako?) ennen kuin näköaistimus siirtyy aivoihin ja tietoisesti tajuttavaksi.

        Kognitiivinen tutkija Donald Hoffman kysyy ted-luennossaan että koemmeko oikeasti maailman sellaisena kuin se on ja esittää hyvät perustelut sille että mielemme konstruoi maailman. Mieli voi sitten tietysti tässä tarkoittaa aistinelinten ja aivojen prosessointia joten melko epämääräiseen 'tietoisuus' käsitteeseen ei tarvitse viitata.

        https://www.ted.com/talks/donald_hoffman_do_we_see_reality_as_it_is

        "Osien lukumäärän kasvaessa lineaarisesti, niiden välisten mahdollisten vuorovaikutusten määrä kasvaa eksponentiaalisesti."

        Tuo on totta ja sen takia olenkin sitä mieltä että vaikka ns. "tietoisuus" olisikin jonkinlainen kosminen perusvaikuttaja niin jotta sillä tietoisuudella olisi mitään sisältöä niin sen tietoisuuden pitäisi jakautua osiin voidakseen vuorovaikuttaa itsensä kanssa.

        "Ihmisenkin osat noudattavat luonnonlakeja."

        Sheldraken mallista aikoinaan jossain määrin inspiroituneena kutsun noita "luonnon tavoiksi" eli vähitellen syntyneiksi säännönmukaisuuksiksi jossa sen säännönmukaisuuden aiheuttaa maailmankaikkeuden osien kulloinenkin keskinäinen konfiguraatio eli en usko luonnonlakien olemassaoloon ilman niitä asioita joiden lakeja ne ovat.

        Luonnonlaki on siis minun mielestäni emergentti asia joka syntyy siitä että hiukkasia tai jonkun tietyn tasoisen systeemin jäseniä on tilastollisesti suuri määrä. Ihmiskunnan hiukkanen on ihmisyksilö jonka käyttäytyminen on suhteellisen vapaata koska kyseessä on monimutkainen avoin systeemi joka eroaa ympäristöstään omien sisäisten säännönmukaisuuksiensa suhteen ainakin suhteellisessa määrin eli rajatussa systeemissä sen systeemin ulkopuolella epätodennäköisiksi määritellyt asiat voivat käyttäytyä todennäköisesti sen systeemin sisäpuolella koska entropian taso on siinä systeemissä matalampi kuin ympäristössä.

        Eliöissä tämän ympäristöä suhteellisen pysyvästi matalamman tason mahdollistanee genomin informaatio ja sen monistuminen kehon jokaiseen soluun. Matalamman tason ylläpitäminen edellyttää tietysti jatkuvaa aineenvaihduntaa ja saatavissa olevan korkeatasoisen informaation saatavuutta ravinnon, hengityksen ja aistivaikutteiden kautta.

        "Kyse on siitä, että ihminen on etääntynyt lajina suorasta havaitsemisesta ja luonut käsitteellisen kahtiajaon itsensä ja luonnon välille."

        Niin se "suora havaitseminen" on varmaan sama asia kuin perustason fysikaalinen vuorovaikutus jossa ns. "eloton luonto" käyttäytyy "luonnollisesti" kun taas ns. "elollisessa luonnossa" havaitseminen prosessoidaan aina enemmän tai vähemmän monimutkaisesti ehkä juuri nimenomaan kvanttilaskennan keinoin.


      • metafysiikkaa
        metafysiikkaa kirjoitti:

        "Maailmankaikkeuden ollessa hyvin pieni, sen osien sijainteihin liittyvät epämääräisyydet olivat suurempia kuin sen koko, eli vaikuttaisi siltä, että maailmankaikkeus ei välttämättä olisi koskaan ollut äärettömän pieni, vaan vain suhteellisen pieni."

        Tuota olen itsekin miettinyt. Noissa big bangin ja mustien aukkojen singulariteeteissa ei tunnu olevan kovin paljon mieltä elleivät ne sitten ole jonkinlaisia rajapintoja niihin muihin maailmankaikkeuksiin tai sinne kvanttityhjiöön josta kai nykyisen bb-teorian mukaan oletetaan maailmankaikkeus syntyneen jonkinlaisen "kvanttifluktuaation" seurauksena mikä taas viittaa siihen että tämä kaikkeus ei ole olekaan täysin suljettu systeemi.

        "Tässä tarkoitetut havainnot ovat luonteeltaan sellaisia, että samat mittaustulokset havaitaan mittaajasta ja monista muista tekijöistä riippumatta. "

        Niin tieteen perustahan on jonkinlaiseen invarianssiin pyrkiminen eli havaintojen tulisi olla havaitsija - ja tutkijariippumattomia eli koejärjestelyiden pitäisi aina olla toistettavissa olevia. Toinen asia on sitten onnistutaanko siinä oikeasti ja onko sellainen edes mahdollista käytännössä. Kvanttifysiikan romahdusmallihan (Bohrin köpistulkinta) oikeastaan syntyi sen takia että havainto sotkee aina tutkittavaa kohdetta ja mitä pienempi se tutkittava kohde on niin sitä merkittävämmästä häiriöstä on kyse.

        "On siis ilmeistä, että taustalla on tekijöitä, jotka ovat luonteeltaan objektiivisia, koska todennäköisyys sille, että samoja mittaustuloksia saataisiin muuten, olisi hyvin pieni. "

        Joo kyllä voidaan varmaan puhua ainakin suhteellisesta objektiivisuudesta eli invarianssista havaintojen välillä mutta toisaalta kyseessä voi olla erikoistapaus eli havaitsijan ajasta ja paikasta riippuvainen lokaali säännönmukaisuus joka ei välttämättä ole extrapoloitavissa eli laajennettavissa koko maailmankaikkeutta koskevaksi yleiseksi luonnonlaiksi.

        "Puhtaita havaintoja sen sijaan on olemassa, koska ne ovat edellytys sille, että niitä voidaan alkaa tulkita. "

        Jokin aika sitten luin artikkelin palkinnon saaneesta suomalaisesta silmän verkkokalvon tutkijasta (liittyi muistaakseni aaltoyliopistoon) joka esitti että silmän verkkokalvo suorittaa miljoonia laskutoimituksia (kvanttilaskentaako?) ennen kuin näköaistimus siirtyy aivoihin ja tietoisesti tajuttavaksi.

        Kognitiivinen tutkija Donald Hoffman kysyy ted-luennossaan että koemmeko oikeasti maailman sellaisena kuin se on ja esittää hyvät perustelut sille että mielemme konstruoi maailman. Mieli voi sitten tietysti tässä tarkoittaa aistinelinten ja aivojen prosessointia joten melko epämääräiseen 'tietoisuus' käsitteeseen ei tarvitse viitata.

        https://www.ted.com/talks/donald_hoffman_do_we_see_reality_as_it_is

        "Osien lukumäärän kasvaessa lineaarisesti, niiden välisten mahdollisten vuorovaikutusten määrä kasvaa eksponentiaalisesti."

        Tuo on totta ja sen takia olenkin sitä mieltä että vaikka ns. "tietoisuus" olisikin jonkinlainen kosminen perusvaikuttaja niin jotta sillä tietoisuudella olisi mitään sisältöä niin sen tietoisuuden pitäisi jakautua osiin voidakseen vuorovaikuttaa itsensä kanssa.

        "Ihmisenkin osat noudattavat luonnonlakeja."

        Sheldraken mallista aikoinaan jossain määrin inspiroituneena kutsun noita "luonnon tavoiksi" eli vähitellen syntyneiksi säännönmukaisuuksiksi jossa sen säännönmukaisuuden aiheuttaa maailmankaikkeuden osien kulloinenkin keskinäinen konfiguraatio eli en usko luonnonlakien olemassaoloon ilman niitä asioita joiden lakeja ne ovat.

        Luonnonlaki on siis minun mielestäni emergentti asia joka syntyy siitä että hiukkasia tai jonkun tietyn tasoisen systeemin jäseniä on tilastollisesti suuri määrä. Ihmiskunnan hiukkanen on ihmisyksilö jonka käyttäytyminen on suhteellisen vapaata koska kyseessä on monimutkainen avoin systeemi joka eroaa ympäristöstään omien sisäisten säännönmukaisuuksiensa suhteen ainakin suhteellisessa määrin eli rajatussa systeemissä sen systeemin ulkopuolella epätodennäköisiksi määritellyt asiat voivat käyttäytyä todennäköisesti sen systeemin sisäpuolella koska entropian taso on siinä systeemissä matalampi kuin ympäristössä.

        Eliöissä tämän ympäristöä suhteellisen pysyvästi matalamman tason mahdollistanee genomin informaatio ja sen monistuminen kehon jokaiseen soluun. Matalamman tason ylläpitäminen edellyttää tietysti jatkuvaa aineenvaihduntaa ja saatavissa olevan korkeatasoisen informaation saatavuutta ravinnon, hengityksen ja aistivaikutteiden kautta.

        "Kyse on siitä, että ihminen on etääntynyt lajina suorasta havaitsemisesta ja luonut käsitteellisen kahtiajaon itsensä ja luonnon välille."

        Niin se "suora havaitseminen" on varmaan sama asia kuin perustason fysikaalinen vuorovaikutus jossa ns. "eloton luonto" käyttäytyy "luonnollisesti" kun taas ns. "elollisessa luonnossa" havaitseminen prosessoidaan aina enemmän tai vähemmän monimutkaisesti ehkä juuri nimenomaan kvanttilaskennan keinoin.

        Yksisoluisillakin olioilla on havaittu mielekästä tietoiseksi toiminnaksi luonnehdittavissa olevaa käyttäytymistä (Paramecium)

        http://www.microscopy-uk.org.uk/index.html?http://www.microscopy-uk.org.uk/mag/artnov14/paramecium-engima.html


        joten elämän ilmiöiden perustasolla esiintynee vahvaa prosessointia ja joka sitten hermoston ja varsinkin aivojen kehityksen kautta muuntuu itsetietoisuudeksi eli varsinaiseksi tietoisuudeksi.

        ....

        "Tilanne, jossa selitysmalli sisältää satunnaismuuttujia ei voi olla tavoite, vaan vain välivaihe ja tavoitteena tulee olla korvata se sellaisella tarkemmalla mallilla, jonka antamat ennusteet ovat testattavissa."

        Juu tuohon tavoitteeseen pitää pyrkiä tieteessä ja jos se ei onnistu jolloin tietyllä tavalla niin mahdollisesti on syytä vähän muuttaa näkökulmaa joko suppeammaksi tai laajemmaksi.

        "On tietysti mahdollista, että tuossa ei onnistuta, mutta se olisi tieteen kannalta valitettavaa, koska etenkin pysyvillä aukkokohdilla on taipumus täyttyä kaikenkirjavalla epätieteellisellä sisällöllä."

        Tieteellisellä metodilla on varmasti joku rajoitus jota on hyvin vaikeaa tai mahdotonta ylittää. Toisaalta se "epätieteellinenkin" ymmärrys voi olla jossain määrin hyödyllistä ainakin ihmisyksilön kannalta. Paljon puhutaan kokemuksen mukanaan tuomasta ns. "hiljaisesta tiedosta" joka ei ole selkeästi käsittein eikä matematiikan avulla eksaktisti ilmaistavissa mutta kuitenkin usein havaittavasti voi auttaa yksilön suoriutumista erilaisissa vastaan tulevissa arkielämän haasteissa.

        "monimaailmamalli on siinä merkityksessä elegantti, että tarvittava käsitteellinen vääntelehtiminen minimoituu. "

        Hyvä näin. Itse näen kaikki käsitteet ja selitysmallit lähinnä työkalujen kaltaisina eli ei ole mielekästä käyttää vasaraa jatkuvasti koska silloin maailma alkaa näyttää nauloilta helposti. Jos monimaailma toimii hyvin kvanttilaskennassa niin hyvä niin mutta se ei silti tarkoita että se monimaailmatulkintasi olisi jotenkin ontologisesti todempi kuin ne muut tulkinnat.

        "Maailmankaikkeuksia tosin tarvitaan erittäin suuri määrä, mutta kyseessä on vain määrällinen, eikä laadullinen asia, eli se ei ole Occamilainen rasite."

        Tuosta voisin olla eri mieltä...

        "Tässä tullaan myös kuvausten kompleksisuuteen, eli vaikka monimaailmamalli onkin deterministinen, niin mikään ei takaa, että sekään olisi vielä todellisuuden täydellinen kuvaus. Voi olla siis niin, että vaikka todellisuus onkin täysin deterministinen, niin sen täydellinen kuvaus ei edes ole mahdollista."

        Tuossa täydellisessä determinismissä on se ongelma että tieteilijän tulee jotenkin olla determinoitu löytämään se perimmäinen totuus olevaisesta. Täydellisessä determinismissä ei ole erehdyksiä eikä onnistumisia eikä siis oikeastaan älykkyyttäkään missään perinteisessä muodossa.

        Olisin taipuvaisempi olettamaan jonkinlaista suhteellista valinnanvapautta tiettyjen reunaehtojen vallitessa eli ihmisillä ja eliöillä suhteellisen korkeasti organisoituneina osittain avoimena systeemeinä olisi jonkin verran enemmän valinnanmahdollisuuksia kuin vähemmän organisoituneella "elottomalla" ympäristöllään.

        "Todellisia hiukkasia ovat ne, jotka voivat suorittaa sellaista laskentaa, jota voidaan käyttää pienentämään entropiaa paikallisesti."

        Tuo lause kolahti eniten kun luin aamulla nuo kaikki kirjoituksesi läpi ennen kuin aloin vastaamaan. Entropian pienentäminen paikallisesti varmaan onkin se kaikkein olennaisin asia elollisessa luonnossa ja tietoisuuden suhteen yleisemmin.

        Luonnossa tietysti tapahtuu entyropian lisääntymistäkin. Eikö silloinkin ne samat todelliset hiukkaset ole asialla?

        "kvanttilaskennan teoreettinen teho on käytännössä mahdollista selittää vain sellaisella maailmankaikkeudella, jossa olisi 2^q alkeishiukkasta, jossa q on kvanttilaskentaan osallistuvien alkeishiukkasten määrä. Eli, tarvittavien hiukkasten määrä on kvanttilaskentaan osallistuvien hiukkasten lukumäärän eksponenttifunktio. "

        Eli siis hiukkasten määrä suhteessa kvanttilaskennan tehoon on vähän samantapainen asia kuin bittien lukumäärä prosessorin käskykannassa (32 bit, 64 bit, 128 bit jne.)? Miten se qubit liittyy tähän? Ei kai siihen laskemiseen nyt kaikkia maailmankaikkeuden hiukkasia tarvita vai käsitinkö väärin?


      • metafysiikkaa
        FirstMatter kirjoitti:

        "Ei se olekaan vähemmän metafyysinen kun periaatteessa koko fysiikan käsitejärjestelmä on metafysiikkaa siinä mielessä että se on vain tulkinta havainnoista ja metafysiikasta vapaata "puhdasta" havainnointia ei ole olemassakaan. "

        Kuulostaa siltä, että sekoitat tuossa fysiikan ja metafysiikan toisiinsa kuin irti päästetty tehosekoitin. Oleellista tässä on se, että fysiikan antamat ennusteet ovat objektiivisesti todennettavissa, mikä tarkoittaa sitä, että ennusteita voidaan käyttää mm. mahdollistamaan entropian paikallinen vähentäminen. Jos ennusteet eivät olisi objektiivisesti paikkansapitäviä, niin tuon onnistuminen olisi tilastollisesti äärimmäisen epätodennäköistä.

        "Hiukkaset ovat matemaattisia konstruktioita ja kausaliteetti tai determinismi pätee vain suurten hiukkasjoukkojen suhteen tilastollisesti. Tässä yritetään vääntää todellisuutta epätoivoisesti tutun ja turvallisen newtonilaisen mallin suuntaan."

        Käsite ja se mihin se viittaa ovat eri asioita. Puhun tässä hiukkasista todellisina olioina, en matemaattisina konstruktioina. Hiukkaset ovat mitä todellisimpia, koska niiden avulla voidaan suorittaa laskentaa. Matemaattisten konstruktioiden avulla sen sijaan ei voida suorittaa laskentaa. Monimaailmatulkinta on deterministinen ja selittää mm. sen miten laskentaa voidaan suorittaa tehokkaasti.

        "Alkeishiukkanen on symboli jossa on osittain konkreettiseen havaintoon tai mittaukseen liittyvää mutta myös siihen abstraktiin teoriaan tai malliin liittyviä asioita eli se on ikäänkuin hyperlinkki..."

        Alkeishiukkanen on käytännössä osoittautunut yhdeksi toimivimmista käsitteistä, joten se ei ole virheellisesti mielletty. Konkreettiset asiat ovat välttämättömyys, joita ilman teoretisointi, filosofointi ja muu kermankuorinta eivät ylipäätään olisi mahdollisia. Esimerkiksi tietoisuus ei pysty suorittamaan yhtään kommunikoitavissa olevaa laskutoimitusta, vaan siihen tarvitaan alkeishiukkasia. Entropian paikallinen vähentäminen ja siten järjestyksen lisääminen on mahdollista vain laskennan avulla ja se taas toisaalta ei edellytä tietoisuutta.

        "Millä konstilla päättelet hiukkasten käyttäytymisestä tai systeemin rakenteesta virheettömästi tietoisuuden olemassaolon?"

        En tiedä, sillä en ole ehtinyt pohtia tuota paljoa. Itseä tarkkailemalla voi kuitenkin havaita, että tietoisuuden taso vaihtelee. Jos oletetaan, että samankaltaisista rakenteista koostuvat olennot omaavat myös samankaltaisia ominaisuuksia, niin silloin myös noilla olennoilla voi olla tietoisuus. Oma arvaukseni on se, että tietoisuuden taso laskee silloin jos järjestelmän osien välinen eläville olennoille tyypillinen tietoliikenne estyy. Tai käänteisesti ilmaisten, tiedon määrällä, jonka olento pystyy tiedostamaan laskennan kannalta hyödynnettävällä tavalla on taipumus kasvaa silloin jos ne osat, joista olennon tietojenkäsittelyjärjestelmä koostuu ovat asianmukaisessa vuorovaikutussuhteessa keskenään. Ilmeisesti tuo asianmukainen vuorovaikutus pitää sisällään mm. sellaisen tiedon vaihtoa, joka kuvaa järjestelmän itsensä tilaa.

        "Kuulostaa siltä, että sekoitat tuossa fysiikan ja metafysiikan toisiinsa kuin irti päästetty tehosekoitin. "

        Jos otetaan huomioon että havainnoissa aina esiintyy vahvaa prosessointia ja tieteessä ja koejärjestelyissä kaikki havainnot ja mittaukset tulkitaan jossain teoreettisessa viitekehyksessä niin tuskin mitään selkeää ja absoluuttista eroa fysiikan ja metafysiikan välillä on.

        "Oleellista tässä on se, että fysiikan antamat ennusteet ovat objektiivisesti todennettavissa, mikä tarkoittaa sitä, että ennusteita voidaan käyttää mm. mahdollistamaan entropian paikallinen vähentäminen. Jos ennusteet eivät olisi objektiivisesti paikkansapitäviä, niin tuon onnistuminen olisi tilastollisesti äärimmäisen epätodennäköistä."

        Juu tuohon on tietysti oleellisinta enkä väitä vastaan.

        "Käsite ja se mihin se viittaa ovat eri asioita. Puhun tässä hiukkasista todellisina olioina, en matemaattisina konstruktioina. "

        Hiukkasia kutsutaan usein kentän eksitaatioiksi joten ontologisesti ja filosofisesti ja ontologisesti se kenttä- tai aaltomalli sitten lienee lähempänä sitä konkreettista todellisuutta.

        Romahdusmallissa oletetaan että se kenttä romahtaa hiukkaseksi vasta havainnon tai vuorovaikutuksen kautta mutta siinäkin on ongelmana että periaattessa kai se havaintolaite (tai havaitsija) suhteessa siihen havainnoitavaan kohteeseen ovat tai voivat olla molemmat osia samasta kvanttifysikaalisesta systeemistä jonka voi ajatella olevan jopa maailmankaikkeuden suuruinen.

        "Hiukkanen todellisena oliona" on ontologinen eli puhtaasti filosofinen kysymys jota tieteen metodilla ei voi ratkaista eli sekin asia on metafysiikkaa. Tämä voi kuullostaa käytännön kvanttilaskennan kannalta hiustenhalkomiselta ja sitä se hyvin pitkälle onkin....

        "Matemaattisten konstruktioiden avulla sen sijaan ei voida suorittaa laskentaa. "

        Jos tuo on totta niin huippumatemaatikkojen aivojen toiminnan tutkiminen voisi olla tapa kehittää matematiikkaa. Perinteisesti ollaan ajateltu että ensin on se perusidea aineettomana ajatuksena jonkun mielessä josta sitten kehitetään fysikaalisten apuvälineiden avulla (paperi & kynä, tietokoneet) fysikaaliseen maailmaan vaikuttavia tuloksia. Newton ja Leibniz eivät siis kehittäneet differentiaalilaskentaa vaan heidän aivojensa hermosolujen konfiguraatio alunperin alkuräjähdyksen kausaliteetin aiheuttamana tuotti fysikaalisen hiukkaskokonaisuuden jota alettiin monistaa muihin aivoihin ja kutsua diffrentiaalilaskennaksi. Sen ymmärrän että hiukkaset voi mieltää aineellisiksi mutta onko niiden hiukkasten keskinäinen rakenne aineellista ja vain pelkästään johdettavissa luonnonlaieista?

        "Alkeishiukkanen on käytännössä osoittautunut yhdeksi toimivimmista käsitteistä, joten se ei ole virheellisesti mielletty. "

        'Alkeishiukkanen'on varmasti hyödyllinen käsite ja teoreettinen työkalu mutta onki sillä jotain ontologista olemassaolo tästä riippumatta on mielestäni taas eri asia.

        "Konkreettiset asiat ovat välttämättömyys, joita ilman teoretisointi, filosofointi ja muu kermankuorinta eivät ylipäätään olisi mahdollisia. "

        Filosofit ovat aikoinaan miettineet kirjoituspöytänsä konkreettisuutta ja alkeishiukkasten tapauksessa siihen filosofis-ontologiseen pohdintaan on varmasti paljon enemmän aihetta. Descartesin "Cogito ergo sum" kiteytti koko jutun siihen että ainoa oikasti konkreettinen asia on mielessä oleva kulloinenkin kokemuksellisuus eli se tietoisuus ja kaikki muu voi olla feikkiä. Tuo sinun kantasi on siis puhtaasti filosofinen valinta eikä sitä voi millään empirialla eikä tieteellisen metodin avulla todentaa.

        "Esimerkiksi tietoisuus ei pysty suorittamaan yhtään kommunikoitavissa olevaa laskutoimitusta, vaan siihen tarvitaan alkeishiukkasia. "

        Tuo "kommunikoitavissa oleva" taitaa olla se olennainen asia koska kaikki tieteellisesti yleisesti tunnettu ja hyväksytty kommunikaatio edellyttää fysikaalista hiukkasten siirtoa ja samalla entropian muutosta.

        "Entropian paikallinen vähentäminen ja siten järjestyksen lisääminen on mahdollista vain laskennan avulla ja se taas toisaalta ei edellytä tietoisuutta."

        Toisaalta se järjestyksen muutoksen havainnointi eli toteaminen taas edellyttää tietoisuutta jos tietoisuudella tässä tarkoitetaan tiedostettua ja prosessoitua havaintoa.

        "Itseä tarkkailemalla voi kuitenkin havaita, että tietoisuuden taso vaihtelee. "

        Niin vaihtelee ja joku kvanttifyysikko taisi aikoinaan esittää että aaltofunktion romahdus on sama asia kuin tietoisuus ja havainto mikä kuullostaa järkevältä ja mistä seuraa että tietoisuus ja samalla aika ja paikka ovat epäjatkuvia. Australiassa joku vuosi tehdyssä delayed choice quantum eraser kokeessa tutkijat totesivat että hiukkanen ei liiku paikasta A paikkaan B.

        "The atoms did not travel from A to B. It was only when they were measured at the end of the journey that their wave-like or particle-like behavior was brought into existence," he said.

        http://www.anu.edu.au/news/all-news/experiment-confirms-quantum-theory-weirdness


      • metafysiikkaa
        FirstMatter kirjoitti:

        "Tuskin. Kyllä matemaattinen mielikuvitus on paljon laajempi kuin fysikaaliset rakenteet.
        (esim. Mandelbrotin joukko)."

        Fysiikka asettaa rajat myös matematiikalle ja mielikuvitukselle. Esimerkiksi vain sellainen matematiikka voidaan havaita, jonka havaitsemiseen on käytettävissä riittävästi alkeishiukkasia, jotka ovat sopivissa suhteissa toisiinsa. Havaitsemattomasta matematiikasta puhuminen taas on puhdasta metafysiikkaa. Mandelbrotin joukon osien rakenne on mahdollista laskea ilman, että siihen tarvittaisiin lainkaan tietoisuutta, mutta tietenkin se edellyttää erittäin suurta paikallista järjestystä.

        "Mikä tahansa looginen konstruktio toimii jos se on sisäisesti koherentti. Ei luonnossa ole autoja, lentokoneita eikä tietokoneitakaan vaan ne ovat inhimillisen luomiskyvyn tuotteita.

        Kyllä ihmiset, autot, lentokoneet ja tietokoneet ovat siinä merkityksessä osa luontoa, että ne toimivat samojen luonnonlakien mukaisesti kuin muutkin asiat. Ihminen on vain luonnonvoimien agentti, joka kasaa nuo vempaimet. Luonnonvoimat siis vetävät naruista ja ihminen sätkii niiden tahdissa :)

        ""Jos tuota kieltä ei olisi koodattu fysikaalisiin rakenteisiin, niin sillä ei olisi mitään kuvaavaa tai ohjaavaa vaikutusta mihinkään.""
        "Niin jos kiellät tietoisuuden olemassaolon ja vaikutuksen niin tuollainen johtopäätös syntyy helposti."

        Kieli ilman fysiikkaa on Liisa ihmemaassa -juttuja, eli ihmisen mielikuvituksen tuotosta tietoisuudesta riippumatta.

        "Sekä että paitsi että se mitä kutsumme tietoisuudeksi on lähinnä monimutkainen takaisinkytkentäjärjestelmä joka ei sinänsä edellytä jatkuvaa tietoisuutta. Inhimillinen toiminta on hyvin suurelta osin automaattista."

        Kuulostaa järkevältä.

        "Tietoisuus sinänsä jää siten aina tieteellisen metodin ulottumattomiin ja ainoastaan voimme havaita tietoisuuden vaikutukset vain epäsuorasti kun kyse ei ole omasta tietoisuudestamme."

        Miten oletetut tietoisuuden vaikutukset voidaan erottaa aineen vaikutuksista, jos siis oletetaan, että noilla kahdella on eroa? Jos niitä ei voida erottaa, niin kyse on kaiketi joko metafysiikasta tai hiustenhalkomisesta.

        "En tiedä kun en edes usko mm-malliin jossa on sotkettu potentiaalisuus aktuaaliseen. Tulevaisuus on aina potentiaalista ja vastaavasti menneisyys on aina aktuaalista ja nykyhetkessä (joka voi olla eri ulotteinen eri oloilla) tapahtuu valinta tai valikoituu mekaanisesti yksi vaihtoehto aktuaaliseksi."

        Kuulostaa klassiselta ajattelulta, mutta todellisuushan on kvanttimekaaninen ja esimerkiksi kvanttien interferenssissä kaikki tosiaan tapahtuu samanaikaisesti. Eli, esimerkiksi kvanttitietokoneissa varsinainen kvanttilaskenta ei siis vie *lainkaan* aikaa. Tuosta huolimatta kyse on nimenomaan aktuaalisesta, eikä potentiaalisesta laskennasta, koska sillä voidaan ratkaista todellisia ongelmia ja vieläpä sellaisiakin ongelmia, joiden ratkaisu olisi muuten käytännössä mahdotontakin.

        "On eri asia pystyä mittaamaan ne superpositiot kuin että ne ovat olemassa."

        Laskennan kannalta niiden olemassaolo ei ole relevanttia, jos niitä ei pystytä mittaamaan, mutta oleellista tässä oli se, että niitä ei takuulla pystytä edes mittaamaan, jos ne ovat hajonneet mistä tahansa muistakaan syistä.

        " Monimaailmamallin mukaan voidaan ajatella, että käytännössä lähes kaikki epäjärjestys sijaitsee niissä rinnakkaisissa maailmankaikkeuksissa, joissa asiat ovat ns. menneet pieleen, eli niissä ei ole muodostunut sellaisia havaitsevia olentoja, jotka olisivat säilyneet hengissä, etenkään niin kauan, että olisivat voineet keskustella esimerkiksi argumenteistaan sen puolesta ja vastaan, että havaitun järjestyksen selittämiseen tarvitaan rinnakkaisia maailmankaikkeuksia tms. "

        Joku filosofi taisi joskus kirjoittaa jotain "parhaasta mahdollisesta maailmasta". Jos muita maailmankaikkeuksia voi tuolla tavalla käyttää roskiksina niin eikö siitä seuraa että niiden kaikkien multiversumeiden kokonaisuus voidaan nähdä yhtenä maailmankaikkeutena jonka ominaisuuksiin kuuluu haarautuminen eri ekolokeroihin vähän samalla tavalla kuin evoluutioteoriassa?

        "Älyllisten olentojen muodostuminen sattumalta ensimmäisellä yrityksellä (eli ensimmäisessä satunnaisessa ja ainoassa maailmankaikkeudessa) olisi erittäin epätodennäköistä, vaikka tuon maailmankaikkeuden massa olisikin näkyvän maailmankaikkeuden suuruusluokaltaan noin 10^23 Auringon massaa, koska oleellista tässä ei ole hiukkasten lukumäärä, vaan hiukkasten kokoonpano ja vaihtoehtoisten kokoonpanojen lukumäärä on hiukkasten lukumäärän eksponenttifunktio. "

        Jep ja juuri tuon takia olen suhtautunut hieman epäilevästi nykyisen evoluutioteorian esittämään skenaarioon. Jos esim. tavallinen kolibakteeri tietyissä olosuhteissa pystyy muuntautumaan 12 päivässä täysin resistansiksi erittäin vahvoille antibiooteille niin se minusta kertoo melko suuresta luonnon älykkyydestä vaikka kuinka otettaisiin huomioon että yhden bakteerin elinikä on hyvin lyhyt.

        https://www.vox.com/2016/9/8/12852924/evolution-bacteria-timelapse-video-mega-harvard

        Jos se ihmisenkin älykkyys on samankaltaista ja perustuu kvanttilaskentaan niin maailmanselittämisen kuvio muuttuu aika olennaisesti paljon nykyistä uskottavammaksi.
        "Tieteelliseltä kannalta oleellisinta on se, mitä testattavia ja mittauksilla todennettavissa olevia ennusteita erilaiset todellisuuden selitysmallit kykenevät tarjoamaan."

        Niin varmaan enkä väitä vastaan. Biologian paradigmana kvanttilaskentamalli voisi toimia paljon nykyistä melko epäuskottavaa tarinaa paremmin.

        "Insinööreillä on tapana kiinnittää huomionsa käytännössä hyödynnettäviin asioihin. "

        Ja se on nimenomaan hyvä asia. Olen taipuvainen arvostelemaan tieteellisten teorioiden todenmukaisuutta nimenomaan niiden teorioiden mahdollistamien teknisten sovellusten kautta ja tekniikan tuottaminen on mielestäni nimenomaan tieteen paras anti ihmiskunnalle. Teoria ilman toimivia käytännön sovelluksia on mielestäni enemmän spekulaatiota ja filosofiaa.

        "Mandelbrotin joukon osien rakenne on mahdollista laskea ilman, että siihen tarvittaisiin lainkaan tietoisuutta, mutta tietenkin se edellyttää erittäin suurta paikallista järjestystä."

        Mandelbrotin joukon laskeminen ja löytäminen onnistuukin vain tietotekniikan avulla ja Benoit Mandelbrot oli muistaakseni töissä IBM:llä kun kehitti tai löysi Mandelbrotin joukon joka lienee monimutkaisin ja kiehtovin matemaattinen algoritmi geometriseksi kuvioksi laskettuna vaikka itse algoritmi onkin hyvin yksinkertainen. Aikoinaan leikin tuon kanssa vuosikausia. Kaaosteorian tutkinen muutenkin mahdollistui vasta tietotekniikan avulla. Ei noita kukaan pysty laskemaan päässä eikä paperilla ilman nopeaa tietotekniikkaa.

        Voisi kyllä kuitenkin väittää Mandelbrotin ja vastaavien kompleksisten matemaattisten objektien konstruktio tai löytäminen ja huomaaminen edellyttää nimenomaan kehittynyttä tietoisuutta. Deterministisessä mekanistismaterialistisessa mallissa kaikkien ideoiden pitäisi olla piilevinä jo alkuräjähdyksen konfiguraatiossa mihin on vaikea uskoa.

        "Kyllä ihmiset, autot, lentokoneet ja tietokoneet ovat siinä merkityksessä osa luontoa, että ne toimivat samojen luonnonlakien mukaisesti kuin muutkin asiat. "

        Joo mutta se on eri asia kuin väittää autojen ja muiden ihmistekniikan saavutusten syntyneen pelkästään luonnonlakien seurauksena ikäänkuin itsestään mekaanisautomaattisesti.

        "Miten oletetut tietoisuuden vaikutukset voidaan erottaa aineen vaikutuksista, jos siis oletetaan, että noilla kahdella on eroa? Jos niitä ei voida erottaa, niin kyse on kaiketi joko metafysiikasta tai hiustenhalkomisesta."

        Kuten aikaisemmin jo oli puhetta niin osaamme erottaa luonnonilmiöt keinotekoisista ilmiöistä. Esim. Mount Rushmore eli presidenttien veistokset USA:ssa harvemmin tulkitaan puhtaasti eroosion tuottamaksi luonnonilmiöksi.

        "Eli, esimerkiksi kvanttitietokoneissa varsinainen kvanttilaskenta ei siis vie *lainkaan* aikaa."
        Liike liittyy aikaan ja jos ei ole aikaa niin ei ole liikettäkään eikä sitten oikeastaan entropiaakaan.


      • metafysiikkaa
        FirstMatter kirjoitti:

        "Tietoisuuden kysymyksen kannalta tällä on suuri merkitys koska se mahdollistaa aidon valinnan eri vaihtoehtojen välillä eli jonkinlaisen vapaan tahdon mitä klassinen deterministinen maailmankaikkeus eikä sen monistus monimaailmoiksi mahdollista."

        Vapaa tahto on diiba-daabaa, jota ei pitäisi ollenkaan sotkea tieteelliseen keskusteluun. Kyseessä ei ole edes tieteellinen käsite.

        "Ilman jonkinlaista vapaata tahtoa tietoisuus on pelkkä haamu ja korkeintaan pelkästään ulkopuolinen tarkkailija ilman minkäänlaista vaikutusmahdollisuutta tapahtumien kulkuun."

        Tuo on oikea kuvaus, eli tietoisuus on tavallaan taaksepäin katsova haamu, joka koostuu alkeishavainnoista ilman, että niillä on mahdollisuutta vaikuttaa mitenkään asioiden etenemiseen, sillä tapahtunuthan on jo tapahtunut myös kvanttimekaniikan puolella. Tietoisuus on myös edellytystensä funktio, kuten muutkin asiat. Eli, esimerkiksi jos on väsynyt, niin tietoisuus yksinkertaisesti häviää.

        Todennäköisesti tietoisuus on vain sivutuote siitä, että asiat ovat siinä järjestyksessä, joka mahdollistaa sen. Eli, tietoisuus edellyttää tietyn järjestyksen, josta voi syntyä illuusio siitä, että tietoisuus olisi jonkinlainen kausaalinen syy tuolle järjestykselle, vaikka todellisuudessa syy-seuraus-suhde menee juuri toisin päin. Vastaavasti esimerkiksi myös vapauden tunne on illuusio, joka syntyy siitä, että havaitsee asioiden etenevän tavalla, joka vastaa toiveita, vaikka asiat oikeasti etenevätkin luonnonlakien mukaisesti, kuten aina muulloinkin.

        "Ketjun aihe olisi tietysti huomattavasti helpompi käsitellä pelkästään momimaailmamallin vs. romahdustulkintojen suhteen."

        Keskustelu on päässyt hiukan rönsyilemään.

        "Tuskin edes tulevaisuuden kvanttitietokoneet pystyvät ennustamaan tulevaisuutta aukottoman tarkasti kun aina jostain voi kehkeytyä kaoottinen prosessi joka pakottaa järjestelmän uusiutumaan."

        Determinismi ja ennustettavuus ovat eri asioita. Vaikka todellisuus mahdollisesti pohjimmiltaan onkin täysin deterministinen, niin sen tarkka ennustaminen on silti hyvin hyvin vaikeaa tai mahdotonta. Lisäksi tietoisuuden rajallisuus mahdollistaa vapauden illuusion, joten subjektiiviselta kannalta todellisen vapauden puute ja tietoisuuden rajallisuus voivat vaikuttaa monista jopa toivottaviltakin asioilta.

        "Kvanttitietokoneiden uusi uljas maailma tuskin on kovin miellyttävä paikkaa elää ihmisyksilön kannalta..."

        Täytyy muistaa korostaa, että maailman ongelmat eivät ole tekniikan vaan sellaisten ihmisten aiheuttamia, jotka käyttävät tekniikkaa ja tietysti myös muita resursseja kyseenalaisiin tarkoituksiin. Mitä pintapuolistumiseen tulee, niin olettaisin, että kvanttitekniikan esiinmarssi tulee vähitellen lisäämään oleellisesti kysyntää matemaattisesti hyvin korkeasti koulutetuille henkilöille. Toisaalta laskennan edistyminen mahdollistaa myös sen, että ns. suuren yleisön tarvitsee vaivata entistäkin vähemmän päätänsä matematiikalla ja ongelmanratkaisulla. Ehkä tuota voidaan jopa pitää win-win -tilanteena.

        "Kvanttioperaatiot ovat välittömiä ja reversiibeleitä, eli ne eivät vie lainkaan aikaa, eivätkä ilmeisesti lisää entropiaa. Ilmainen lounas on siis mahdollinen, mutta se edellyttää erittäin vaativaa ja tarkkaa säätöä. "

        Tuo ilmainen lounas kuullostaa vähän ikiliikkujalta paitsi että tuossa kai pitää ottaa huomioon kuinka paljon kvanttitietokoneen ylläpito kuluttaa energiaa ja tuottaa entropiaa tähän systeemiin.

        "Varsinaista vastinetta klassiselle ohjelmalle ei ainakaan vielä kehityksen nykyvaiheessa ole, koska varsinainen kvanttilaskenta suoritetaan suoraan laitteistokomponenteilla varsin suoraviivaisesti. "

        Niin oletinkin koska nykyisissä prosessoreissa se konekielinen käskykin on vielä aika kaukana siitä sähköisestä toteutuksesta varsinkin jos konekielen käskykanta on vielä x86 pohjainen.

        "Vapaa tahto on diiba-daabaa, jota ei pitäisi ollenkaan sotkea tieteelliseen keskusteluun. Kyseessä ei ole edes tieteellinen käsite."

        Toiseksi näin on. En usko mihinkään absoluuttiseen vapaaseen valintaan ilman mitään reunaehtoja mutta toisaalta en sitten usko myöskään mihinkään superdeterminismiin jossa kaikki on mekaanis-kausaalisesti säädetty alkuräjähdyksestä lähtien. Jos ei ole minkäänlaista edes rajoitettua valinnanmahdollisuutta niin ei ole älykkyyttäkään eikä oikeastaan missään tieteessäkään ole mitään mieltä eikä missään kommunikoinnissa ylipäätänsä.

        "Eli, esimerkiksi jos on väsynyt, niin tietoisuus yksinkertaisesti häviää. "

        No minä olen jo aika väsynyt kun tämä kirjoittelu ja keskittyminen vaatii paljon energiaa mutta ei se tietoisuus häviä vaan sen ylläpitäminen vaatii yhä enemmän ponnistelua ja vaivannäköä. Tietysti jos tätäkin jatkaa tuntikausia yhteenmenoon niin alkaa tulla kirjoitus - ja ajatusvirheitä ja vireyden palautuminen vaatii lepohetken.

        "Todennäköisesti tietoisuus on vain sivutuote siitä, että asiat ovat siinä järjestyksessä, joka mahdollistaa sen."

        Kokemamme tietoisuus on varmasti monen fysikaalisen takaisinkytkennän monimutkainen tuote mutta silti uskon tietoisuuden olevan maailmankaikkeuden perusontologiaan kuuluva asia ja ehkä nimenomaan juurikin sen aineen konfiguraation kokemuksellinen komponentti eli aine on se miltä todellisuus aisteille näyttää kun taas tietoisuus on se miltä se subjektiivisesti tuntuu.

        " Vastaavasti esimerkiksi myös vapauden tunne on illuusio, joka syntyy siitä, että havaitsee asioiden etenevän tavalla, joka vastaa toiveita, vaikka asiat oikeasti etenevätkin luonnonlakien mukaisesti, kuten aina muulloinkin."

        Varmasti vapauden tunne on usein hyvin suuressa määrin illuusio kun ei tiedosteta niitä sidonnaisuuksia ja syy-seuraussuhteita eri asioihin mutta toisaalta eikö se niiden sidonnaisuuksien tiedostaminen mahdollista oman käytöksen muuttamista ja takaisinkytkentöjen säätöä termostaatin tavoin eli tavallaan oman kehon itsesäätelyä joka on tavallaan tahdon vapautta.

        "Keskustelu on päässyt hiukan rönsyilemään."

        Keskustelen eri ihmisten kanssa vähän eri tavalla joten se rönsyily vähän lisääntyi kun tuli muita mukaan. Kirjoitellessa yritän keskittyä keskustelukumppanin ajatteluun eli kyseessä on takaisinkytkentä. Asiallisille keskustelijoille pyrin vastaamaan asiallisesti ja vähemmän asiallisille joko jätän vastaamatta tai vastaan samalla mitalla fiiliksistä ja vireystilasta riippuen.

        "Determinismi ja ennustettavuus ovat eri asioita. Vaikka todellisuus mahdollisesti pohjimmiltaan onkin täysin deterministinen, niin sen tarkka ennustaminen on silti hyvin hyvin vaikeaa tai mahdotonta."

        Joo ne ovat eri asioita mutta toisaalta mistä tietää että todellisuus on pohjimmiltaan deterministinen jos sitä ei voi todentaa käytännössä eli ennustamalla oikein?

        "Lisäksi tietoisuuden rajallisuus mahdollistaa vapauden illuusion, joten subjektiiviselta kannalta todellisen vapauden puute ja tietoisuuden rajallisuus voivat vaikuttaa monista jopa toivottaviltakin asioilta."

        Tietoisuuden lisääntyminen nimenomaan mielestäni mahdollistaa rajoitetun ja realistisen valinnan vapauden kun tiedostaa ne todellliset rajoitukset toiminnalleen ja voi sitten yrittää kiertää tai ylittää ne rajoitukset.


        "Täytyy muistaa korostaa, että maailman ongelmat eivät ole tekniikan vaan sellaisten ihmisten aiheuttamia, jotka käyttävät tekniikkaa ja tietysti myös muita resursseja kyseenalaisiin tarkoituksiin. "
        Tuo on totta. Toisaalta ihmiskunta ei ehkä ole vielä valmis kvanttilaskennan mahdollistamiin asioihin eikä tuskin ole valmis vielä perinteisen tietotekniikankan suhteen. Vähän sama kuin pikkulapsille antaisi tulitikut leikkiin.


      • metafysiikkaa
        metafysiikkaa kirjoitti:

        "Kvanttioperaatiot ovat välittömiä ja reversiibeleitä, eli ne eivät vie lainkaan aikaa, eivätkä ilmeisesti lisää entropiaa. Ilmainen lounas on siis mahdollinen, mutta se edellyttää erittäin vaativaa ja tarkkaa säätöä. "

        Tuo ilmainen lounas kuullostaa vähän ikiliikkujalta paitsi että tuossa kai pitää ottaa huomioon kuinka paljon kvanttitietokoneen ylläpito kuluttaa energiaa ja tuottaa entropiaa tähän systeemiin.

        "Varsinaista vastinetta klassiselle ohjelmalle ei ainakaan vielä kehityksen nykyvaiheessa ole, koska varsinainen kvanttilaskenta suoritetaan suoraan laitteistokomponenteilla varsin suoraviivaisesti. "

        Niin oletinkin koska nykyisissä prosessoreissa se konekielinen käskykin on vielä aika kaukana siitä sähköisestä toteutuksesta varsinkin jos konekielen käskykanta on vielä x86 pohjainen.

        "Vapaa tahto on diiba-daabaa, jota ei pitäisi ollenkaan sotkea tieteelliseen keskusteluun. Kyseessä ei ole edes tieteellinen käsite."

        Toiseksi näin on. En usko mihinkään absoluuttiseen vapaaseen valintaan ilman mitään reunaehtoja mutta toisaalta en sitten usko myöskään mihinkään superdeterminismiin jossa kaikki on mekaanis-kausaalisesti säädetty alkuräjähdyksestä lähtien. Jos ei ole minkäänlaista edes rajoitettua valinnanmahdollisuutta niin ei ole älykkyyttäkään eikä oikeastaan missään tieteessäkään ole mitään mieltä eikä missään kommunikoinnissa ylipäätänsä.

        "Eli, esimerkiksi jos on väsynyt, niin tietoisuus yksinkertaisesti häviää. "

        No minä olen jo aika väsynyt kun tämä kirjoittelu ja keskittyminen vaatii paljon energiaa mutta ei se tietoisuus häviä vaan sen ylläpitäminen vaatii yhä enemmän ponnistelua ja vaivannäköä. Tietysti jos tätäkin jatkaa tuntikausia yhteenmenoon niin alkaa tulla kirjoitus - ja ajatusvirheitä ja vireyden palautuminen vaatii lepohetken.

        "Todennäköisesti tietoisuus on vain sivutuote siitä, että asiat ovat siinä järjestyksessä, joka mahdollistaa sen."

        Kokemamme tietoisuus on varmasti monen fysikaalisen takaisinkytkennän monimutkainen tuote mutta silti uskon tietoisuuden olevan maailmankaikkeuden perusontologiaan kuuluva asia ja ehkä nimenomaan juurikin sen aineen konfiguraation kokemuksellinen komponentti eli aine on se miltä todellisuus aisteille näyttää kun taas tietoisuus on se miltä se subjektiivisesti tuntuu.

        " Vastaavasti esimerkiksi myös vapauden tunne on illuusio, joka syntyy siitä, että havaitsee asioiden etenevän tavalla, joka vastaa toiveita, vaikka asiat oikeasti etenevätkin luonnonlakien mukaisesti, kuten aina muulloinkin."

        Varmasti vapauden tunne on usein hyvin suuressa määrin illuusio kun ei tiedosteta niitä sidonnaisuuksia ja syy-seuraussuhteita eri asioihin mutta toisaalta eikö se niiden sidonnaisuuksien tiedostaminen mahdollista oman käytöksen muuttamista ja takaisinkytkentöjen säätöä termostaatin tavoin eli tavallaan oman kehon itsesäätelyä joka on tavallaan tahdon vapautta.

        "Keskustelu on päässyt hiukan rönsyilemään."

        Keskustelen eri ihmisten kanssa vähän eri tavalla joten se rönsyily vähän lisääntyi kun tuli muita mukaan. Kirjoitellessa yritän keskittyä keskustelukumppanin ajatteluun eli kyseessä on takaisinkytkentä. Asiallisille keskustelijoille pyrin vastaamaan asiallisesti ja vähemmän asiallisille joko jätän vastaamatta tai vastaan samalla mitalla fiiliksistä ja vireystilasta riippuen.

        "Determinismi ja ennustettavuus ovat eri asioita. Vaikka todellisuus mahdollisesti pohjimmiltaan onkin täysin deterministinen, niin sen tarkka ennustaminen on silti hyvin hyvin vaikeaa tai mahdotonta."

        Joo ne ovat eri asioita mutta toisaalta mistä tietää että todellisuus on pohjimmiltaan deterministinen jos sitä ei voi todentaa käytännössä eli ennustamalla oikein?

        "Lisäksi tietoisuuden rajallisuus mahdollistaa vapauden illuusion, joten subjektiiviselta kannalta todellisen vapauden puute ja tietoisuuden rajallisuus voivat vaikuttaa monista jopa toivottaviltakin asioilta."

        Tietoisuuden lisääntyminen nimenomaan mielestäni mahdollistaa rajoitetun ja realistisen valinnan vapauden kun tiedostaa ne todellliset rajoitukset toiminnalleen ja voi sitten yrittää kiertää tai ylittää ne rajoitukset.


        "Täytyy muistaa korostaa, että maailman ongelmat eivät ole tekniikan vaan sellaisten ihmisten aiheuttamia, jotka käyttävät tekniikkaa ja tietysti myös muita resursseja kyseenalaisiin tarkoituksiin. "
        Tuo on totta. Toisaalta ihmiskunta ei ehkä ole vielä valmis kvanttilaskennan mahdollistamiin asioihin eikä tuskin ole valmis vielä perinteisen tietotekniikankan suhteen. Vähän sama kuin pikkulapsille antaisi tulitikut leikkiin.

        "Mitä pintapuolistumiseen tulee, niin olettaisin, että kvanttitekniikan esiinmarssi tulee vähitellen lisäämään oleellisesti kysyntää matemaattisesti hyvin korkeasti koulutetuille henkilöille. "

        Varmaan pitää paikkansa mutta sellaisia on toistaiseksi hyvin vähän tässä ihmispopulaatiossa. Kehittyneen tekniikan kääntöpuoli on se että siitä tullaan helposti äärimmäisen riippuvaisiksi ja se taas voi aiheuttaa katastrofi ja poikkeustilanteissa uhan koko sivilisaation ja ihmiskunnan olemassaololle.

        "Toisaalta laskennan edistyminen mahdollistaa myös sen, että ns. suuren yleisön tarvitsee vaivata entistäkin vähemmän päätänsä matematiikalla ja ongelmanratkaisulla. Ehkä tuota voidaan jopa pitää win-win -tilanteena."

        Eli koneet tulevatm koko ajan fiksummiksi ja ihmiset vastaavasti keskimäärin tyhmemmiksi kun aina vain vähemmän tarvitsee itse vaivautua ajattelemaan ja ponnistelemaan missään asiassa.

        ...

        Kiitos. Tämä oli varmaan paras ja samalla asiallisin keskustelu mihin itse olen tällä palstalla osallistunut. Työlästä mutta erittäin antoisaa. Tunne on vähän kuin olisi maratonin juossut. Nyt täytyy lähteä asioille ja pitää sitten lepohetki.


      • metafysiikkaa
        metafysiikkaa kirjoitti:

        "Kuulostaa siltä, että sekoitat tuossa fysiikan ja metafysiikan toisiinsa kuin irti päästetty tehosekoitin. "

        Jos otetaan huomioon että havainnoissa aina esiintyy vahvaa prosessointia ja tieteessä ja koejärjestelyissä kaikki havainnot ja mittaukset tulkitaan jossain teoreettisessa viitekehyksessä niin tuskin mitään selkeää ja absoluuttista eroa fysiikan ja metafysiikan välillä on.

        "Oleellista tässä on se, että fysiikan antamat ennusteet ovat objektiivisesti todennettavissa, mikä tarkoittaa sitä, että ennusteita voidaan käyttää mm. mahdollistamaan entropian paikallinen vähentäminen. Jos ennusteet eivät olisi objektiivisesti paikkansapitäviä, niin tuon onnistuminen olisi tilastollisesti äärimmäisen epätodennäköistä."

        Juu tuohon on tietysti oleellisinta enkä väitä vastaan.

        "Käsite ja se mihin se viittaa ovat eri asioita. Puhun tässä hiukkasista todellisina olioina, en matemaattisina konstruktioina. "

        Hiukkasia kutsutaan usein kentän eksitaatioiksi joten ontologisesti ja filosofisesti ja ontologisesti se kenttä- tai aaltomalli sitten lienee lähempänä sitä konkreettista todellisuutta.

        Romahdusmallissa oletetaan että se kenttä romahtaa hiukkaseksi vasta havainnon tai vuorovaikutuksen kautta mutta siinäkin on ongelmana että periaattessa kai se havaintolaite (tai havaitsija) suhteessa siihen havainnoitavaan kohteeseen ovat tai voivat olla molemmat osia samasta kvanttifysikaalisesta systeemistä jonka voi ajatella olevan jopa maailmankaikkeuden suuruinen.

        "Hiukkanen todellisena oliona" on ontologinen eli puhtaasti filosofinen kysymys jota tieteen metodilla ei voi ratkaista eli sekin asia on metafysiikkaa. Tämä voi kuullostaa käytännön kvanttilaskennan kannalta hiustenhalkomiselta ja sitä se hyvin pitkälle onkin....

        "Matemaattisten konstruktioiden avulla sen sijaan ei voida suorittaa laskentaa. "

        Jos tuo on totta niin huippumatemaatikkojen aivojen toiminnan tutkiminen voisi olla tapa kehittää matematiikkaa. Perinteisesti ollaan ajateltu että ensin on se perusidea aineettomana ajatuksena jonkun mielessä josta sitten kehitetään fysikaalisten apuvälineiden avulla (paperi & kynä, tietokoneet) fysikaaliseen maailmaan vaikuttavia tuloksia. Newton ja Leibniz eivät siis kehittäneet differentiaalilaskentaa vaan heidän aivojensa hermosolujen konfiguraatio alunperin alkuräjähdyksen kausaliteetin aiheuttamana tuotti fysikaalisen hiukkaskokonaisuuden jota alettiin monistaa muihin aivoihin ja kutsua diffrentiaalilaskennaksi. Sen ymmärrän että hiukkaset voi mieltää aineellisiksi mutta onko niiden hiukkasten keskinäinen rakenne aineellista ja vain pelkästään johdettavissa luonnonlaieista?

        "Alkeishiukkanen on käytännössä osoittautunut yhdeksi toimivimmista käsitteistä, joten se ei ole virheellisesti mielletty. "

        'Alkeishiukkanen'on varmasti hyödyllinen käsite ja teoreettinen työkalu mutta onki sillä jotain ontologista olemassaolo tästä riippumatta on mielestäni taas eri asia.

        "Konkreettiset asiat ovat välttämättömyys, joita ilman teoretisointi, filosofointi ja muu kermankuorinta eivät ylipäätään olisi mahdollisia. "

        Filosofit ovat aikoinaan miettineet kirjoituspöytänsä konkreettisuutta ja alkeishiukkasten tapauksessa siihen filosofis-ontologiseen pohdintaan on varmasti paljon enemmän aihetta. Descartesin "Cogito ergo sum" kiteytti koko jutun siihen että ainoa oikasti konkreettinen asia on mielessä oleva kulloinenkin kokemuksellisuus eli se tietoisuus ja kaikki muu voi olla feikkiä. Tuo sinun kantasi on siis puhtaasti filosofinen valinta eikä sitä voi millään empirialla eikä tieteellisen metodin avulla todentaa.

        "Esimerkiksi tietoisuus ei pysty suorittamaan yhtään kommunikoitavissa olevaa laskutoimitusta, vaan siihen tarvitaan alkeishiukkasia. "

        Tuo "kommunikoitavissa oleva" taitaa olla se olennainen asia koska kaikki tieteellisesti yleisesti tunnettu ja hyväksytty kommunikaatio edellyttää fysikaalista hiukkasten siirtoa ja samalla entropian muutosta.

        "Entropian paikallinen vähentäminen ja siten järjestyksen lisääminen on mahdollista vain laskennan avulla ja se taas toisaalta ei edellytä tietoisuutta."

        Toisaalta se järjestyksen muutoksen havainnointi eli toteaminen taas edellyttää tietoisuutta jos tietoisuudella tässä tarkoitetaan tiedostettua ja prosessoitua havaintoa.

        "Itseä tarkkailemalla voi kuitenkin havaita, että tietoisuuden taso vaihtelee. "

        Niin vaihtelee ja joku kvanttifyysikko taisi aikoinaan esittää että aaltofunktion romahdus on sama asia kuin tietoisuus ja havainto mikä kuullostaa järkevältä ja mistä seuraa että tietoisuus ja samalla aika ja paikka ovat epäjatkuvia. Australiassa joku vuosi tehdyssä delayed choice quantum eraser kokeessa tutkijat totesivat että hiukkanen ei liiku paikasta A paikkaan B.

        "The atoms did not travel from A to B. It was only when they were measured at the end of the journey that their wave-like or particle-like behavior was brought into existence," he said.

        http://www.anu.edu.au/news/all-news/experiment-confirms-quantum-theory-weirdness

        Näköjään yksi osuus unohtui iltapäivällä copypastata editorista tänne:

        Aika-paikka koordinaatisto (eli fysiikkatieteen perusta) on tietoisuuden luoma konstruktio eli nopeus ja paikka ovat tavallaan illuusioita ja mielen konstruktioita.

        "Jos oletetaan, että samankaltaisista rakenteista koostuvat olennot omaavat myös samankaltaisia ominaisuuksia, niin silloin myös noilla olennoilla voi olla tietoisuus."

        Rakenteeltaan samankaltaisilla ja varsinkin samalla tavalla käyttäytyvillä systeemeillä voi olla samankaltainen tietoisuus jos niillä on mitään tietoisuuteen viittaavaa. Vaikka aine näyttäytyy aina atomaarisena eli osista koostuneena niin tietoisuuden kokemus on suppeimmillaankin aina yhtenäinen ja siis holistinen kokemus.

        Kommunikaation mahdollisuus lienee edellyttää melko suurta samankaltaisuutta ja jos hyvin erilaisilla olioilla on olemassa kehittynyt tietoisuus niin on mahdollista että niillä on myös hyvin erilainen käsitys kokemusmaailmastaan ja kommunikointi voi olla hyvin vaikeaa tai mahdotonta. Ehkä ne tavallaan elävät täysin meistä riippumattomassa rinnakkaisuniversumissa. Ehkä on myös ennenaikaista olettaa että se meille konkreettisena koettu fysikaalinen todellisuus on se "oikea" kun se fysiikkatieteenkin maailmankuva on riippuvainen nimenomaan ihmislajin rakenteesta ja varsinkin aivojen toiminnasta.

        "Oma arvaukseni on se, että tietoisuuden taso laskee silloin jos järjestelmän osien välinen eläville olennoille tyypillinen tietoliikenne estyy."

        Ainakin ulkoisten ärsykkeiden väheneminen auttaa keskittymisessä. Aivokurkiaisen eli aivopuoliskoja yhdistävän kudoksen katkaiseminen taas tuottaa samaan kehoon kaksi erilaista itsenäistä tietoisuutta ja vähän samantapainen ilmiö esiintyy monipersoonasyndroomeissa jossa hyvinkin erilaiset luonteet ja persoonat voivat esiintyy vuorotellen samassa kehossa.

        Tietoisuus siis pikemminkin jakautuu tai dissosioituu kun aivojen eri osien kommunikointi estyy. Yksilöllinen tietoisuus siis lienee jonkinlainen dissosiaatioilmiö. Kaikissa näissä tapauksissa se kokemuksellisuus on aina yhtenäistä.

        "Tai käänteisesti ilmaisten, tiedon määrällä, jonka olento pystyy tiedostamaan laskennan kannalta hyödynnettävällä tavalla on taipumus kasvaa silloin jos ne osat, joista olennon tietojenkäsittelyjärjestelmä koostuu ovat asianmukaisessa vuorovaikutussuhteessa keskenään."

        Niin keskittyminen ja toimiva ajattelu edellyttää melko suurta koherenssia ja energian kulutuskin lienee varsin suurta jos verrrataan aivojen käyttämää energiamäärää muiden kehon osien kuluttamaan energiaan. Aivoilla ja tietotekniikalla lienee kyllä jonkinlainen rakenneyhteys vaikka tuskin kyseessä on digitaalinen laskenta perinteisen tietotekniikan kaltaisesti. Ehkä aivot ovat jonkinlainen kvanttitietokone.


    • Kvanttimekaniikassa havainto vaikuttaa havaittavaan silloinkin, kun havainnon tekijä on tyhmä laite ja mittaustuloksen puretaan sen muistista vasta jälkikäteen. Mitään välitöntä tietoisuuden vaatimusta ei systeemiin vaikuttavan havainnon yhteydessä ole siis olemassa. Puun kaatumisesta metsässä kuuluu ääni silloinkin, kun sitä ei ole kukaan kuulemassa.

      Jos vaadit tietoisen havainnoitsijan mutta hyväksyt viiveen havainnon tekemisestä sen tiedostamiseen (kuten tuossa yllä) niin silloin alkuräjähdyksen vaatiman havainnon tekemiseen riität sinä itse. Viivettä välissä on 13.8 miljardia vuotta mutta sillähän ei olekaan väliä...

      • metafysiikkaa

        "Kvanttimekaniikassa havainto vaikuttaa havaittavaan silloinkin, kun havainnon tekijä on tyhmä laite ja mittaustuloksen puretaan sen muistista vasta jälkikäteen. "

        Paitsi jos oikeasti se havainto ja aaltofunktion romahdus tapahtuu vasta siinä vaiheessa kun se mittaustulos puretaan ja joku tietoinen havaitsija näkee sen tuloksen. Delayed choice quantum eraser kokeessa periaatteessa tapahtuu tällainen (ikäänkuin) retrokausaalisuus.


      • metafysiikkaa kirjoitti:

        "Kvanttimekaniikassa havainto vaikuttaa havaittavaan silloinkin, kun havainnon tekijä on tyhmä laite ja mittaustuloksen puretaan sen muistista vasta jälkikäteen. "

        Paitsi jos oikeasti se havainto ja aaltofunktion romahdus tapahtuu vasta siinä vaiheessa kun se mittaustulos puretaan ja joku tietoinen havaitsija näkee sen tuloksen. Delayed choice quantum eraser kokeessa periaatteessa tapahtuu tällainen (ikäänkuin) retrokausaalisuus.

        Odottelen innolla sitä päivää, jolloin joku tuon kaltainen koejärjestely tekee niin ison reiän suhtikseen tai kvanttimekaniikkaan, että siitä kaivamalla löytyy uusi ja parempi versio jommastakummasta tai mielellään molemmista. Vielä jos löytyisi sellainen/sellaiset, jossa sekä yleinen suhteellisuusteoria että kvanttimekaniikka sopisivat yhteen ilman renormalisaatio-ongelmia. Yrityksen puutteestahan ei tosiaan ole kysymys.

        Kun suhtiksen ja kvanttimekaniikan kanssa on eletty jo yli sata vuotta, niin alkaisi olla aika tuolle.


    • Huutiukko

      Kohdistin puppugeneraattoritutkani tähän ketjuun ja se tutka ölähti kovasti! Liekö huomannut jotain puppua?

    • Valonnopeus on ääretön; mistä seuraa : suhteellisuusteoria surkastuu lähes olemattomiin, massa ei häviä ydinreaktioissa, massan säilymisen laki, kosmologisia mustia aukkoja ei ole, avaruus ei erityisemmin laajene eikä kosmologisia mustia on ole. Elektronin varausta ei voi määrittää, mistä seuraa; kirjatiedot elektronin varauksesta, massasta, protonin massasta ja varauksesta, Avogadron luvusta ja atomin säteesta ja neutronin massasta ovat vääriä.

      • metafysiikkaa

        "Valonnopeus on ääretön; mistä seuraa..."

        Kuullostaa hyvin lupaavalta (":-)").

        Harva taitaa tietää että Einstein itse kannatti eetterin olemassaoloa 1920 kirjoituksissaan mutta tulkitsi sen eri tavalla kuin Michelson-Morley kokeessa. Nykyinen vastine eetterille lienee ns. tyhjiöenergia.

        "The ether of the general theory of relativity is a medium which is itself devoid of all mechanical and kinematical qualities, but helps to determine mechanical (and electromagnetic) events."

        ja

        “To deny ether is ultimately to assume that empty space has no physical quality whatever. The fundamental facts of quantum mechanics do not harmonize with this view.”

        ...

        Tuskin valonnopeus on ääretön mutta toisaalta vuorovaikutukset voivat olla paljon nopeampia kuin ns. herz-aallot jos ajatellaan Teslan skalaariaallot jotka ovat oikeastaan paineaaltoja ja voivat vaikuttaa hyvin paljon valoa nopeammin esim. gravitaatioilmiössä.

        Tesla esitti mm. että ääni kulkee avaruuden eetterissä paljon valoa nopeammin. Tuskin sähkämagnetismikaan rajoittuu valon nopeuteen koska muuten se tarkoittaisi sitä että esim. maapallon magneettiset etelä & pohjoisnapa eivät voisi vuorovaikuttaa toistensa kanssa kuin viiveellä puhumattakaan auringosta ja galakseista.

        " : suhteellisuusteoria surkastuu lähes olemattomiin, massa ei häviä ydinreaktioissa, massan säilymisen laki, kosmologisia mustia aukkoja ei ole, avaruus ei erityisemmin laajene eikä kosmologisia mustia on ole. Elektronin varausta ei voi määrittää, mistä seuraa; kirjatiedot elektronin varauksesta, massasta, protonin massasta ja varauksesta, Avogadron luvusta ja atomin säteesta ja neutronin massasta ovat vääriä."

        Koko fysiikan kokonaisuus koostuu toisiinsa kiinteästi nivoutuneista enemmän tai vähemmän valistuneista arvauksista eikä tarvita kun yhden perusoletuksen kumoutuminen tai osoittautuminen epäilyttäväksi niin suurin osa teoreettisesta mallista joutaa saman tien roskiin.

        Tuskin massa häviää ydinreaktiossa vaan kyseessä on energian purkautumin eetterin nollapiste-energiasta.

        Teslalla oli paljon mielenkiintoisia ja oikeasti toimivia ideoita ja hän oli paljon käytännönläheisempi ja luovempi tyyppi kuin Einstein joka oli pelkästään ajatuskokeita tekevä filosofinen teoreetikko.

        https://teslaresearch.jimdo.com/dynamic-theory-of-gravity/

        Ehkä olisi hyödyllisempää yrittää yhdistää Teslan ideat kvanttifysiikkaan ja unohtaa kokonaan Einstein ja suhtis jonka kanssa on tuhlattu aikaa jo melkein sata vuotta hyvin vaatimattomin tuloksin. Teslan koko henkilökohtainen tuotanto muuten takavarikoitiin hänen kuollessaan ja se on vieläkin salaista. Ehkä siellä on jotain mitä valtaapitävät eivät halunneet julkisuuteen ja Einstein suhtiksineen kelpasi lähinnä vain tavallisen kansan ja akateemisen seurapiirin harhautustarkoituksiin.

        Suhtiksesta on tullut jonkinlainen kanonisoitu uskonkappale jonka kanssa mikään tutkimus ei saa olla ristiriidassa. Tesla kehitti vaihtovirran joka käytössä joka paikassa kun taas Einsteinin suhtiksen tekniset sovellukset voi laskea yhdellä sormella eli GPS ja sekin on melko kyseenalaista että liittyykö se gps yhtään mitenkään suhtikseen.


        ps. Tässä siis huutiukon kehittämälle huipputeknologialle vähän lisää testiaineistoa... ;-)


      • Huutiukko
        metafysiikkaa kirjoitti:

        "Valonnopeus on ääretön; mistä seuraa..."

        Kuullostaa hyvin lupaavalta (":-)").

        Harva taitaa tietää että Einstein itse kannatti eetterin olemassaoloa 1920 kirjoituksissaan mutta tulkitsi sen eri tavalla kuin Michelson-Morley kokeessa. Nykyinen vastine eetterille lienee ns. tyhjiöenergia.

        "The ether of the general theory of relativity is a medium which is itself devoid of all mechanical and kinematical qualities, but helps to determine mechanical (and electromagnetic) events."

        ja

        “To deny ether is ultimately to assume that empty space has no physical quality whatever. The fundamental facts of quantum mechanics do not harmonize with this view.”

        ...

        Tuskin valonnopeus on ääretön mutta toisaalta vuorovaikutukset voivat olla paljon nopeampia kuin ns. herz-aallot jos ajatellaan Teslan skalaariaallot jotka ovat oikeastaan paineaaltoja ja voivat vaikuttaa hyvin paljon valoa nopeammin esim. gravitaatioilmiössä.

        Tesla esitti mm. että ääni kulkee avaruuden eetterissä paljon valoa nopeammin. Tuskin sähkämagnetismikaan rajoittuu valon nopeuteen koska muuten se tarkoittaisi sitä että esim. maapallon magneettiset etelä & pohjoisnapa eivät voisi vuorovaikuttaa toistensa kanssa kuin viiveellä puhumattakaan auringosta ja galakseista.

        " : suhteellisuusteoria surkastuu lähes olemattomiin, massa ei häviä ydinreaktioissa, massan säilymisen laki, kosmologisia mustia aukkoja ei ole, avaruus ei erityisemmin laajene eikä kosmologisia mustia on ole. Elektronin varausta ei voi määrittää, mistä seuraa; kirjatiedot elektronin varauksesta, massasta, protonin massasta ja varauksesta, Avogadron luvusta ja atomin säteesta ja neutronin massasta ovat vääriä."

        Koko fysiikan kokonaisuus koostuu toisiinsa kiinteästi nivoutuneista enemmän tai vähemmän valistuneista arvauksista eikä tarvita kun yhden perusoletuksen kumoutuminen tai osoittautuminen epäilyttäväksi niin suurin osa teoreettisesta mallista joutaa saman tien roskiin.

        Tuskin massa häviää ydinreaktiossa vaan kyseessä on energian purkautumin eetterin nollapiste-energiasta.

        Teslalla oli paljon mielenkiintoisia ja oikeasti toimivia ideoita ja hän oli paljon käytännönläheisempi ja luovempi tyyppi kuin Einstein joka oli pelkästään ajatuskokeita tekevä filosofinen teoreetikko.

        https://teslaresearch.jimdo.com/dynamic-theory-of-gravity/

        Ehkä olisi hyödyllisempää yrittää yhdistää Teslan ideat kvanttifysiikkaan ja unohtaa kokonaan Einstein ja suhtis jonka kanssa on tuhlattu aikaa jo melkein sata vuotta hyvin vaatimattomin tuloksin. Teslan koko henkilökohtainen tuotanto muuten takavarikoitiin hänen kuollessaan ja se on vieläkin salaista. Ehkä siellä on jotain mitä valtaapitävät eivät halunneet julkisuuteen ja Einstein suhtiksineen kelpasi lähinnä vain tavallisen kansan ja akateemisen seurapiirin harhautustarkoituksiin.

        Suhtiksesta on tullut jonkinlainen kanonisoitu uskonkappale jonka kanssa mikään tutkimus ei saa olla ristiriidassa. Tesla kehitti vaihtovirran joka käytössä joka paikassa kun taas Einsteinin suhtiksen tekniset sovellukset voi laskea yhdellä sormella eli GPS ja sekin on melko kyseenalaista että liittyykö se gps yhtään mitenkään suhtikseen.


        ps. Tässä siis huutiukon kehittämälle huipputeknologialle vähän lisää testiaineistoa... ;-)

        Toimii. Älähti kovasti. Kiitos testiaineistosta!


      • metafysiikkaa

        "Toimii. Älähti kovasti. Kiitos testiaineistosta!"

        Joo oli siellä tosiaan jotain pielessä. Itse asiassa näköjään CIA julkaisi 2001 Teslan skalaariaaltoja koskevaa aikaisemmin salaista aineistoa:

        https://www.cia.gov/library/readingroom/docs/CIA-RDP96-00792R000500240001-6.pdf

        Huom Dokumentti on suoraan se CIA:n arkistosta ja siinä on vieläkin tummennettuja osia.

        Foliot varmuuden vuoksi sitten päähän ennen latausta koska ihan takuuvarmasti he siellä seuraavat kuka tuota käy lukemassa.


      • kjhkhkjkh
        metafysiikkaa kirjoitti:

        "Toimii. Älähti kovasti. Kiitos testiaineistosta!"

        Joo oli siellä tosiaan jotain pielessä. Itse asiassa näköjään CIA julkaisi 2001 Teslan skalaariaaltoja koskevaa aikaisemmin salaista aineistoa:

        https://www.cia.gov/library/readingroom/docs/CIA-RDP96-00792R000500240001-6.pdf

        Huom Dokumentti on suoraan se CIA:n arkistosta ja siinä on vieläkin tummennettuja osia.

        Foliot varmuuden vuoksi sitten päähän ennen latausta koska ihan takuuvarmasti he siellä seuraavat kuka tuota käy lukemassa.

        Ihan varmasti. Uskonmävielätonkin tai sitten en.


      • metafysiikkaa kirjoitti:

        "Valonnopeus on ääretön; mistä seuraa..."

        Kuullostaa hyvin lupaavalta (":-)").

        Harva taitaa tietää että Einstein itse kannatti eetterin olemassaoloa 1920 kirjoituksissaan mutta tulkitsi sen eri tavalla kuin Michelson-Morley kokeessa. Nykyinen vastine eetterille lienee ns. tyhjiöenergia.

        "The ether of the general theory of relativity is a medium which is itself devoid of all mechanical and kinematical qualities, but helps to determine mechanical (and electromagnetic) events."

        ja

        “To deny ether is ultimately to assume that empty space has no physical quality whatever. The fundamental facts of quantum mechanics do not harmonize with this view.”

        ...

        Tuskin valonnopeus on ääretön mutta toisaalta vuorovaikutukset voivat olla paljon nopeampia kuin ns. herz-aallot jos ajatellaan Teslan skalaariaallot jotka ovat oikeastaan paineaaltoja ja voivat vaikuttaa hyvin paljon valoa nopeammin esim. gravitaatioilmiössä.

        Tesla esitti mm. että ääni kulkee avaruuden eetterissä paljon valoa nopeammin. Tuskin sähkämagnetismikaan rajoittuu valon nopeuteen koska muuten se tarkoittaisi sitä että esim. maapallon magneettiset etelä & pohjoisnapa eivät voisi vuorovaikuttaa toistensa kanssa kuin viiveellä puhumattakaan auringosta ja galakseista.

        " : suhteellisuusteoria surkastuu lähes olemattomiin, massa ei häviä ydinreaktioissa, massan säilymisen laki, kosmologisia mustia aukkoja ei ole, avaruus ei erityisemmin laajene eikä kosmologisia mustia on ole. Elektronin varausta ei voi määrittää, mistä seuraa; kirjatiedot elektronin varauksesta, massasta, protonin massasta ja varauksesta, Avogadron luvusta ja atomin säteesta ja neutronin massasta ovat vääriä."

        Koko fysiikan kokonaisuus koostuu toisiinsa kiinteästi nivoutuneista enemmän tai vähemmän valistuneista arvauksista eikä tarvita kun yhden perusoletuksen kumoutuminen tai osoittautuminen epäilyttäväksi niin suurin osa teoreettisesta mallista joutaa saman tien roskiin.

        Tuskin massa häviää ydinreaktiossa vaan kyseessä on energian purkautumin eetterin nollapiste-energiasta.

        Teslalla oli paljon mielenkiintoisia ja oikeasti toimivia ideoita ja hän oli paljon käytännönläheisempi ja luovempi tyyppi kuin Einstein joka oli pelkästään ajatuskokeita tekevä filosofinen teoreetikko.

        https://teslaresearch.jimdo.com/dynamic-theory-of-gravity/

        Ehkä olisi hyödyllisempää yrittää yhdistää Teslan ideat kvanttifysiikkaan ja unohtaa kokonaan Einstein ja suhtis jonka kanssa on tuhlattu aikaa jo melkein sata vuotta hyvin vaatimattomin tuloksin. Teslan koko henkilökohtainen tuotanto muuten takavarikoitiin hänen kuollessaan ja se on vieläkin salaista. Ehkä siellä on jotain mitä valtaapitävät eivät halunneet julkisuuteen ja Einstein suhtiksineen kelpasi lähinnä vain tavallisen kansan ja akateemisen seurapiirin harhautustarkoituksiin.

        Suhtiksesta on tullut jonkinlainen kanonisoitu uskonkappale jonka kanssa mikään tutkimus ei saa olla ristiriidassa. Tesla kehitti vaihtovirran joka käytössä joka paikassa kun taas Einsteinin suhtiksen tekniset sovellukset voi laskea yhdellä sormella eli GPS ja sekin on melko kyseenalaista että liittyykö se gps yhtään mitenkään suhtikseen.


        ps. Tässä siis huutiukon kehittämälle huipputeknologialle vähän lisää testiaineistoa... ;-)

        GPS ei käytä suhteellisuusteoria a , eikä satelliittejä.. GPS-laite määrittää kahden lähimmän GSM-tukiaseman suunnan ja laskee sijainnin siitä.


      • thoyssa kirjoitti:

        GPS ei käytä suhteellisuusteoria a , eikä satelliittejä.. GPS-laite määrittää kahden lähimmän GSM-tukiaseman suunnan ja laskee sijainnin siitä.

        Miksiköhän GPS sitten toimii vaikka keskellä ulappaa tabletissa tai navigaattorissa, jossa ei edes ole GSM:ää?

        Pohdipa sitä omassa rauhassasi: pitkään ja hartaasti.


      • ValonnopeudenMitannut
        Kollimaattori kirjoitti:

        Miksiköhän GPS sitten toimii vaikka keskellä ulappaa tabletissa tai navigaattorissa, jossa ei edes ole GSM:ää?

        Pohdipa sitä omassa rauhassasi: pitkään ja hartaasti.

        Thoyssa on aito sekopää, johon mikään argumentti ei pure. Hänen maailmassaan valonnopeus on ääretön eikä satelliitteja ole olemassa. Mikään todiste ei hänen mieltään käännä, jos ei sitten psykiatri onnistu saamaan lääkitystä kohilleen.
        En tiedä, miksi kaikki latvaB:eet tunkevat juurikin tiedepalstoille Savorisesta lähtien.


      • Huutiukko
        ValonnopeudenMitannut kirjoitti:

        Thoyssa on aito sekopää, johon mikään argumentti ei pure. Hänen maailmassaan valonnopeus on ääretön eikä satelliitteja ole olemassa. Mikään todiste ei hänen mieltään käännä, jos ei sitten psykiatri onnistu saamaan lääkitystä kohilleen.
        En tiedä, miksi kaikki latvaB:eet tunkevat juurikin tiedepalstoille Savorisesta lähtien.

        Miksi ei?


      • Kollimaattori kirjoitti:

        Miksiköhän GPS sitten toimii vaikka keskellä ulappaa tabletissa tai navigaattorissa, jossa ei edes ole GSM:ää?

        Pohdipa sitä omassa rauhassasi: pitkään ja hartaasti.

        GPS ei toimi keskellä Atlanttia gsm-kentän ulkopuolella. Navigaattorissa on gsm-vastaanotin.


      • Varmasti-Toimii
        thoyssa kirjoitti:

        GPS ei toimi keskellä Atlanttia gsm-kentän ulkopuolella. Navigaattorissa on gsm-vastaanotin.

        Joissain navigaattoreissa on MYÖS GSM vastaanotin, jolla se saa karkean paikannuksen ja lukittuu nopeammin satelliitteihin. GPS toimii takuuvarmasti myös alueilla, joissa ei GSM verkkoa ole, kuten Atlantilla. Olen ollut Lapissa vaelluksella alueella, jossa viimeinenkin GSM tukiasema painui radiohorisontin taakse, mutta GPS toimi kuten muuallakin.
        Kirjoitat täyttä sontaa.


      • Kyllä GSM toimii koko Lapissa.


      • ValonnopeudenMitannut kirjoitti:

        Thoyssa on aito sekopää, johon mikään argumentti ei pure. Hänen maailmassaan valonnopeus on ääretön eikä satelliitteja ole olemassa. Mikään todiste ei hänen mieltään käännä, jos ei sitten psykiatri onnistu saamaan lääkitystä kohilleen.
        En tiedä, miksi kaikki latvaB:eet tunkevat juurikin tiedepalstoille Savorisesta lähtien.

        Taidatpa olla oikeassa.


      • HöpötitiHöpöttää
        thoyssa kirjoitti:

        Kyllä GSM toimii koko Lapissa.

        Ei toimi ainakaan operaattoreiden kuuluvuuskarttojen perusteella.

        https://kartat.dna.fi/Peittokartta/

        Sodankylästä pohjoiseen Suomen rajojen sisäpuolella on iso alue, jolla kuuluvuutta ei ole. Sama jotto Suomen päälaen paikkeilla Mieraslompolon ja Näätämön välissä. Väitteesi perusteella siellä GPSsän ei pitänyt toimia eli höpötät.


    • On satelliittejä olemassa

      • metafysiikkaa

        Vielä tuosta GPS:tä: Vaikka nyt olettetaisiin suhtis oikeaksi niin en silti ymmärrä miksi siihen gps-tekniikkaan pitää sotkea jotain relativistisia laskutoimituksia.

        " Special Relativity predicts that the on-board atomic clocks on the satellites should fall behind clocks on the ground by about 7 microseconds per day because of the slower ticking rate due to the time dilation effect of their relative motion [2]. "

        Kovasti tässäkin artikkelissa yritetään vakuuttaa että suhtis ei ole vain pelkästään abstrakti matemaattinen teoria:

        "Relativity is not just some abstract mathematical theory: understanding it is absolutely essential for our global navigation system to work properly! "

        http://www.astronomy.ohio-state.edu/~pogge/Ast162/Unit5/gps.html

        Ei kai ne satelliittien nopeudet ole niin suuria että oikeasti joku relativistinen aikadilaatio vaikuttaisi oikeasti asiaan eikä ne satelliitit kulje kuin muutaman sadan kilometrin korkeudella ja nopeuskin paljon valon nopeutta hitaampi.

        Jos siis suhtiksen mukaan satelliitin atomikello jätättää n. 7 mikrosekuntia (=sekunnin miljoonasosa) päivässä verrattuna maanpäälliseen atomikelloon niin tuskin se niin paljoa siihen maanpäälliseen paikantamiseen vaikuttaa varsinkin kun sen satelliitin kellon voisi synkronisoida vaikka kerran tunnissa tai jopa kerran päivässäkin varmaan riittäisi riittävään navigointitarkkuuteen. Ei oikein edelleenkään vakuuta tuo gps:n toimivuuden liittäminen suhtikseen.


      • jyhfiufivuhf
        metafysiikkaa kirjoitti:

        Vielä tuosta GPS:tä: Vaikka nyt olettetaisiin suhtis oikeaksi niin en silti ymmärrä miksi siihen gps-tekniikkaan pitää sotkea jotain relativistisia laskutoimituksia.

        " Special Relativity predicts that the on-board atomic clocks on the satellites should fall behind clocks on the ground by about 7 microseconds per day because of the slower ticking rate due to the time dilation effect of their relative motion [2]. "

        Kovasti tässäkin artikkelissa yritetään vakuuttaa että suhtis ei ole vain pelkästään abstrakti matemaattinen teoria:

        "Relativity is not just some abstract mathematical theory: understanding it is absolutely essential for our global navigation system to work properly! "

        http://www.astronomy.ohio-state.edu/~pogge/Ast162/Unit5/gps.html

        Ei kai ne satelliittien nopeudet ole niin suuria että oikeasti joku relativistinen aikadilaatio vaikuttaisi oikeasti asiaan eikä ne satelliitit kulje kuin muutaman sadan kilometrin korkeudella ja nopeuskin paljon valon nopeutta hitaampi.

        Jos siis suhtiksen mukaan satelliitin atomikello jätättää n. 7 mikrosekuntia (=sekunnin miljoonasosa) päivässä verrattuna maanpäälliseen atomikelloon niin tuskin se niin paljoa siihen maanpäälliseen paikantamiseen vaikuttaa varsinkin kun sen satelliitin kellon voisi synkronisoida vaikka kerran tunnissa tai jopa kerran päivässäkin varmaan riittäisi riittävään navigointitarkkuuteen. Ei oikein edelleenkään vakuuta tuo gps:n toimivuuden liittäminen suhtikseen.

        Ei pelkkä nopeus vaan myös gravitaatio.

        Mikrosekunneista seuraa kertautuva viive ja vaikutus maan päällisessä koordinaatistossa on suurempi kuin ymmärrät.

        Siksi satelliittien kelloja synkronoidaankin.

        Miksi edes lainaat tekstejä kun ilmeisen selvästi et kykene niistä mitään ymmärtämään? Lainaamassasi teksinpätkässähän kerrottiin juurikin vastaukset siihen mitä nyt kyselet.

        Suosittelisin keskittymistä johonkin maanläheisempään harrastukseen mihin kykysi riittäisivät. Nyt on veneentervaussesonki.


      • metafysiikkaa
        jyhfiufivuhf kirjoitti:

        Ei pelkkä nopeus vaan myös gravitaatio.

        Mikrosekunneista seuraa kertautuva viive ja vaikutus maan päällisessä koordinaatistossa on suurempi kuin ymmärrät.

        Siksi satelliittien kelloja synkronoidaankin.

        Miksi edes lainaat tekstejä kun ilmeisen selvästi et kykene niistä mitään ymmärtämään? Lainaamassasi teksinpätkässähän kerrottiin juurikin vastaukset siihen mitä nyt kyselet.

        Suosittelisin keskittymistä johonkin maanläheisempään harrastukseen mihin kykysi riittäisivät. Nyt on veneentervaussesonki.

        "Ei pelkkä nopeus vaan myös gravitaatio."

        Sama gravitaatiovaikutushan on myös maan pinnalla kun etäisyydet ja suhteelliset nopeudet ovat melko pieniä.

        "Miksi edes lainaat tekstejä kun ilmeisen selvästi et kykene niistä mitään ymmärtämään? "

        Oletko ihan varma että itse ymmärrät?

        "Lainaamassasi teksinpätkässähän kerrottiin juurikin vastaukset siihen mitä nyt kyselet. "

        Joo mutta tuossa linkin selityksessä ei ole mitään järkeä eli se haiskahtaa pahasti huijaukselta jonka sinäkin ilmeisesti olet omaksunut ajattelematta asiaa.


      • ikuweiuqwfuqef

        >>Sama gravitaatiovaikutushan on myös maan pinnalla kun etäisyydet ja suhteelliset nopeudet ovat melko pieniä. <<

        Maan pinnalla vahvempi, kiertoradalla heikompi. Oletko läpäissyt peruskoulun?

        >>Oletko ihan varma että itse ymmärrät?<<

        Niin varma että ei ole järjellistä epäillä sitä.

        >>Joo mutta tuossa linkin selityksessä ei ole mitään järkeä eli se haiskahtaa pahasti huijaukselta jonka sinäkin ilmeisesti olet omaksunut ajattelematta asiaa.<<

        Linkin selityksessä on järkeä ja se on teoreettisesti ja kokeellisesti vahvistettu. Epäilyissäsi ei ole järkeä.

        Jos aiot harrastaa tai peräti opiskella tiedettä, ei ole kovin hedelmällistä epäillä ja luulla kaikkea huijaukseksi jo lähtökohtaisesti. Siinä otat vain itsellesi takamatkaa muihin nähden, ja näet lähtöviivan häämöttävän kaukana kun toiset ovat jo pinkaisseet siltä vauhtiin, jos ymmärrät vertauksen. Siinä vaiheessa kun olet vakuuttanut itsesi nykyisten tietojen paikkansa pitävyydestä ja pitäisi ryhtyä opettelemaan uutta, olet jo vanha mies ja kykenemätön oppimaan uutta, puhumattakaan että olisit kykeneväinen edistämään tiedettä.

        Mutta väliäkö tuolla, sillä tiedämmehän että et ole harrastamassa tai opiskelemassa alaa vaan olet vain p*ska trolli. Toivottavasti, tai muuten toivottoman tyhmä ihminen.


      • metafysiikkaa
        metafysiikkaa kirjoitti:

        "Ei pelkkä nopeus vaan myös gravitaatio."

        Sama gravitaatiovaikutushan on myös maan pinnalla kun etäisyydet ja suhteelliset nopeudet ovat melko pieniä.

        "Miksi edes lainaat tekstejä kun ilmeisen selvästi et kykene niistä mitään ymmärtämään? "

        Oletko ihan varma että itse ymmärrät?

        "Lainaamassasi teksinpätkässähän kerrottiin juurikin vastaukset siihen mitä nyt kyselet. "

        Joo mutta tuossa linkin selityksessä ei ole mitään järkeä eli se haiskahtaa pahasti huijaukselta jonka sinäkin ilmeisesti olet omaksunut ajattelematta asiaa.

        Kaikkein hämärin osuus on tässä:

        "For example, to counteract the General Relativistic effect once on orbit, the onboard clocks were designed to "tick" at a slower frequency than ground reference clocks, so that once they were in their proper orbit stations their clocks would appear to tick at about the correct rate as compared to the reference atomic clocks at the GPS ground stations. Further, each GPS receiver has built into it a microcomputer that, in addition to performing the calculation of position using 3D trilateration, will also compute any additional special relativistic timing calculations required [3], using data provided by the satellites."


        Ne atomikellot (muka) pitää esisäätää hitaammaksi ennen satelliitin radalle lähettämistä jolloin ne sitten näyttäisivät toimivan (huomaa alkup. sanavalinta "appear") ns. "oikealla" nopeudella.

        Kaiken lisäksi tekstin mukaan gps-vastaanottimessa tehdään vastaavat relativistiset laskutoimitukset vielä toiseen kertaan (eikö ne kumoa toisensa jos molemmissa päissä tehdään samat säädöt?). Ihan kuin satelliittien ja maan pinnan kellojen synkronisointi ei muka onnistuisi paljon helpomminkin ihan newtonilaistakin fysiikkaa käyttäen.... :-)

        Kukaan ei sitten tietenkään oikeasti pysty tarkistamaan onko noissa virityksissä yhtään mitään relativistista kun koko järjestelmä on huippusalainen koska sitä käytetään sotilaallisissa tarkoituksissa ja on nimenomaan kehitetty alunperin sotilaskäyttöön.

        Nokkelaa esittää ainoana suhtiksen käytännön sovelluksena tekniikka joka on huippusalaista ja jota ei pääse kukaan tarkistamaan että löytyykö sieltä mitään suhtikseen liittyvää.

        Uskokoon ken haluaa minä en jaksa....


      • metafysiikkaa
        ikuweiuqwfuqef kirjoitti:

        >>Sama gravitaatiovaikutushan on myös maan pinnalla kun etäisyydet ja suhteelliset nopeudet ovat melko pieniä. <<

        Maan pinnalla vahvempi, kiertoradalla heikompi. Oletko läpäissyt peruskoulun?

        >>Oletko ihan varma että itse ymmärrät?<<

        Niin varma että ei ole järjellistä epäillä sitä.

        >>Joo mutta tuossa linkin selityksessä ei ole mitään järkeä eli se haiskahtaa pahasti huijaukselta jonka sinäkin ilmeisesti olet omaksunut ajattelematta asiaa.<<

        Linkin selityksessä on järkeä ja se on teoreettisesti ja kokeellisesti vahvistettu. Epäilyissäsi ei ole järkeä.

        Jos aiot harrastaa tai peräti opiskella tiedettä, ei ole kovin hedelmällistä epäillä ja luulla kaikkea huijaukseksi jo lähtökohtaisesti. Siinä otat vain itsellesi takamatkaa muihin nähden, ja näet lähtöviivan häämöttävän kaukana kun toiset ovat jo pinkaisseet siltä vauhtiin, jos ymmärrät vertauksen. Siinä vaiheessa kun olet vakuuttanut itsesi nykyisten tietojen paikkansa pitävyydestä ja pitäisi ryhtyä opettelemaan uutta, olet jo vanha mies ja kykenemätön oppimaan uutta, puhumattakaan että olisit kykeneväinen edistämään tiedettä.

        Mutta väliäkö tuolla, sillä tiedämmehän että et ole harrastamassa tai opiskelemassa alaa vaan olet vain p*ska trolli. Toivottavasti, tai muuten toivottoman tyhmä ihminen.

        "Jos aiot harrastaa tai peräti opiskella tiedettä, ei ole kovin hedelmällistä epäillä ja luulla kaikkea huijaukseksi jo lähtökohtaisesti. "

        Kaikkea ei ole tosiaan tarpeen epäillä mutta jotkut asiat (kuten tämä gps; ja suhtiksen liitto) vaikuttavat epäilyttäviltä monestakin syystä.

        Ja sinä sitten uskot kaiken mitä joku auktoriteetti sanoo "tieteeksi" ilman minkäänlaista omaa arvostelukykyä ja pohdintaa? Miten tuo eroaa keskiajan ihmisestä joka uskoi sokeasti kaiken mitä pappi sanoi ja kun uskonnolliset tekstitkin oli silloin latinaa ja kansa enimmäkseen lukutaidotonta ?

        Ihminen on hyvin suggestioaltis otus ja tiedeyhteisö on nykyajan papisto ja pitkälle matematisoitu tiede taas nykyajan munkkilatinaa harvoille valituille ja tohtoreiksi vihityille.


      • hgjhgjhgjhgj
        metafysiikkaa kirjoitti:

        "Jos aiot harrastaa tai peräti opiskella tiedettä, ei ole kovin hedelmällistä epäillä ja luulla kaikkea huijaukseksi jo lähtökohtaisesti. "

        Kaikkea ei ole tosiaan tarpeen epäillä mutta jotkut asiat (kuten tämä gps; ja suhtiksen liitto) vaikuttavat epäilyttäviltä monestakin syystä.

        Ja sinä sitten uskot kaiken mitä joku auktoriteetti sanoo "tieteeksi" ilman minkäänlaista omaa arvostelukykyä ja pohdintaa? Miten tuo eroaa keskiajan ihmisestä joka uskoi sokeasti kaiken mitä pappi sanoi ja kun uskonnolliset tekstitkin oli silloin latinaa ja kansa enimmäkseen lukutaidotonta ?

        Ihminen on hyvin suggestioaltis otus ja tiedeyhteisö on nykyajan papisto ja pitkälle matematisoitu tiede taas nykyajan munkkilatinaa harvoille valituille ja tohtoreiksi vihityille.

        "Kaikkea ei ole tosiaan tarpeen epäillä mutta jotkut asiat (kuten tämä gps; ja suhtiksen liitto) vaikuttavat epäilyttäviltä monestakin syystä. "

        Mandrake hyvä. Siinä ei ole mitään epäilyttävää. Voit työntää metafysiikkasi sinne mihin päivä ei paista eikä morfiset kentät ulotu.


      • metafysiikkaa
        hgjhgjhgjhgj kirjoitti:

        "Kaikkea ei ole tosiaan tarpeen epäillä mutta jotkut asiat (kuten tämä gps; ja suhtiksen liitto) vaikuttavat epäilyttäviltä monestakin syystä. "

        Mandrake hyvä. Siinä ei ole mitään epäilyttävää. Voit työntää metafysiikkasi sinne mihin päivä ei paista eikä morfiset kentät ulotu.

        "Mandrake hyvä. Siinä ei ole mitään epäilyttävää. Voit työntää metafysiikkasi sinne mihin päivä ei paista eikä morfiset kentät ulotu."

        Asialliset argumentit sinulta taisi näköjään taas loppua. Katson voittaneeni taas tämänkin väittelyn... :-)


      • ikuweiuqwfuqef
        metafysiikkaa kirjoitti:

        "Jos aiot harrastaa tai peräti opiskella tiedettä, ei ole kovin hedelmällistä epäillä ja luulla kaikkea huijaukseksi jo lähtökohtaisesti. "

        Kaikkea ei ole tosiaan tarpeen epäillä mutta jotkut asiat (kuten tämä gps; ja suhtiksen liitto) vaikuttavat epäilyttäviltä monestakin syystä.

        Ja sinä sitten uskot kaiken mitä joku auktoriteetti sanoo "tieteeksi" ilman minkäänlaista omaa arvostelukykyä ja pohdintaa? Miten tuo eroaa keskiajan ihmisestä joka uskoi sokeasti kaiken mitä pappi sanoi ja kun uskonnolliset tekstitkin oli silloin latinaa ja kansa enimmäkseen lukutaidotonta ?

        Ihminen on hyvin suggestioaltis otus ja tiedeyhteisö on nykyajan papisto ja pitkälle matematisoitu tiede taas nykyajan munkkilatinaa harvoille valituille ja tohtoreiksi vihityille.

        Luulet nyt kahta yhdeksi, etkä ole mitään väittelyä voittanut, älykääpiö.

        Jep, kuoriutuihan sieltä aito hörhö tiedeuskontohoureineen ja auktoriteettiusko-itkuineen. Miten voittekin olla kaikki noin kaavamaisen samanlaisia. Teistä pitäisi itse asiassa tehdä tieteellinen tutkimus tai peräti väitöskirja, tarjoatte mitä parhaimman raakamateriaalin ihan itse ja ilmaiseksi.

        "matematisoitu tiede"

        Heh heh. Käsilläkö huitomalla tai kuperkeikkoja heittelemälläkö tiedettä mielestäsi pitäisi tehdä? Tai kertomalla jänniä juttuja? Noitarumpua päräyttelemällä?

        Tieteellä on niin paljon arjen sovelluksia että ei ole järjellistä luulotella kuten sinä. Itse asiassa olet terveenä, ainakin fyysisesti. tieteen ansiosta etkä esim. polion rampauttama, ja suollat roskaasi tiedemiesten sinulle kehittämällä koneella ja viestintävälineellä, ja kehtaat olla noinkin kiittämätön tieteelle ja tiedemiehille. Ihan puoliaivoisille hörhöille tyypillistä tosin.


        "Uskokoon ken haluaa minä en jaksa...."

        Kuule, ei tarvitse jaksaa eikä uskoa. Nämä asiat eivät vaadi sinun uskoasi eivätkä sinua. Voit painua v*ttuun täältä ja ryömiä takaisin sinne hörhöpalstoillesi, ristiretkesi ei nyt onnistunut.


      • metafysiikkaa
        ikuweiuqwfuqef kirjoitti:

        Luulet nyt kahta yhdeksi, etkä ole mitään väittelyä voittanut, älykääpiö.

        Jep, kuoriutuihan sieltä aito hörhö tiedeuskontohoureineen ja auktoriteettiusko-itkuineen. Miten voittekin olla kaikki noin kaavamaisen samanlaisia. Teistä pitäisi itse asiassa tehdä tieteellinen tutkimus tai peräti väitöskirja, tarjoatte mitä parhaimman raakamateriaalin ihan itse ja ilmaiseksi.

        "matematisoitu tiede"

        Heh heh. Käsilläkö huitomalla tai kuperkeikkoja heittelemälläkö tiedettä mielestäsi pitäisi tehdä? Tai kertomalla jänniä juttuja? Noitarumpua päräyttelemällä?

        Tieteellä on niin paljon arjen sovelluksia että ei ole järjellistä luulotella kuten sinä. Itse asiassa olet terveenä, ainakin fyysisesti. tieteen ansiosta etkä esim. polion rampauttama, ja suollat roskaasi tiedemiesten sinulle kehittämällä koneella ja viestintävälineellä, ja kehtaat olla noinkin kiittämätön tieteelle ja tiedemiehille. Ihan puoliaivoisille hörhöille tyypillistä tosin.


        "Uskokoon ken haluaa minä en jaksa...."

        Kuule, ei tarvitse jaksaa eikä uskoa. Nämä asiat eivät vaadi sinun uskoasi eivätkä sinua. Voit painua v*ttuun täältä ja ryömiä takaisin sinne hörhöpalstoillesi, ristiretkesi ei nyt onnistunut.

        "Luulet nyt kahta yhdeksi, etkä ole mitään väittelyä voittanut, älykääpiö."

        Se oli tarkoitettu lähinnä vitsiksi (hymynaama lopussa) ja syötiksi johon sinä heti tartuit kunnon sätkynukketyyliin.

        Ota iisisti ja muista ottaa nitrot. Tuollainen hillitön tunnekuohuilu voi käydä pumpun päälle.

        Vahvan uskomuksen tunnistaa aina tunnereaktiosta. Todelinen tieteilijä ja tiedefani osaa olla neutraali eikä pidä tiedettä minään "pyhänä" uskonnollisena asiana kuten tyypilliset kaltaisesi kiihkouskovaiset joille tiede on jonkinlainen uskonnon korvike kuten tutti vauvoille. Sinulla ja tyypillisellä fundamentalistiuskovaisella on enemmän yhteistä kuin tajuatkaan.

        Jätän nyt sinut rauhoittumaan.


      • ikuweiuqwfuqef
        metafysiikkaa kirjoitti:

        "Luulet nyt kahta yhdeksi, etkä ole mitään väittelyä voittanut, älykääpiö."

        Se oli tarkoitettu lähinnä vitsiksi (hymynaama lopussa) ja syötiksi johon sinä heti tartuit kunnon sätkynukketyyliin.

        Ota iisisti ja muista ottaa nitrot. Tuollainen hillitön tunnekuohuilu voi käydä pumpun päälle.

        Vahvan uskomuksen tunnistaa aina tunnereaktiosta. Todelinen tieteilijä ja tiedefani osaa olla neutraali eikä pidä tiedettä minään "pyhänä" uskonnollisena asiana kuten tyypilliset kaltaisesi kiihkouskovaiset joille tiede on jonkinlainen uskonnon korvike kuten tutti vauvoille. Sinulla ja tyypillisellä fundamentalistiuskovaisella on enemmän yhteistä kuin tajuatkaan.

        Jätän nyt sinut rauhoittumaan.

        Et nyt tajua kuka sen syötin oikeasti jätti ja kuka siihen tarttui; todistit itsesi trolliksi kuten suunnittelinkin.

        ;)


      • aihetta.tutkimukseen
        metafysiikkaa kirjoitti:

        "Luulet nyt kahta yhdeksi, etkä ole mitään väittelyä voittanut, älykääpiö."

        Se oli tarkoitettu lähinnä vitsiksi (hymynaama lopussa) ja syötiksi johon sinä heti tartuit kunnon sätkynukketyyliin.

        Ota iisisti ja muista ottaa nitrot. Tuollainen hillitön tunnekuohuilu voi käydä pumpun päälle.

        Vahvan uskomuksen tunnistaa aina tunnereaktiosta. Todelinen tieteilijä ja tiedefani osaa olla neutraali eikä pidä tiedettä minään "pyhänä" uskonnollisena asiana kuten tyypilliset kaltaisesi kiihkouskovaiset joille tiede on jonkinlainen uskonnon korvike kuten tutti vauvoille. Sinulla ja tyypillisellä fundamentalistiuskovaisella on enemmän yhteistä kuin tajuatkaan.

        Jätän nyt sinut rauhoittumaan.

        "Todelinen tieteilijä ja tiedefani osaa olla neutraali eikä pidä tiedettä minään "pyhänä" uskonnollisena asiana kuten tyypilliset kaltaisesi kiihkouskovaiset joille tiede on jonkinlainen uskonnon korvike kuten tutti vauvoille. "

        Kukaan ei ole pitänyt tiedettä minään pyhänä ja vielä vähemmän uskonnollisena asiana muualla kuin pöljissä jutuissasi. tuollaiset houreet kertovat vain kirjoittajansa harhoista tiedettä koskien.

        Sinunlaisillasi ei ole mitään annettavaa tiedepalstoille, joten miksi et tyydy hörhöilemään hörhöpalstoilla? Täällä et tee muuta kuin itsestäsi naurettavan kuten nyt.

        Mikä siinä on että täytyy kerta toisensa jälkeen nöyryyttää itseänsä julkisesti, ja korvata tietoisuus siitä katteettomalla hybriksellä?

        Nakertaako pattia kun et ymmärrä tiedettä tai matematiikkaa? Veivätkö tiede ja ilkeät tiedemiehet hörhöltä ufo/kummitus/henkimaailma/yksisarvis-tikkarin vai mistä moinen katkeruus kumpuaa?

        Kerro motiivisi niin ehkäpä sinulla olisi sen yhden kerran tiedonjanoisille ihmisille jotakin annettavaa.


      • kjhkjhkjhkjh
        metafysiikkaa kirjoitti:

        "Mandrake hyvä. Siinä ei ole mitään epäilyttävää. Voit työntää metafysiikkasi sinne mihin päivä ei paista eikä morfiset kentät ulotu."

        Asialliset argumentit sinulta taisi näköjään taas loppua. Katson voittaneeni taas tämänkin väittelyn... :-)

        Kommentoija vaihtui, etkö huomannut? Madrakehan sinä kuityenkin olet, vaikka oletkin pitäytynyt kertomasta morfisista kentistäsi tällä kertaa. Samaa umpityhmää pseudotieteellistä jankutusta kuin ennenkin.


    • Kuka olisi havainnoinut alkuräjähdystä, AP ?

      • Perustele miksi tarvitaan "havainnoitsija" romahduttamaan alkuräjähdyksen aaltofunktio
        (Onko joku sanonut, että se romahti silloin ja tällöin? - siis ennen kuin aivomme ymmärsivät kysyä tuollaisia asioita)


    Ketjusta on poistettu 4 sääntöjenvastaista viestiä.

    Luetuimmat keskustelut

    1. Mitä hittoa tapahtuu nuorille miehillemme?

      Mikä on saanut heidän päänsä sekaisin ja kadottamaan järjellisyytensä normaalista elämästä ja ryhtymään hörhöiksi? https
      Maailman menoa
      354
      4041
    2. En sitten aio sinua odotella

      Olen ollut omasta halustani yksin, mutta jossain vaiheessa aion etsiä seuraa. Tämä on aivan naurettavaa pelleilyä. Jos e
      Ikävä
      83
      1759
    3. Muistatko kun kerroin...

      että palelen..? Myös nyt on kylmä. Tahtoisin peittosi alle.
      Ikävä
      43
      1530
    4. Martina jättää triathlonin: "Aika kääntää sivua"

      Martina kirjoittaa vapaasti natiivienkusta suomeen käännetyssä tunteikkaassa tekstissä Instassaan. Martina kertoo olevan
      Kotimaiset julkkisjuorut
      61
      1497
    5. En vain ole riittävä

      Muutenhan haluaisit minut oikeasti ja tekisit jotain sen eteen. Joo, ja kun et varmaan halua edes leikisti. Kaikki on o
      Ikävä
      28
      1338
    6. Oon pahoillani että

      Tapasit näin hyödyttömän, arvottoman, ruman ja tylsän ihmisen niinku minä :(
      Ikävä
      58
      1315
    7. Kuka sinä oikeen olet

      Joka kirjoittelet usein minun kanssa täällä? Olen tunnistanut samaksi kirjoittajaksi sinut. Miksi et anna mitään vinkkej
      Ikävä
      51
      1312
    8. Persut vajosivat pinnan alle

      Sosiaali- ja terveysministeri Kaisa Juuson (ps) tietämättömyys hallinnonalansa leikkauksista on pöyristyttänyt Suomen ka
      Maailman menoa
      199
      1282
    9. Hei, vain sinä voit tehdä sen.

      Only you, can make this world seem right Only you, can make the darkness bright Only you and you alone Can make a change
      Ikävä
      6
      1190
    10. Taasko se show alkaa

      Koo osottaa taas mieltään
      Ikävä
      26
      1187
    Aihe